Sunteți pe pagina 1din 212

BRILLIANT PUBLIC SCHOOL,

SITAMARHI
(Affiliated up to +2 level to C.B.S.E., New Delhi)

Class IX
S.A.-I & II Maths
I.I.T.Foundation, N.T.S.E. & Olympiad
Study Package

Session: 2014-15

Office: Rajopatti, Dumra Road, Sitamarhi (Bihar), Pin-843301


Ph.06226-252314 , Mobile:9431636758, 9931610902
Website: www.brilliantpublicschool.com; E-mail: brilliantpublic@yahoo.com
Subjexct : Mathematics Class : IX

CONTENTS
S.No. Topics Page No.
1. Number System 1 - 35
2. Polynomials 36 - 56
3. Coordinate Geometry 57 - 62
4. Linear Equation in two Variable 63 - 71
5. Introduction of Euclid’s Geometry 72 - 77
6. Lines and Angles 78 - 88
7. Triangles 89 - 98
8. Quadrilateral 99 - 114
9. Area of parallelograms and triangle 115 - 128
10. Circle 129 - 155
11. Constructions 156 - 164
12. Heron’s Formula 165 - 172
13. Surface Area & Volume 173 - 181
14. Statistics 182 - 197
15. Probability 198 - 204
16. Proof In Mathematics 205 - 211
17. Mathematical Modeling 212 - 220

NUMBER SYSTEM
CLASSIFICATION OF NUMBERS
(I) Natural numbers:
Set of all non-fractional number from 1 to + ∞ , N = {1,2,3,4,....}.
(II) Whole numbers :
Set of numbers from 0 to + ∞ , W = {0,1,2,3,4,.....}.
(III) Integers :
Set of all-non fractional numbers from ∞ to + ∞ , I or Z = (...., -3,-2,-1,0,1,2,3,....}.
(IV) Rational numbers :
These are real numbers which can be expressed in the form of p/q, where p and q are integers and q ≠ 0.
e.g. 2/3, 37/15, -17/19.
 All natural numbers, whole numbers and integers are rational.
 Rational numbers include all Integers (without any decimal part to it), terminating fractions (fractions in
which the decimal parts terminating e.g. 0.75, - 0.02 etc.) and also non-terminating but recurring decimals
e.g. 0.666...., -2.333....., etc.
Fractions :

1
(a) Common fraction : Fractions whose denominator is not 10.
(b) Decimal fraction : Fractions whose denominator is 10 or any power of 10.
3
(c) Proper fraction : Numerator < Denominator i.e. .
5
5
(d) Improper fraction : Numerator > Denominator i.e. .
3
2
(e) Mixed fraction : Consists of integral as well as fractional part i.e. 3 .
7
2 /3
(f) Compound fraction : Fraction whose numerator and denominator themselves are fractions. i.e. .
5 /7
 Improper fraction can be written in the form of mixed fractions.
(v) Irrational Numbers :
All real number which are not rational are irrational numbers. These are non-recurring as well as non-

terminating type of decimal numbers e.g. 2 , 3 4 ,2 + 3 , 2 + 3 , 4 7


3 etc.

(vi) Real numbers : Number which can represent actual physical quantities in a meaningful way are known
as real numbers. These can be represented on the number line. Number line in geometrical straight line
with arbitrarily defined zero (origin).
(vii) Prime number : All natural numbers that have one and itself only as their factors are called prime
numbers i.e. prime numbers are exactly divisible by 1 and themselves. e.g. 2,3,5,7,11,13,17,19,23....etc. If P is
the set of prime number then P = {2,3,5,7....}.
(viii) Composite numbers : All natural number, which are not prime are composite numbers. If C is the set
of composite number then C = {4,6,8,9,10,12,.....}.
 1 is neither prime nor composite number.
(ix) Co-prime numbers : If the H.C.F. of the given numbers (not necessarily prime) is 1 then they are known
as co-prime numbers. e.g. 4, 9, are co-prime as H.C.F. of (4, 9) = 1.
 Any two consecutive numbers will always be co-prime.
(x) Even Numbers : All integers which are divisible by 2 are called even numbers. Even numbers are
denoted by the expression 2n, where n is any integer. So, if E is a set even numbers, then E = {...., -4, -2, 0, 2,
4,....}.
(xi) Odd Numbers: All integers which are not divisible by 2 are called odd numbers. Odd numbers are
denoted by the general expression 2n - 1 where n is any integer. If O is a set of odd numbers, then O = {...., -
5, -3, -1, 1, 3, 5,....}.
(xii) Imaginary Numbers: All the numbers whose square is negative are called imaginary numbers. e.g. 3i,

4i, i, ..... where i = −1 .


(xiii) Complex Numbers : The combined form of real and imaginary numbers is known as complex
numbers. It is denoted by Z = A + iB where A is real part and B is imaginary part of Z and A, B ∈ R.
 The set of complex number is the super set of all the sets of numbers.
IDENTIFICATION PRIME NUMBER
Step 1 : Find approximate square root of given number.

2
Step 2 : Divide the given number by prime numbers less than approximate square root of number. If given
number is not divisible by any of this prime number then the number is prime otherwise not.
Ex.1 571, is it a prime ?
Sol. Approximate square root of 571 = 24.
Prime number < 24 are 2, 3, 5, 7, 11, 13, 17, 19, & 23. But 571 is not divisible by any of these prime numbers
so 571 is a prime number.
Ex.2 Is 1 prime or composite number ?
Sol. 1 is neither prime nor composite number.
REPRESENTATIO FO RATIONAL NUMBER OF A REAL NUMBER LINE
(i) 3/7 Divide a unit into 7 equal parts.

13
(ii)
7
4
(iii) −
9
(a) Decimal Number (Terminating) :

(i) 2.5

(ii) 2.65 (process of magnification)

Ex.3 Visualize the representation of 5.37 on the number line upto 5 decimal place. i.e. 5.37777.

3
(b) Find Rational Numbers Between Two Integral Numbers :
Ex.4 Find 4 rational numbers between 2 and 3.
Sol. Steps :
(i) Write 2 and 3 multipling in Nr and Dr with (4+1).

2 × ( 4 + 1) 1 3 × ( 4 + 1) 15
(ii) i.e. 2 = &3 = =
( 4 + 1) 5 ( 4 + 1) 5

11 12 13 14
(iii) So, the four required numbers are , , , .
5 5 5 5
Ex.5 Find three rational no’s between a and b (a < b).

Sol. a<b
⇒ a+a<b+a
⇒ 2a < a + b
a+b
⇒ a<
2
Again, a < b
⇒ a + b < b + b.
⇒ a + b < 2b
a+b
⇒ < b.
2
a+b
∴ a< < b.
2
a+b
i.e. lies between a and b.
2
a+b
Hence 1st rational number between a and b is .
2
For next rational number
a+b 2a + a + b
a+
2 = 2 3a + b 3a + b a + b
= ∴ a< < < b.
2 2 4 4 2
a+b
+b
2 a + b + 2 b a + 3b
Next, = =
2 2×2 4
3a + b a + b a + 3b
∴ a< < < < b , and continues like this.
4 2 4

1 1
Ex.6 Find 3 rational numbers between & .
3 2

1 1 2+3
+
5 1 5 1
Sol. 1st Method 3 2 = 6 = ∴ , ,
2 2 12 3 12 2

1 5 4+5
+
3 12 = 12 = 9 1 9 5 1
= ∴ , , ,
2 2 24 3 24 12 2

5 1 5 6
+ +
12 2 = 12 12 = 11 1 9 5 11 1
= ∴ , , , , .
2 2 24 3 24 12 24 2

4
8 9 10 11 12  8 1 1
Verify : < < < < . as = & 
24 24 24 24 24  24 3 2 

2nd Method : Find n rational numbers between a and b (a < b).

b−a
(i) Find d = .
n+1

(ii) 1st rational number will be a + d.


2nd rational number will be a + 2d.
3rd rational number will be a + 3d and so on....
nth rational number is a + nd.
3 4
Ex.7 Find 5 rational number between and
5 5
4 3

3 4 b−a 5 5 1 1 1
Here, a = , b = d = = = × = .
5 5 n+1 5+1 5 6 30

3 1 19 3 2
1st = a + b = + = , 2nd = a + 2d = + ,
5 30 20 5 30

3 3 21 3 4 22
3rd = a + 3d = + = , 4th = a + 4d = + = ,
5 30 30 5 30 30

3 5 23
5th = a + 5d = + = .
5 30 30

RATIONAL NUMBER IN DECIMAL REPRESENTATION


(a) Terminating Decimal :
1
In this a finite number of digit occurs after decimal i.e. = 0.5, 0.6875, 0.15 etc.
2
(b) Non-Terminating and Repeating (Recurring Decimal) :
In this a set of digits or a digit is repeated continuously.
2
Ex.8 = 0.6666 − − − − − −− = 0.6 .
3
5
Ex.9 = 0.454545 − − − − − − = 0.45 .
11
PROPERTIES OF RATIONAL NUMBER
If a,b,c are three rational numbers.
(i) Commutative property of addition. a + b = b + a(ii) Associative property of addition (a+b)+c = a+(b+c)
(iii) Additive inverse a + (-a) = 0 0 is identity element, -a is called additive inverse of a.
(iv) Commutative property of multiplications a.b. = b.a. (v) Associative property of multiplication (a.b).c = a.(b.c)

1
(vi) Multiplicative inverse (a ) ×   = 1
a

5
1
1 is called multiplicative identity and is called multiplicative inverse of a or reciprocal of a.
a
(vii) Distributive property a.(b+c) = a.b + a.c
∴ L.H.S. ≠ R.H.S.

Hence in contradicts our assumption that 2 + 3 rational.

∴ 2+ 3 is irrational.

Ex.9 Prove that 3 − 2 is an irrational number

Sol. Let 3 − 2 = r where r be a rational number


Squaring both sides

⇒ ( 3− 2 )
2
= r2

⇒ 3 + 2 - 2 6 = r2

⇒ 5 - 2 6 = r2

Here, 5 - 2 6 is an irrational number but r2 is a rational number

∴ L.H.S. ≠ R.H.S.

Hence it contradicts our assumption that 3 − 2 is a rational number.

(b) Irrational Number in Decimal Form :

2 = 1.414213 ...... i.e. it is not-recurring as well as non-terminating.

3 = 1.732050807 ...... i.e. it is non-recurring as well as non-terminating.


Ex.10 Insert an irrational number between 2 and 3.

Sol. 2×3 = 6
Ex.11 Find two irrational number between 2 and 2.5.

Sol. 1st Method : 2 × 2.5 = 5

Since there is no rational number whose square is 5. So 5 is irrational..

Also 2 × 5 is a irrational number.

2nd Method : 2.101001000100001.... is between 2 and 5 and it is non-recurring as well as non-terminating.


Also, 2.201001000100001......... and so on.

Ex.12 Find two irrational number between 2 and 3.

4
Sol. 1st Method : 2× 3 = 6 =46 Irrational number between 2 and 6

2 ×4 6 = 4 2 ×8 6

2nd Method : As 2 = 1.414213562 ...... and 3 = 1.732050808......

As , 3 > 2 and 2 has 4 in the 1st place of decimal while 3 has 7 is the 1st place of decimal.

∴ 1.501001000100001......., 1.601001000100001...... etc. are in between 2 and 3

6
Ex.13 Find two irrational number between 0.12 and 0.13.
Sol. 0.1201001000100001......., 0.12101001000100001 .......etc.
Ex.14 Find two irrational number between 0.3030030003..... and 0.3010010001 .......
Sol. 0.302020020002...... 0.302030030003.... etc.
Ex.15 Find two rational number between 0.2323323332..... and 0.252552555255552.......
Sol. 1st place is same 2.
2nd place is 3 & 5.
3rd place is 2 in both.
4th place is 3 & 5.
Let a number = 0.25, it falls between the two irrational number.
Also a number = 0.2525 an so on.
(c) Irrational Number on a Number Line :

Ex.16 Plot 2 , 3 , 5 6 on a number line.


Sol.

Another Method for :


(i) Plot 2, 3

So, OC = 2 and OD = 3
(ii) Plot 5, 6, 7 8

OC = 5

OD = 6 OH = 7 .......

(d) Properties of Irrational Number :


(i) Negative of an irrational number is an irrational number e.g. − 3 − 4 5 are irrational.

(ii) Sum and difference of a rational and an irrational number is always an irrational number.

(iii) Sum and difference of two irrational numbers is either rational or irrational number.

(iv) Product of a non-zero rational number with an irrational number is either rational or irrationals

(v) Product of an irrational with a irrational is not always irrational.


Ex.17 Two number’s are 2 and 3 , then

Sum = 2 + 3 , is an irrational number.


Difference = 2 - 3 , is an irrational number.
Also 3 − 2 is an irrational number.

7
3
Ex.18 Two number’s are 4 and 3 , then
3
Sum = 4 + 3 , is an irrational number.
3
Difference = 4 - 3 , is an irrational number.

Ex.19 Two irrational numbers are 3 ,− 3 , then

Sum = ( )
3 + − 3 = 0 which is rational.

Difference = 3 − (− 3 ) = 2 3 , which is irrational.

Ex.20 Two irrational numbers are 2 + 3 and 2 - 3 , then

( ) ( )
Sum = 2 + 3 + 2 − 3 = 4 , a rational number

Two irrational numbers are 3 + 3m 3 − 3

Difference = 3 + 3 − 3 + 3 = 6 , a rational number

Ex.21 Two irrational numbers are 3 − 2 , 3 + 2 , then

Sum = 3 − 2 + 3 + 2 = 2 3 , an irrational

Ex.22 2 is a rational number and 3 is an irrational.

2 × 3 = 2 3 , an irrational.

Ex.23 0 a rational and 3 an irrational.

0× 3 = 0, a rational.

4 4 4
Ex.24 × 3= 3= is an irrational.
3 3 3

Ex.25 3 × 3 = 3 × 3 = 9 = 3 a rational number.

Ex.26 2 3 × 3 2 = 2 × 3 3 × 2 = 6 6 and irrational number.

Ex.27 3
3 × 3 3 2 = 3 3 × 3 2 = 3 3 3 = 3 a rational number.

Ex.28 (2 + 3 )(2 − 3 ) = (2) − ( 3 ) 2 2


= 4 − 3 = 1 a rational number.

8
Ex.29 (2 + 3 )(2 + 3 ) = (2 + 3 ) 2

= (2) + ( 3 ) ( 3)
2 2
+ 2( 2 ) ×

= 4+3+4 3

= 7 + 4 3 an irrational number

NOTE :

(i) − 2 ≠ − 2 , it is not a irrational number.

(ii) −2× −3 ≠ ( − 2 × −3 = 6 )
Instead − 2 , − 3 are called Imaginary numbers.

− 2 = i 2 , where i ( = iota) = −1
∴ (A) i2 = -1
(B) i3 = i2 × i = (-1) × i = -i
(C) i4 = i2 × i2 = (-1) × (-1) = 1
(iii) Numbers of the type (a + ib) are called complex numbers where (a, b) ∈ R.e.g. 2 + 3i, -2 + 4i, -3i, 11 - 4i,
are complex numbers.

GEOMETRICAL REPRESENTATION OF REAL NUMBERS

To represent any real number of number line we follows the following steps :
STEP I : Obtain the positive real number x (say).
STEP II : Draw a line and mark a point A on it.
STEP III : Mark a point B on the line such that AB = x units.
STEP IV : From point B mark a distance of 1 unit and mark the new point as C.
STEP V : Find the mid - point of AC and mark the point as O.
STEP VI : Draw a circle with centre O and radius OC.
STEP VII : Draw a line perpendicular to AC passing through B and intersecting the semi circle at D.

Length BD is equal to x.

9
4. Examine whether the following numbers are rational or irrational :

3 −1
(
(i) 2 − 3 )
2
(ii) ( 2+ 3 )
2
( )(
(iii) 3 + 2 3 − 2 ) (iv)
3 +1

5. Represent 8.3 on the number line.

6. Represent ( 2 + 3 ) on the number line.

7. Prove that ( )
2 + 5 is an irrational number.

8. Prove that 7 is not a rational number.

9. Prove that ( 2 + 2 ) is an irrational number.

10. Multiply 27 a 3b 2 c 4 × 3 128a7 b9 c 2 × 6 729ab12 c2 .

11. Express the following in the form of p/q.

(i) 0.3 (ii) 0.37 (iii) 0.54 (iv) 0.05 (v) 1.3 (vi) 0.621

12. Simplify : 0.4 + .018

NUMBER SYSTEM
SURDS
n
Any irrational number of the form a is given a special name surd. Where ‘a’ is called radicand, it should

always be a rational number. Also the symbol n is called the radical sign and the index n is called order

of the surd.
1
n
a is read as ‘nth root a’ and can also be written as a n .
(a) Some Identical Surds :
3
(i) 4 is a surd as radicand is a rational number.

Similar examples 3
5 , 4 12 , 5 7 , 12 ,.........

(i) 2 3 is a surd (as surd + rational number will give a surd)

Similar examples 3 + 1, 3 3 + 1,....

10
(iii) 7 − 4 3 is a surd as 7 - 4 3 is a perfect square of 2 − 3 ( )
Similar examples 7 + 4 3 , 9 − 4 5 , 9 + 4 5 ,.........

1
 1 3 1
(i) 3
3 is a surd as 3
3 = 32  = 36 = 6 3
 
 
3 3
Similar examples 5 , 4 5 6 ,....... ...

(b) Some Expression are not Surds :

(i) 3 8 because 3
8 = 3 2 3 = 2 , which is a rational number.

(ii) 2 + 3 because 2 + 3 is not a perfect square.

3
(iii) 1 + 3 because radicand is an irrational number.

LAWS OF SURDS

(i) ( a)n n
= n an = a

e.g. (A) 3
8 = 3 23 = 2 (B) 4
81 = 4 3 4 = 3
n
(ii) a × n b = n ab [Here order should be same]
3
e.g. (A) 2 × 3 6 = 3 2 × 6 = 3 12
3
but, 3 × 4 6 ≠ 3×6 [Because order is not same]
1st make their order same and then you can multiply.

a
(iii) n
a +nb =n
b

nm
(iv) a = nm
a =mna e.g. = 2 =88
n×p
(v) n
a= ap [Important for changing order of surds]

n n ×p
or, am = a m ×p

3 3
e.g. 6 2 make its order 6, then 6 2 = 3× 2 6 2× 2 = 6 6 4 .

e.g. 3
6 make its order 15, then 3
6 = 3×5 61×5 = 15 6 5 .

11
OPERATION OF SURDS
(a) Addition and Subtraction of Surds :
Addition and subtraction of surds are possible only when order and radicand are same i.e. only for surds.
Ex.1 Simplify

(i) 6 − 216 + 96 = 15 6 − 6 2 × 6 + 16 × 6 [Bring surd in simples form]

= 15 6 − 6 6 + 4 6

= (15 - 6 + 4 ) 6

= 13 6 Ans.

(ii) 53 250 + 7 3 16 − 143 54 = 53 125 × 2 + 7 3 8 × 2 − 143 27 × 2

= 5 × 53 2 + 7 × 2 3 2 − 14 × 3 × 3 2

= ( 25 + 14 − 42 )3 2

= −33 2 Ans.

(ii) 53 250 + 7 3 16 − 143 54 = 53 125 × 2 + 7 3 8 × 2 − 143 27 × 2

= 5 × 53 2 + 7 × 2 3 2 − 14 × 3 × 3 2

= ( 25 + 14 − 42 )3 2

= −33 2 Ans.

5 1 5 1× 3
(iii) 4 3 + 3 48 − = 4 3 + 3 16 × 3 −
2 3 2 3× 3
5 1
= 4 3 + 3× 4 3 − × 3
2 3
5
= 4 3 + 12 3 − 3
6

 5
=  4 + 12 −  3
 6
91
= 3 Ans.
6

(b) Multiplication and Division of Surds :


3
Ex.2 (i) 3
4 × 3 22 = 3 4 × 22 = 2 3 × 11 = 2 3 11
12
(i) 3
2 ×4 3 = 2 4 × 12 3 3 = 12 2 4 × 3 3 = 12 16 × 27 = 12
432

Ex.3 Simplify 8a 5b × 3 4a 2 b 2

6
Hint : 83 a15 b 3 × 6 4 2 a 4 b 4 = 6 2 13 a19 b7 = 6 2ab . Ans.

12
24 6
( 24)3 216
Ex.4 Divide 24 ÷ 3 200 = 3
= =6 Ans..
200 6
( 200) 2 625

(c) Comparison of Surds :


n
It is clear that if x > y > 0 and n > 1 is a positive integer then x >ny.

Ex.5 3
16 > 3 12 , 5 35 > 5 25 and so on.
Ex.6 Which is greater is each of the following :

3 5 1 1
(i) 16 and 8 (ii) and 3
2 3
L.C.M. of 3 and 5 15. L.C.M. of 2 and 3 is 6.
3 2
1 1
3
6 = 3× 5 6 5 = 15 7776 6   and 3  
2 3

1 1  1 1
5
8 = 3×5 85 = 15 512 6 and 6 As 8 < 9 ∴ 8 > 9 
8 9

75 1 6 1
∴ 7776 > 15 512 so, 6 >
8 9

3 1 1
⇒ 6 >58 ⇒ > 3
2 3

Ex.7 Arrange 2 , 3 3 and 4


5 is ascending order.
Sol. L.C.M. of 2, 3, 4 is 12.

∴ 2 = 2×6 2 6 = 12 64

3× 4
3
3= 3 4 = 12 81

4× 3
4
5= 53 = 12 125
As, 64 < 81 < 125.
12
∴ 64 < 12 81 < 12 125

⇒ 2 <33<45

Ex.8 Which is greater 7 − 3 or 5 − 1?

( 7 − 3 )( 7 + 3 ) 7−3 4
Sol. 7− 3= = =
( 7 + 3) 7+ 3 7+ 3

( 5 − 1)( 5 + 1) 5−1 4
And, 5 −1 = = =
( 5 + 1) 5 +1 5 +1

Now, we know that 7 > 5 and 3 > 1 , add

So, 7 + 3 > 5 +1

13
1 1 4 4
⇒ < ⇒ < ⇒ 7 − 3 < 5 −1
7+ 3 5 +1 7+ 3 5 +1

So, 5 −1 > 7 − 3
RATIONALIZATION OF SURDS
Rationalizing factor product of two surds is a rational number then each of them is called the rationalizing
factor (R.F.) of the other. The process of converting a surd to a rational number by using an appropriate
multiplier is known as rationalization.
Some examples :

(i) R.F. of a is a (∴ a× a =a . )
(ii) R.F. of 3
a is 3 a 2 ∴ 3 a × 3 a 2 = 3 a 3 = a  .
 

(iii) R.F. of a + b is a − b & vice versa ∴ [ ( a + b )( a − b ) = a − b].


(iv) R.F. of a + b is a − b & vice versa [∴ (a + b )(a − b ) = a − b] 2

(v) R.F. of 3
a + 3 b is 
3
a − ab + b  ∴ ( a + b ) a − ab + b 
2 3 3 2 3 3 3 2 3 3 2
   

∴ ( a) + ( b)
3 3 3 3
= a + b which is rational.
 

(vi) R.F. of ( a + b + c is ) ( ) (
a + b − c nd a + b − c + 2 ab . )
Ex.9 Find the R.G. (rationalizing factor) of the following :
3 3 4
(i) 10 (ii) 12 (iii) 162 (iv) 4 (v) 16 (vi) 162 (vii) 2 + 3
3
(viii) 7 − 4 3 (ix) 3 3 + 2 2 (x) 3 +3 2 (xi) 1 + 2 + 3

(i) 10

Sol. [∴ 10 × 10 = 10 × 10 = 10] as 10 is rational number.

∴ R.F. of 10 is 10 Ans.

(ii). 12 Sol. First write it’s simplest from i.e. 2 3 .

Now find R.F. (i.e. R.F. of 3 is 3)

∴ R.F. of 12 is 3 Ans.

(iii) 162 Sol. Simplest from of 162 is 9 2 . R.F. of 2 is 2.

∴ R.F. of 162 is 2 Ans.

3 3 3
(iv) 3
4 Sol. 3
4 × 42 = 43 = 4 ∴ R.F. of 3
4 is 42 Ans.
3
(v). 16

3
Sol. Simplest from of 3
16 is 2 3 2 Now R.F. of 3
2 is 22

3
∴ R.F. of 3
16 is 22 Ans.

14
4
(vi) 162

Sol. Simplest form of 4


162 is 3 4 2

Now R.F. of 4
2 is 4 2 3
4
R.F. of ( 4 162 ) is 23 Ans.

(vii) 2+ 3

Sol. ( )(
As 2 + 3 2 − 3 = ( 2 )2 − ) ( 3) 2
= 4 − 3 = 1, , which is rational.

∴ R.F. of ( 2 + 3 ) is ( 2 − 3 ) Ans.

(viii) 7−4 3

Sol. ( )(
As 7 − 4 3 7 + 4 3 = (7 )2 − 4 − 3 ) ( )
2
= 49 - 48 = 1, which is rational

∴ R.F. of (7 − 4 3 ) is 7 + 4 3 ( ) Ans.

(ix). 3 3 +2 2

Sol. ( )(
As 3 3 + 2 2 3 3 − 2 2 = 3 3 ) ( ) − (2 2 ) 2 2
= 27 − 8 = 19 , which is rational.

∴ R.F. of ( 3 3 + 2 2 ) is ( 3 3 − 2 2 ) Ans.

(x) 3
3 +3 2

( 3 + 3 2  3 32 − 3 3 × 3 2 + 2 2  =  3 33 + 2 3  = 3 + 2 = 5, which is rational.
)
3 3 3
Sol. As
   

∴ R.F. of ( 3 3 + 3 2 ) is  3 32 − 3 3 × 3 2 + 3 2 2  Ans.
 

(xi) 1+ 2 + 3

Sol. (1 + )(
2 + 3 1+ 2 − 3 = 1+ 2 ) ( ) − ( 3)
2 2

+ ( 2 ) + 2( 1)( 2 ) − 3
2
= 1)2

= 1+2+2 2 −3

= 3+2 2 −3

=2 2

2 2 × 2 = 2×2 = 4

∴ R.F. of 1 + 2 + 3 is 1 + 2 − 3 and ( ) 2 . Ans.

NOTE : R.F. of a + b or a − b type surds are also called conjugate surds & vice versa.

15
Ex.10 (i) 2 − 3 is conjugate of 2 + 3

(ii) 5 + 1 is conjugate of 5 −1
NOTE : Sometimes conjugate surds and reciprocals are same.

Ex.11 (i) 2 + 3 , it’s conjugate is 2 − 3 , its reciprocal is 2 − 3 & vice versa.

(ii) 5 − 2 6 , it’s conjugate is 5 + 2 6 , its reciprocal is 5 − 2 6 & vice versa.

(iii) 6 − 35 ,6 + 35

(iv) 7 − 4 3 , 7 + 4 3

(v) 8 + 3 7 , 8 − 3 7 ............... and so on.

Ex.12 Express the following surd with a rational denominator.

8 8  ( 15 + 1) + ( 5+ 3  )
Sol.
15 + 1 − 5 − 3
=
[( 15 + 1) − ( 15 + 3 )] ×  ( 15 + 1) + ( 
)
5 + 3 

8( 15 + 1 + 5 + 3 )
=
( 15 + 1) − ( 5 + 3 )
2 2

8( 15 + 1 + 5 + 3 )
=
15 + 1 + 2 15 − (5 + 3 + 2 15 )

8( 15 + 1 + 5 + 3 )
=
8

= ( 15 + 1 + 5 + 3 ) Ans.

a2
Ex.13 Rationalize the denominator of
a2 + b2 + b

a2 a2 a2 + b2 − b
Sol. = ×
a 2 + b2 + b a 2 + b2 + b a2 + b2 − b

a 2  a 2 + b 2 − b 
=  
2
 a 2 + b 2  − ( b)2
 

a 2  a 2 + b 2 − b 
= 2  =  a2 + b2 − b 
  Ans.
a + b2 − b 2  

16
3+2 2
Ex.14 If = a + b 2 , where a and b are rational then find the values of a and b.
3− 2

3 + 2 2 ( 3 + 2 2 )( 3 + 2 )
Sol. L.H.S. =
3− 2 ( 3 − 2 )(3 + 2 )

9+3 2 +6 2 +4
=
9−2

13 + 9 2
=
7
13 9
= + 2
7 7
13 9
∴ + 2 = a+b 2
7 7

Equating the rational and irrational parts


13 9
We get a = ,b = Ans.
7 7
1
Ex.15 If 3 = 1.732 , find the value of
3 −1

1 1 3 +1
Sol. = ×
3 −1 3 −1 3 +1

3 +1
=
3−1

3 +1
=
2
1.732 + 1
=
2
2.732
=
2
= 1.366 Ans.

Ex.16 If 5 = 2.236 and 2 = 1.414, then


3 4
Evaluate : +
5+ 2 5− 2

3 4 3 5 − 2 + 4( 5 + 2 )
Sol. + =
5+ 2 5− 2 ( 5 − 2 )( 5 + 2 )

3 5 −3 2 +4 5 +4 2
=
5−2

7 5+ 2
=
5−2

17
7 5+ 2
=
3
7 × 2.236 + 1.414
=
3
15.652 + 1.414
=
3
17.066
=
3
= 5.689 (approximate)
1
Ex.17 If = find the value of x3 - x2 - 11x + 3.
2+ 3
1
Sol. As, x = =2− 3
2+ 3

⇒ x-2=- 3

⇒ (x - 2)2 = − 3 ( )
2
[By squaring both sides]

⇒ x2 + 4 - 4x = 3
⇒ x2 - 4x + 1 = 0
Now, x3 - x2 - 11x + 3 = x3 - 4x2 + x + 3x2 - 12x + 3
= x (x2 - 4x + 1) + 3 (x2 - 4x + 1)
= x(0) + 3 (0)
=0+0=0 Ans.
1
Ex.18 If x = 3 - 8 , find the value of x 3 + .
x3

Sol. x=3- 8
1 1
∴ =
x 3− 8
1
⇒ = 3+ 8
x
1
Now, x+ = 3− 8 +3+ 8 = 6
x
3
1  1 1 1
⇒ x3 + 3
=  x +  − 3x  x + 
x  x x x
1
⇒ x3 + = (6 )3 − 3( 6)
x3
1
⇒ x3 + = 216 − 18
x3
1
⇒ x3 + = 198 Ans.
x3

18
Ex.19 If x = 1 + 21/3 + 2 2/3, show that x3 - 3x2 - 3x - 1 = 0

Sol. x = 1 + 21/3 + 2 2/3


⇒ x - 1 (2 1/3 + 22/3)
⇒ (x - 1)3 = (2 1/3 + 22/3)3
⇒ (x - 1)3 = (21/3) + (2 2/3)3 + 3.21/3.2 2/3(2 1/3 + 2 1/3-)
⇒ (x - 1)3 = 2 + 22 + 3.21 (x - 1)
⇒ (x - 1)3 = 6 + 6 (x - 1)
⇒ x3 - 3x2 + 3x - 1 = 6x
⇒ x3 - 3x2 - 3x - 1 = 0 Ans.

Ex.20 Solve : x + 3 + x − 2 = 5.

Sol. x+3 = 5− x−2


⇒ ( x+3 ) = (5 −
2
x−2 )
2
[By squaring both sides]
⇒ x + 3 = 25 + (x - 2) - 10 x − 2
⇒ x + 3 = 25 + x - 2 - 10 x − 2
⇒ 3 - 23 = - 10 x − 2
⇒ -20 = -10 x − 2
⇒ 2 = x−2
⇒ x-2=4 [By squaring both sides]
⇒ x=6 Ans.

Ex.21 If x = 1 + 2 + 3 , prove that x4 - 4x3 - 4x2 + 16 - 8 = 0.

Hint : x = 1 + 2 + 3
⇒ x-1= 2 + 3
⇒ ( x − 1)2 = ( 2+ 3 )
2
[By squaring both sides]
⇒ x2 + 1 - 2x = 2 + 3 + 2 6
⇒ x2 - 2x - 4 = 2 6
⇒ (x2 - 2x - 4)2 = ( 2 6 )2
⇒ x4 + 4x2 + 16 - 4x3 + 16x - 8x2 = 24
⇒ x4 - 4x3 - 4x2 + 16x + 16 - 24 = 0
⇒ x4 - 4x3 - 4x2 + 16x - 8 = 0 Ans.

EXPONENTS OF REAL NUMBER


(a) Positive Integral Power :
For any real number a and a positive integer ‘n’ we define an as :
an = a × a × a × ............... x a (n times)
an is called then nth power of a. The real number ‘a’ is called the base and ‘n’ is called the exponent of the nth
power of a.
e.g. 2 3 = 2 × 2 × 2 = 8

NOTE : For any non-zero real number ‘a’ we define a0 = 1.


0
3
e.g. thus, 3 0 = 1, 50,   = 1 and so on.
4

19
(b) Negative Integral Power :
1
For any non-zero real number ‘a’ and a positive integer ‘n’ we define a − n =
an
Thus we have defined an find all integral values of n, positive, zero or negative. an is called the nth power of
a.
RATIONAL EXPONENTS OR A REAL NUMBER
(a) Principal of nth Root of a Positive Real Numbers :
If ‘a’ is a positive real number and ’n’ is a positive integer, then the principal nth root of a is the unique
positive real number x such that xn = a.
n
The principal nth root of a positive real number a is denoted by a1/n or a.
(b) Principal of nth Root of a Negative Real Numbers :
If ‘a’ is a negative real number and ‘n’ is an odd positive integer, then the principle nth root of a is define as -
|a|1/n i.e. the principal nth root of -a is negative of the principal nth root of |a|.
Remark :
It ‘a’ is negative real number and ‘n’ is an even positive integer, then the principle nth root of a is not
defined, because an even power of real number is always positive. Therefore (-9)1/2 is a meaningless
quantity, if we confine ourselves to the set of real number, only.
(c) Rational Power (Exponents) :
p
For any positive real number ‘a’ and a rational number ≠ where q ≠ 0 , we define ap / q = (a p )1 / q i.e. ap/q
q

is the principle qth root of ap.


LAWS OF RATIONAL EXPONETNS
The following laws hold the rational exponents
(i) a m × an = am+1 (ii) am ÷ an = am-n
1
(ii) (am)n = amn (iv) a − n =
an

(v) am/n = (am)1/n = (a1/n)m i.e. am/n = n


am = ( a)
n m
(vi) (ab)m = ambm
m
a am
(vii)   = (viii) abn = a b+b+b….n tmes
b bm
Where a,b are positive real number and m,n are relational numbers.
ILLUSTRATIONS :

Ex.22 Evaluate each of the following:


3 −3
 11  3
(i) 5 2 × 54 (ii) 58 ÷ 53 ( )
(iii) 32
2
(iv)  
 12 
(v)  
4

20
Sol. Using the laws of indices, we have

(i) 5 2.54 = 5 2+4 = 56 = 15625 ∵a m × a n = a m + n

58
(ii) 58 ÷ 5 3 = = 58 − 3 = 5 5 = 3125 ∵a m + a n = am − n
53

( )
(iii) 32
3
= 32×3 = 36 = 729 ∵(am )n = am× n
3 m
 11  113 1331 a am
(iv)   = 3 = ∵  =
 12  12 1728 b bm
−3
3 1 1 1 64 1
(v)   = = = = ∵a − n =
4 3
3
33 27 27 an
  64
4 43

Ex.23 Evaluate each of the following :


4 2 3 5 4 −1
 2   11   3  1 −2 3
(i)   ×   ×   (ii)   ×   × 
 11   3   2  2  3  5
3 −3 2
2 2 3
(iii) 2 55 × 2 60 − 2 97 × 2 18 (iv)   ×   ×  
   
3 5 5

Sol. We have.
4 2 3
 2   11   3  2 4 112 33
(i)   ×   ×   = 4 × 2 × 3
 11   3   2  11 3 2
2×3
=
112
6
= Ans.
121
(ii) We have,

1
5 4 −1 5 4  
1 −2 3 1 −2  3 
  ×  ×  =   ×  ×
2  3  5 2  3   5 
 
 

15 (− 2 )2 5
= × 4 ×
25 3 3
1 × 16 × 5
=
32 × 81 × 3
5
=
2 × 81 × 3
5
= Ans.
486

21
(iii) We have,

2 55 × 2 60 − 2 97 × 2 18 = 55 + 60 −2 97 + 18

= 2 15 − 2 115
=0 Ans.
(iv) We have,
3 −3 −2
2 2 3 23 1 32
  ×  ×  = 3
× × 2
   
3 5 5 3 (2 / 5) 5
23 1 32
= × ×
3 3 2 3 / 5 3 52

2 3 × 53 × 32
=
3 3 × 2 3 × 52
5
= Ans.
3
Ex.24 Simplify :

(25)3 / 2 × (243)3 / 5 16 × 2 n + 1 − 4 × 2 n
(i) (ii)
(16 )5 / 4 × (8)4 / 3 16 × 2 n + 2 − 2 × 2 n + 2

Sol. We have,

(i)
(25 )3 / 2 × (243 )3 / 5
=
(5 ) 2 3/2
( )
× 35
3/5

(16 )5 / 4 × (8)4 / 3 (2 ) 4 5 /4
× (2 )3 4 /3

52 ×3 / 2 × 35× 3 / 5
=
2 4× 5 / 4 × 2 3× 4 / 3

53 × 33
=
25 × 24
125 × 27
=
32 × 16
3375
= Ans.
512

16 × 2 n + 1 − 4 × 2 n 24 × 2n +1 − 22 × 2n
(ii) =
16 × 2 n + 2 − 2 × 2 n + 2 24 × 2n+2 − 2 × 2n+2

2n+5 − 2n+2
=
2n +6 − 2n +3

2n +5 − 2n +2
=
2.2 n + 5 − 2.2 n + 2

2n+5 − 2n+2 1
= = Ans.
(
2 2n +5 − 2n +2 2 )

22
 81 
−3 / 4  25  −3 / 2
5 
−3
Ex.25 Simplify   ×   ÷  
 16   9   2  

Sol. We have

−3 / 4  25  − 3 / 2  5  − 3   3 4 
−3 / 4  52 
−3 / 2 −3 
 81  5
  ×   ÷    =  4  ×  2 
 ÷  
 16   9   2    2   3  2 
 
−3 / 4 −3 / 2
 3  4   5  2   5  − 3 
=    ×    ÷   
 2    3    2  

3
4x −3 / 4  5  2 x − 3 / 2  5  −3 
=  ×   ÷  
2  3   2  

−3 
5 
−3 −3
3 5
=   ×   ÷   
2  3   2  

 2   5  5 
3 −3 −3
=   ×   ×   
 3   3   2  

2 3  33 2 3 
= × ÷ 
33  53 53 

2 3  33 53 
= × × 
33  53 2 3 

= 1 Ans.

23
EXERCISE
OBJECTIVE DPP - 3.1

1 1
1. If x= 3 + 8 and y = 3 - 8 then + =
x2 y2

(A) -34 (B) 34 (C) 12 8 (D) -12 8

3+ 7
2. If = a + b 7 then (a,b) =
3− 7
(A) (8, -3) (B) (-8, -3) (C) (-8, 3) (D) (8, 3)

5 −2 5 +2
3. − =
5 +2 5 −2

(A) 8 5 (B) − 8 5 (C) 6 5 (D) − 6 5

3+ 2 x−y
4. If x = and y = 1, the value of is :
3− 2 x − 3y

5 5 6 −4 6+4
(A) (B) (C) (D)
5 −4 6+4 5 5
5. Which one is greatest in the following :
3 3 3
(A) 2 (B) 3 (C) 4 (D) 2

6. The value of 5
(32 )−3 is :
(A) 1/8 (B) 1/16 (C) 1/32 (D) None

3− 2 3+ 2
7. If x = and y = the value of x2 + xy + y2 is :
3+ 2 3− 2

(A) 99 (B) 100 (C) 1 (D) 0

2 1 3
8. Simplify : + −
5+ 3 3+ 2 5+ 2
(A) 1 (B) 0 (C) 10 (D) 100
9. Which of the following is smallest ?
4 5
(A) 5 (B) 4 (C) 4 (D) 3

24
3
10 The product of 3 and 5 is :
6 6 6 6
(A) 375 (B) 675 (C) 575 (D) 475

11. The exponential from of 2 × 2 × 2 is :


(A) 21/16 (B) 83/4 (C) 23/4 (D) 81/2
12. The value of x, if 5 x-3 . 32x-8 = 225, is :
(A) 1 (B) 2 (C) 3 (D) 5
5
13. If 25x ÷ 2x = 2 20 then x =
1
(A) 0 (B) - 1 (C) (D) 1
2

14. 3
(729 )2.5 =
1
(A) (B) 81 (C) 243 (D) 729
81

4 3
15. x2 =
1 1 1
(A) x (B) x 2 (C) x 3 (D) x 6
SUBJECTIVE DPP - 3.2
1. Arrange the following surds in ascending order of magnitude :

(i) 4 10 ,3 6 , 3 (ii) 3 4 ,4 5 , 3

2. Whish is greater :

17 − 12 or 11 − 6 .

8
3. Simplify : .
15 + 1 − 5 − 3

4+ 2
4. If p and q are rational number and p − q = find p and q.
3+ 2

5. Find the simplest R.F. of :

(i) 3
32 (ii) 3
36 (iii) 2 3 / 5

6. Retionalise the denominator :

3 2+ 5
(i) (ii)
5 3

25
7. Retionalise the denominator and simplify :

3− 2 1+ 2 4 3 +5 2
(i) (ii) (iii)
3+ 2 3+2 2 48 + 18
8. Simplify :

5+ 3 5− 3 7+3 5 7−3 5
(i) + (ii) −
5− 3 5+ 3 3+ 5 3− 5

9. Find the value of a and b

11 − 7 5+ 6
(i) = a − b 77 (ii) =a+b 6
11 + 7 5− 6

3 +1
10. If x = find the value of 4x3 + 2x2 - 8x + 7.
2

5 − 21  1   1   1
11. If x = show that  x 3 + 3  − 5 x 2 + 2  +  x +  = 0.
2  x   x   x

a+2 + a−2
12. Show that a = x + 1/x, where x = .
a+2 − a−2
1 1 1 1 1
13. Prove that : − + − + = 5.
3− 8 8− 7 7− 6 6− 5 5 −2

5+ 2 5− 2
14. If x = and y = find the value of 3x2 + 4xy - 3y2.
5− 2 5+ 2
15. Evaluate:

5 +2 + 5 −2
− 3−2 2.
5 +1

16. If x= 3, b = 4 then find the values of :


(i) a b + ba (ii) a a + bb (iii) ab - ba
17. Simplify :

( x) −2 / 3
y 4 ÷ xy − 1 / 2 .

18. Simplify :
1

[
(i) 16 − 1 / 5 ]
5/2
(ii) [0.001]
3

19. If
[
9n × 32 × 3 −−n / 2 ]
−2
− ( 27 )n
=
1
, then prove than m - n = 1.
33m × 2 3 27

20. Find the value of x, if 5 x-3(2x-3) = 625.

26
ANSWER KEY

(Objective DPP # 1.1)

Qus. 1 2 3 4 5 6

Ans. A B C B D C
(Subjective DPP # 1.2)
1. (i) Non-terminating and repeating (ii) Non-terminating and non-repeating
(iii) Non-terminating and repeating (iv) Terminating
−7 −4 −3 −5 −11
2. , , , ,
6 3 2 3 6

3. -4, -3, -2, -1

−5 22 23 24 25 26 27
4. 5. , , , , ,
24 7 7 7 7 7 7

9 10 11
6. , ,
24 24 24

−5 −4 −3
7. , ,
14 14 14
(Objective DPP # 2.1)

Qus. 1 2 3 4 5 6 7 8

Ans. A B A A C D C C
(Subjective DPP # 2.2)
3. 0.110101001000100001

4. (i) irrational (ii) irrational (iii) rational (iv) irrational

10. 36A 4B 6C 3 6 108

37 6 5 4 4 23
11. (i) 1/3 (ii) (iii) (iv) (v) (vi) (v)
99 11 99 3 3 37

19
12.
30

27
(Objective DPP # 3.1)

Qus. 1 2 3 4 5 6 7 8 9 10

Ans. B D B D C A A B B B

Qus. 11 12 13 14 15

Ans. C D D C D
(Subjective DPP # 3.2)

1. (i) 4 10 > 3 6 > 3 (ii) 4 5 > 3 4 > 3 2. 11 − 6

10 2 3 3
3. 16 + 1 + 5 + 5 4. P = ,Q = 5. (i) 2 (ii) 6 (iii) 22/5
7 49

3 6 + 15 9+4 6
6. (i) 5 (ii) 7. (i) 5 − 2 6 (ii) 7 + 5 2 (iii)
2 3 15

31 10
8. (i) 8 (ii) 5 9. (i) a = 9/2, b = 1/2 (ii) a = ,b =
19 19

12 + 56 10
10. 10 14.
3

15. 1 16. (i) 145 (ii) 283 (iii) 17

y9 / 4
17. 18. (i) 1/4 (ii) 0.1 20. 1
x5 / 6

28
POLYNOMIALS
POLYNOMIALS
An algebraic expression (f(x) of the form f(x) = a 0 + a1x + a2x2 + ........ + anxn, where a0,a 1,a2 ......., an are real

numbers and all the index of ‘x’ are non-negative integers is called a polynomials in x.
(a) Degree of the Polynomial :
Highest Index of x in algebraic expression is called the degree of the polynomial, here a0, a 1x , a2x2 ..... anxn,

are called the terms o the polynomial and z 0. a 1, a2......, an are called various coefficients of the polynomial

f(x).
NOTE : A polynomial in x is said to be in standard form when the terms are written either in increasing
order or decreasing order of the indices of x in various terms.
(b) Different Types of Polynomials :
Generally, we divide the polynomials in the following categories.
(i) Based on degrees :
There are four types of polynomials based on degrees. These are listed below :
(A) Linear Polynomials : A polynomials of degree one is called a linear polynomial. The general
formula of linear polynomial is ax + b, where a and b are any real constant and a ≠ 0.
(B) Quadratic Polynomials : A polynomial of degree two is called a quadratic polynomial. The general
form of a quadratic polynomial is ax2 + b + c, where a ≠ 0.
(C) Cubic Polynomials : A polynomial of degree three is called a cubic polynomial. The general form
of a cubic polynomial is ax3 + bx2 + cx + d, where a ≠ 0.
(D) Biquadratic (or quadric) Polynomials : A polynomial of degree four is called a biquadratic
(quadratic) polynomial. The general form of a biquadratic polynomial is ax4 + bx3 + cx2 + dx + e ,
where a ≠ 0.
NOTE : A polynomial of degree five or more than five does not have any particular name. Such a
polynomial usually called a polynomial of degree five or six or ....etc.
(ii) Based on number of terms
There are three types of polynomials based on number of terms. These are as follows :
(A) Monomial : A polynomial is said to be monomial if it has only one term. e.g. x, 9x2, 5x3 all are
monomials.

(B) Binomial : A polynomial is said to be binomial if it contains two terms e.g. 2x2 + 3x, 3 x + 5x3 , -8x3 +
3, all are binomials.

29
5
(C) Trinomials : A polynomial is said to be a trinomial it if contains three terms. e.g. 3x3 - 8 + ,
2
7x10 8x4 - 3x2, 5 - 7x + 8x9, are all trinomials.
NOTE : A polynomial having four or more than four terms does not have particular Name. These are
simply called polynomials.
(iii) Zero degree polynomial : Any non-zero number (constant) is regarded as polynomial of degree zero or
zero degree polynomial. i.e. f(x) = a, where a ≠ 0 is a zero degree polynomial, since we can write f(x) = a as
f(x) = ax0.
(iv) Zero polynomial : A polynomial whose all coefficients are zeros is called as zero polynomial i.e. f(x) =
0, we cannot determine the degree of zero polynomial.
ALGEBRAIC IDENTITY
An identity is an equality which is true for all values of the variables
Some important identities are
(i) (a + b)2 = a2 + 2ab + b2
(ii) (a - b)2 = a2 - 2ab + b2
(iii) a 2 - b2 = (a + b) (a - b)
(iv) a 3 + b3 = (a + b) (a2 - ab + b2)
(v) a 3 - b3 = (a - b) (a2 + ab + b2)
(vi) (a + b)3 = a3 + b3 + 3ab (a + b)
(vii) (a - b)3 = a 3 - b3 - 3ab (a - b)
(viii) a 4 + a2b2 + b4 = (a2 + ab + b2) (a2 - ab + b2)
(ix) a3 + b3 + c3 - 3abc = (a + b + c) (a2 + b2 + c2 - ab - bc - ac)
Special case : if a + b + c = 0 then a3 + b3 + c3 = 3abc.
(a) Value Form :
(i) a2 + b2 = (a + b)2 - 2ab, if a + b and ab are given
(ii) a 2 + b2 = (a - b)2 + 2ab if a - b and ab are given

(iii) a + b = (a − b )2 + 4ab if a - b and ab are given

(iv) a -b = (a + b)2 − 4ab if a + b and ab are given


2
1  1 1
(v) a 2 + = a +  −2 if a + is given
a2  a a
2
1  1 1
(vi) a 2 + = a +  + 2 if a - is given
a2  a a

(vii) a 3 + b3 = (a + b)3 - 3ab(a + b) if (a + b) and ab are given


3 3 3
(viii) a - b = (a - b) + 3ab(a - b) if (a - b) and ab are given

30
3
1  1  1 1
(ix) x 3 + =  a +  − 3 a +  if a + is given
a3  a  a a
3
1  1  1  1
(x) a 3 − =  a −  + 3 a − 1  , if  a −  is given
a3  a  a  a

(xi) a4 + b4 = (a2 + b2)2 - 2a 2b2 = [(a + b)2 - 2ab]2 - 2a2b2, if (a + b) and ab are given
(xii) a4 - b4 = (a2 + b2) (a2 - b2) = [(a + b)2 -2ab](a + b) (a - b)
(xiii) a5 + b5 = (a3 + b3) (a2 + b2) - a2b2 (a + b)
ILLUSTRATION
Ex.1 Find the value of :
(i) 36x2 + 49y2 + 84xy, when x = 3, y = 6
(ii) 25x2 + 16y2 - 40xy, when x = 6, y = 7
Sol. (i) 36x2 + 49y2 + 84xy = (6x)2 + (7y)2 + 2 × (6x) × (7y)
= (6x + 7y)2
= (6 × 3 + 7 × 6)2[When x = 3, y = 6]
= (18 + 42)2
= (60)2
= 3600. Ans.
2 2 2 2
(ii) 25x + 16y - 40xy = (5x) + (4y) - 2 × (5x) × (4y)
= (5x - 4y)2
= (5 × 6 - 4 × 7)2 [When x = 6, y = 7]
= (30 - 28)2
= 22
=4
Ans.
1  1
Ex.2 If x2 + = 23, find the value of  x +  .
x2  x 
1
Sol. x2 + = 23 ....(i)
x2
1
⇒ x2 + + 2 = 25 [Adding 2 on both sides of (i)]
x2
2
1 1
⇒ (x2) +   + 2 . x . = 25
x x
2
 1
⇒  x +  = (5)2
 x
1
⇒ x+ = 5 Ans.
x

31
1
Ex.3 Prove that a2 + b2 + c2 - ab - bc - ca =
2
[ ]
(a − b )2 + (b − c )2 + (c − a )2 .
Sol. Here, L.H.S. = a2 + b2 + c2 - ab -+ bc - ca
1
= [2a2 + 2b2 + 2c2 - 2ab - 2bc - 2ca]
2
1 2
= [(a - 2ab + b2) + (b2 - 2bc + c2) + (c2 - 2ca + a2)]
2
1
= [(a - b)2 + (b - c)2 + (c - a)2]
2
= RHS Hence Proved.
Ex.4 Evaluate :
(i) (107)2 (ii) (94)2 (iii) (0.99)2
Sol. (i) (107)2 = (100 + 7)2
= (100)2 + (7)2 + 2 × 100 × 7
= 10000 + 49 + 1400
= 11449 Ans.
2 2
(ii) (94) = (100 - 6)
= (100)2 + (6)2 - 2 × 100 × 6
= 10000 + 36 - 1200
= 8836 Ans.
2 2
(iii) (0.99) = (1 - 0.01)
= (1)2 + (0.01)2 - 2 × 1 × 0.01
= + 0.0001 - 0.02
= 0.9801 Ans.
NOTE : We may extend the formula for squaring a binomial to the squaring of a trinomial as given below.
(a + b + c)2 = [a + (b + c)]2
= a2 + (b + c)2 + 2 × a × (b + c) [Using the identity for the square of binomial]
2 2 2
= a + b + c + 2bc + 2 (b + c) [Using (b + c)2 = b2 + c2 + 2bc]
= a2 + b2 + c2 + 2bc + 2ab + 2ac [Using the distributive law]
2 2 2
= a + b + c + 2ab + 2bc + 2ac
∴ (a + b + c) = a2 + b2 + c2 + 2ab + 2bc + 2ac
2

Ex.5 Simplify : (3x + 4)3 - (3x - 4)3.


Sol. We have,
(3x + 4)3 - (3x - 4)3 = [(3x)3 + (4)3 + 3 × 3x × 4 × (3x + 4)] - [(3x)3 - (4)3 - 3 × 3x × 4 × (3x - 4)]
= [273 + 64 + 36x (3x + 4)] - [273 - 64 - 36x (3x - 4)]
= [27x3 + 64 + 108x2 + 144x] - [27x3 - 64 - 108x2 + 144x]
= 27x3 + 64 + 108x2 + 144x - 27x3 + 64 + 108x2 - 144x
= 128 + 216x2
∵ (3x + 4)3 - (3x - 4)3 = 128 + 216x2 Ans.

32
Ex.6 Evaluate :
(i) (1005)3 (ii) (997)3
Sol. (i) (1005)3 = (1000 + 5)3
= (1000)3 + (5)3 + 3 × 1000 × 5 × (1000 + 5)
= 1000000000 + 125 + 15000 + (1000 + 5)
= 1000000000 + 125 + 15000000 + 75000
= 1015075125. Ans.
(ii) (997)3 = (1000 - 3)3
= (1000)3 - (3)3 - 3 × 1000 × 3 × (1000 - 3)
= 1000000000 - 27 - 9000 × (1000 - 3)
= 1000000000 - 27 - 900000 + 27000
= 991026973 Ans.
1 1
Ex.7 If x - = 5, find the value of x3 - 3
x x
1
Sol. We have, x − =5 ...(i)
x
 13 
⇒ x − = ( 5)3  [Cubing both sides of (i)]
 x
 
1 1 1
⇒ x3 − − 3x. . x −  = 125
x3 x x
1  1
⇒ x3 − − 3 x −  = 125
x3  x 
1  1
⇒ x3 − − 3 × 5 = 125 [Substituting  x −  = 5]
x3  x
1
⇒ x3 − − 15 = 125
x3
1
⇒ x3 − = ( 125 + 15) = 140 Ans.
x3
Ex.8 Find the following products of the following expression :
(i) (4x + 3y) (16x2 - 12xy + 9y2) (ii) (5x - 2y) (25x2 + 10xy + 4y2)
2 2
Sol. (i) (4x + 3y) (16x - 12xy + 9y )
= (4x + 3y) [(4x)2 - (4x) × (3y) + (3y)2]
= (x + b) (x2 - ab + b2) [Where a = 4x, b = 3y]
= a3 + b3
= (4x)3 + (3y)3 = 64x3 + 27y3 Ans.
2 2
(ii) (5x - 2y) (25x + 10xy + 4y )
= (5x - 2y) [(5x2 + (5x) × (2y) + (2y)2]
= (a - b) (a2 + ab + b2) [Where a = 5x, b = 2y]
3 3
=a -b
= (5x)3 - (2y)3
= 125x3 - 8y3 Ans.

33
Ex.9 Simplify :
(a 2
) (3
− b2 + v2 − c2 + c2 − a2 ) (
3
.
)
3

(a − b )3 + (b − c )3 + (c − a )3
Sol. (
Here a 2 − b 2 + b 2 − c 2 ) ( ) + (c
3 2
)
− a2 = 0

∴ (a 2
− b2 ) + (b
3 2
− c2 ) + (c
3 2
− a2 )3
( )( )(
= 3 a2 − b2 b2 − c2 c2 − a2 )
Also, (a − b) + (b − c ) + (c − a ) = 0
∴ (a − b)3 + (b − c )3 + (c − a )3 = 3(a − b)(b − c )(c − a )
3(a − b )(a + b )(b − c )(b + c )(c − a )(c + a )
∴ Given expression =
3(a − b )(b − c )(c − a )
3(a − b )(a + b )(b − c )(b + c )(c − a )(c + a )
=
3(a − b )(b − c )(c − a )

= (a + b )(b + c)(c + a ) Ans.

Ex.10 Prove that : (x - y)3 + (y - z)3 + (z - x)3 = 3(x - y) (y - z) (z - x).


Sol. Let (x - y) = a, (y - z) = b and (z - x) = c.
Then, a + b + c = (x - y) + (y - z) + (z - x) = 0
∴ a + b + c3 = 3abc
3 3

Or (x - y)3 + (y - z)3 + (z - x)3 = 3(x - y) (y - z) (z - x) Ans.


Ex.11 Find the value of (28)3 - (78)3 + (50)3.
Sol. Let a = 28, b = - 78, c = 50
Then, a + b + c = 28 - 78 + 50 = 0
∴ a + b3 + c3 = 3abc.
3

So, (28)3 + (-78)3 + (50)3 = 3 × 28 × (-78) × 50


Ex.12 If a + b + c = 9 and ab + bc + ac = 26, find the value of a3 + b3 + c3 - 3abc.
Sol. We have a + b + c = 9 ...(i)
⇒ (a + b + c)2 = 81 [On squaring both sides of (i)]
⇒ a + b + c + 2(ab + bc + ac) = 81
2 2 2

⇒ a 2 + b2 + c2 + 2 × 26 = 81 [ ∵ ab + bc + ac = 26]
⇒ a 2 + b2 + c2 = (81 - 52)
⇒ a 2 + b2 + 2 = 29.
Now, we have
a 3 + b3 + c3 - 3abc = (a + b + c) (a 2 + b2 + c2 - ab - bc - ac)
= (a + b + c) [(a2 + b2 + c2) - (ab + bc + ac)]
= 9 × [(29 - 26)]
= (9 × 3)
= 27 Ans.

34
(b) A Special Product :
We have (x + a) (x + b) = x (x + b) + a (x + b)
= x2 + xb + ax + ab
= x2 + bx + ax + ab [ ∵ xb = bx]
= x2 + ax + bx + ab
= x2 + (a + b) x + ab
Thus, we have the following identity
(x + a) (x + b) = x2 + (a + b)x + ab.

Ex.13 Find the following products :


(i) (x + 2) (x + 3) (ii) (x + 7) (x - 2)
(ii) (y - 4) (y + 3) (iv) (y - 7) (y + 3)
(v) (2x - 3) (2x - 5) (vi) (3x + 4) (3x - 5)

Sol. Using the identity : (x + a) (x + b) = x2 + (a + b) x + ab, we have


(i) (x + 2) (x + 3) = x2 + (2 + 3) x + 2 × 3
= x2 + 5x + 6. Ans.
(ii) (x + 7) (x - 2) = (x + 7) (x + (-2))
= x2 + 7x + (-2)x + 7 × (-2)
= x2 + 5x - 14. Ans.
(iii) (y - 4) (y- 3) = {y + (-4)} {y+ (-3)}
= y2 + {(-4) + (-3)}y + (-4) × (-3)
= y2 - 7y + 12 Ans.
(iv) (y - 7) (y + 3) = {y + (-7)} (y + 3)
= y2 + {(-7) + 3} + (-7) × 3
= y2 - 4y - 21. Ans.
(v) (2x - 3) (2x + 5) = (y - 3) (y + 5), where y = 2x
= {y + (-3)} (y + 5)
= y2 + {(-3) + 5} y + (-3) × 5
= y2 + 2y - 15
= (2x)2 + 2 × 2x - 15
= 4x2 + 4x - 15. Ans.
(vi) (3x + 4) (3x - 5) = (y + 4) (y - 5), where y = 3x
= (y+ 4) {y + (-5)}
= y2 + {4 + (-5)} + 4 × (-5)
= y2 - y - 20
= (3x)2 - 3x - 20
= 9x2 - 3x - 20. Ans.
Ex.14 Evaluate : (i) 35 × 37 (ii) 103 × 96
Sol. (i) 35 × 37 = (40 - 5) (40 - 3)
= (40 + (-5)) (40 + (-3))
= 402 + (-5 - 3) × 40 + (-5 × - 3)
= 1600 - 320 + 15
= 1615 - 320
= 1295 Ans.

35
(ii) 103 × 96
= (100 + 3) [100 + (-4)]
= 1002 + (3 + (-4)) × 100 + (3 × - 4)
= 10000 - 100 - 12
= 9888 Ans.
FACTORS OF A POLYNOMIAL
If a polynomial f(x) can be written as a product of two or more other polynomial f1(x), f2(x), f3(x),..... then

each of the polynomials f1(x), f2(x),..... is called a factor of polynomial f(x). The method of finding the factors

of a polynomials is called factorisations.

(a) Factorisation by Making a Trinomial a Perfect Square :


Ex.15 81a2b2c2 + 64a6b2 - 144a 4b2c
Sol. 81a2b2bc2 + 64a6b2 - 144a4b2c
= [9abc]2 -2 [9abc][8a3b] + [8a3b]2
= [9abc - 8a 3b]2 = a 2b2[9c - 8a2]2 Ans.

2
 1  1  1  1 
Ex.16  3a −  −  3a −  + 9 +  c + − 2a  3a − − 3 
 b  b  b  b 
2
 1  1  1  1 
Sol.  3a −  − 6 3a −  + 9 +  c + − 2a  3a − − 3 
 b  b  b  b 
2
 1  1  1  1 
=  3a −  − 2.3 3a −  + ( 3)2 +  c + − 2a  3a − − 3 
 b  b  b  b 
2
 1   1  1 
=  3a − − 3  +  c + − 2 a  3a − − 3 
 b   b  b 

 1  1 1 
=  3a − − 3  3a − + 3 + − 2 a
 b  b b 

 1 
=  3a − − 3 [a + c − 3] Ans.
 b 

(b) Factorisation by Using the Formula for the Difference of Two Squares :
a2 - b2 = ( + b) (a - b)
Ex.17 Factorise : 4(2a + 3b - 4c)2 - (a - 4b + 5c).2
Sol. = 4(2a + 3b - 4c)2 - (a - 4b + 5c)2
= [2(2a + 3b - 4c)]2 - (a - 4b + 5c)2
= [4a + 6b - 8c + a - 4b + 5c] [4a + 6b - 8c - a + 4b - 5c]
= [5a + 2b - 3c] [3a + 10b - 13c] Ans.

36
1
Ex.18 Factorise : 4x 2 + + 2 − 9y2 .
4x2
1
Sol. 4x2 + + 2 − 9y2
4x2
2
 1   1 
= ( 2 x)2 + 2.( 2 x ).  +   − ( 3 y )2
 2x   2x 
2
 1  2
=  2x +  − ( 3y )
 2x 

 1  1 
=  2x + + 3 y  2 x + − 3y  Ans.
 2x  2x 
1
Ex.19 Factorise : x 4 + − 3.
a4
2
 1   1 
Sol. ( a 2 )2 +  2  − 2.( a 2 ) 2  − 1
a  a 
2
 1 
=  a 2 − 2  − ( 1) 2
 a 

 1  1 
=  a 2 − 2 + 1  a 2 − 2 − 1  Ans.
 a  a 

Ex.20 Factorise : x4 + x2y2 + y4.


Sol. x2 + x2y2 + y4 = (x2)2 + 2.x2.y2 + (y2)2 - x2y2
= x2 + y2)2 - (xy)2
= (x2 + y2 + xy( (x2 + y2 - xy) Ans.

(c) Factorisation by Using Formula of a3 + b3 and a3 - b3 :

Ex.21 Factorise : 64a13b + 343ab13.


Sol. 64a13b + 343ab13 = ab[64a12 + 343b12]
= ab[(4a4)3 + (7b4)3]
= ab[4a4 + 7b4] [(4a 4)2 - (4a 4) (7b4) + (7b4)2]
= ab[4a4 + 7b4][16a8 - 28a 4b4 + 49b8] Ans.
x
Ex.22 Factorise : p 3q2x4 + 3p2qx3 + 3px2 + - q2r3x
q

Sol. In above question, If we take common then it may become in the form of 3 + b3.
x
∴ p 3q2x4 + 3p2qx3 + 3px2 + - q2r3x
q

37
x 3 3 3
= [p q x + 3p2q2x2 + 3pqx + 1 - q3r3]
q

x
= [(pqx)3 + 3(pqx)2 .1 + 3pqx . (1)2 + (1)3 - q3r3]
q

Let pqx = A & 1 = B


x
= [A3 + 3A2B + 3AB2 + B3 - q3r3]
q

x x
= [(pqx + 1)3 - (qr)3] = [pqx + 1 - qr][(pqx + 1)2 + (pqx + 1) qr + (qr)2]
q q

x
= [pqx + 1 - qr][p2q2x2 + 1 + 2pqx + pq2xr + qr + q2r2] Ans.
q

Ex.23 Factories : x3 - 6x2 + 32


Sol : x3 + 32 - 6x2
= x3 + 8 + 24 - 6x2
= [(x)3 + (2)3] + 6[4 - x2]
= (x + 2)[x2 - 2x + 4] + 6[2 + x][2 - x]
= (x + 2) [x2 -2x + 4 + 6(2 - x)]
= (x + 2)[x2 -2x + 4 + 12 - 6x]
= (x + 2) [x2 - 8x + 16]
= (x + 2) (x - 4)2 Ans.

38
EXERCISE

OBJECTIVE DPP - 4.1

1. The product of (x + a) (x + b) is :
(A) x2 + (a + b) x + ab (B) x2 - (a - b) x + ab (C) a2 + (a - b)x + ab (D) x2 + (a - b)x - ab.

2. The value of 150 × 98 is :


(A) 10047 (B) 14800 (C) 14700 (D) 10470

3 The expansion of (x + y - z)2 is :


(A) x2 + y2 + z2 + 2xy + 2yz + 2zx (B) x2 + y2 - z2 - 2xy + yz + 2zx
(C) x2 + y2 + z2 + 2xy - 2yz - 2zx (D) x2 + y2 - z2 + 2zy - 2yz - 2zx

4. The value of (x + 2y + 2z)2 + (x - 2y - 2z)2 is:


(A) 2x2 + 8y2 + 8z2 (B) 2x2 + 8y2 + 8z2 + 8xyz
(C) 2x2 + 8y2 + 8z 2 - 8yz (D) 2x2 + 8y2 + 8z 2 + 16yz
5. The value of 25x2 + 16y2 + 40 xy at x = 1 and y = -1 is :
(A) 81 (B) -49 (C) 1 (D) None of these
6. On simplifying (a + b)3 + (a - b)3 + 6a(a2 - b2) we get :
(A) 8a2 (B) 8a2b (C) 8a 3b (D) 8a 3

a 3 + b 3 + c 3 − 3abc
7. Find the value of , when a = -5, 5 = -6 , c = 10.
ab + bc + ca − a 2 − b 2 − c 2
(A) 1 (B) -1 (C) 2 (D) -2
8. If (x + y + z) = 1, xy + yz + zx = -1 xyz = -1 then value of x3 + y3 + z 3 is :
(A) -1 (B) 1 (C) 2 (D) -2
9. In method of factorisation of an algebraic expression. Which of the following statement is false ?
(A) Taking out a common factor from two or more terms.
(B) Taking out a common factor from a group of terms.
(C) By using remainder theorem.
(D) By using standard identities.
10. Factors of (a + b)3 - (a - b)3 is :
(A) 2ab(3a 2 + b2) (B) ab(3a 2 + b2) (C) 2b(3a2 + b2) (D) 3a 2 + b2
11. Degree of zero polynomial is :
(A) 0 (B) 1 (C) Both 0 & 1 (D) Not defined

39
SUBJECTIVE DPP 4.2

1 1
1. If a 4 + = 119 , then find the value of − 3 .
a4 a

2. If x = 152, y = -91 find the value of 9x2 + 30xy + 25y2.

3. Evaluate :

2
 1
(i) (5x + 4y)2 (ii) (4x - 5y)2 (iii)  2 x − 
 x

4. If x + y = 3 and xy = - 18, find the value of x3 + y3.

1 1
5. If x 2 + 2
= 51 find the value of x 3 − 3 . .
x x

6. Evaluate :

3 3 3
 1 1 5
(i) 253 - 753 + 503 (ii)   +   −   (iii) (0.2)3 - (0.3)3 + (0.1)3
2 3 6

7. Find the product of :

1 1
(i) (x + 4) (x + 7) (ii) ( x + )( x + 5) (iii) (P2 + 16) (P 2 − )
5 4

8. Evaluate :

(i) 102 × 106 (ii) 994 × 1006 (iii) 34 × 36

9. Factorise : 4x4 + (7a)4.

10. Factorise : x12 = 1.

11. Evaluate
( a − b )2 (b − c )2 (c − a )2
.
+ +
(b − c )(c − a ) (a − b )(c − a ) (a − b )(b − c )
12. Write the following polynomials in standard forms :

(i) x6 - 3a 4 + 2 x + 5x2 + 7x5 + 4

(ii) m7 + 8m5 + 4m6 + 6m - 3m2 - 11

13. Factorise : (x + 1) (x + 2) (x + 3) (x + 4) - 3.

14. Factorise : 64a3 - 27b3 - 144a2b + 108ab2.

15. Factorise : x4 + 2x3y - 2xy3 - y4.

40
POLYNOMIALS

ML – 5
ZEROS OR ROOTS OF A POLYNOMIAL

A real number α is a root or zero of polynomial f(x) = a n x n + a n − 1x n − 1 + a n − 2 x n − 2 + ...... + a1x + a0 ,

if f (α ) = 0. i.e. a n α n + a n − 1 + a n − 2 α n − 2 + ...... + a1α + a0 = 0.

For example x = 3 is root of the polynomial f(x) = x3 - 6x2 + 11x - 6, because


f(3) = (3)3 - 6(3)2 + 11(3) - = 27 - 54 + 33 - 6 = 0.
but x = - is not a root of the above polynomial,
∵ f(-2) = (-2).3 - 6(-2)2 + 11(-2) - 6
f(-2) = -8 -24 - 22 - 6
f(-2) = - 60 ≠ 0.
(a) Value of a Polynomial :
The value of a polynomial f(x) at x= α is obtained by substituting x = α in the given polynomial and is
denoted by f (α ) . e.g. If f(x) = 2x3 - 13x2 + 17x + 12 then its value at x = 1 is.

f(1) = 2(1)3 - 13(1)2 + 17(1) + 12


= 2 - 13 + 17 + 12 = 18.
Ex.1 Show that x = 2 is a root of 2x3 + x2 - 7x - 6.
Sol. p(x) = 2x3 + x2 - 7x - 6 then,
p(2) = 2(2)3 + (2)2 7(2) - 6 = 16 + 4 - 14 - 6 = 0
Hence x = 2 is a root of p(x). Ans.
4
Ex. 2 If x = is a root of the polynomial f(x) = 6x3 - 11x2 + kx - 20 then find the value of k.
3
Sol. f(x) = 6x3 - 11x2 + kx - 20
3 2
4 4 4 4
⇒ f   = 6  − 11  + k   − 20 = 0
3 3 3 3
64 16 4k
⇒ 6. − 11. + − 20 = 0
9.3 9 3
⇒ 128 - 176 + 12k - 180 = 0
⇒ 12k + 128 - 356 = 0
⇒ 12k = 228
⇒ k = 19 Ans.
Ex.3 If x = 2 & x = 0 are two roots of the polynomial f(x) = 2x3 - 5x2 + ax + b. Find the values of and b.

41
Sol. f(x) = 2(2)3 - 5(2)2 + a(2) + b = 0
⇒ 16 - 20 + 2a + b = 0
⇒ 2a + b = 4 ....(i)
3 2
⇒ f(0) = 2(0) - 5(0) + a(0) + b = 0
⇒ b=0
So, 2a = 4
Hence, a = 2, b = 0 Ans.
REMAINDER THEOREM
Let ‘p(x)’ be any polynomial of degree greater than or equal to one and a be any real number and If p(x) is
divided by (x - a), then the remainder is equal to p(a).
Let q(x) be the quotient and r(x) be the remainder when p(x) is divided by (x - a) then
Dividend = Divisor × Quotient + Remainder
p(x) = (x - a) × q(x) + [r(x) or r], where r(x) = 0 or degree of r(x) < degree of (x - ). But (x - a) is a polynomial
of degree 1 and a polynomial of degree less than 1 is a constant. Therefore, either r(x) = 0 or r(x) = Constant.
Let r(x) = r, then p(x) = (x - a)q(x) + r,
putting x = a in above equation p(a) = (a - a)q(a) + r = 0. q(a) + r
p(a) = 0 + r
⇒ p(a) = r
This shows that the remainder is p(a) when p(x) is divided by (x - a).
REMARK : If a polynomial p(x) is divided by (x + a), (ax - b), (x + b), (b - ax) then the remainder in the
b b b b  b b
value of p(x) at x = − a , ,− , i.e. p(-a), p  , p −  , p  respectively.
a a a a  a a
Ex.4 Find the remainder when f(x) = x3 - 6x2 + 2x - 4 is divided by g(x) = 1- 2x.
1
Sol. 1 - 2x = 0 ⇒ 2x = 1 ⇒ x =
2
3 2
1 1 1 1
f   =   − 6  + 2   − 4
2 2 2 2
1 3
= − +1− 4
8 2
1 − 12 + 8 − 32 35
= =− Ans.
8 8
Ex.5 The polynomials ax3 + 3x2 - 13 and 2x3 - 5x + a are divided by x + 2 if the remainder in each case is the same,
find the value of a.
Sol. p(x) = ax3 + 3x2 - 13 and q(x) = 2x3 - 5x + a
when p(x) & q(x) are divided by x + 2 = 0 ⇒ x = -2
p (-2) = q(-2)
⇒ a(-2)3 + 3(-2)2 - 13 = 2 (-2)3 - 5(-2) + a
⇒ -8a + 12 - 13 = - 16 + 10 + a
⇒ -9a = - 5
5
⇒ a= Ans.
9
(a) Factor Theorem :

42
Let p(x) be a polynomial of degree greater than or equal to 1 and ‘a’ be a real number such that p(a) = 0,
than (x - a) is a factor of p(x). Conversely, if (x - a) is a factor of p(x), then p(a) = 0.
Ex.6 Show that x + 1 an d 2x - 3 are factors of 2x3 - 9x2 + x + 12.
3
Sol. To prove that (x + 1) and (2x - 3) are factors of 2x3 - 9x2 + x + 12 it is sufficient to show that p(-1) and p 
2
both are equal to zero.
p(-1) = 2(-1)3 - 9(-1)2 + (-1) + 12 = - 2 - 9 - 1 + 12 = - 12 + 12 = 0
3
3 3 3 3
And, p  = 2  − 9  +   + 12
2 2 2 2
27 81 3 27 − 81 + 6 + 48 −81 + 81
= − + + 12 = = =0
4 4 2 4 4
Hence, (x + 1) and (2x - 3) are the factors 2x3 - 9x2 + x + 12. Ans.
a
Ex.7 Find and β if x + 1 and x + 2 are factors of p(x) = x3 + 3x2 - 2 αx + β .

Sol. When we put x + 1 = 0 or x = -1 and x + 2 = 0 or x = - 2 in p(x)


Then, p(-1) = 0 & p(-2) = 0
Therefore, p(-1) = (-1)3 + 3(-1)2 - 2 α (-1) + β = 0
⇒ - 1 + 3 + 2α + β = 0 ⇒ β = - 2α - 2 ....(i)
And, p(-2) = (-2)3 + 3(-2)2 - 2α (-2) + β = 0
⇒ - 8 + 12 + 4α + β = 0 ⇒ β = - 4α - 4 .....(i)
From equation (i) and (ii)
- 2 α - 2 = - 4α - 4
⇒ 2α = - 2 ⇒ α = - 1
Put α = -1 in equation (i) ⇒ β = -2(-1) - 2 = 2 - 2 = 0.
Hence, α = - 1 β = 0. Ans.
Ex.8 What must be added to 3x + x - 22x + 9 so that the result is exactly divisible by 3x2 + 7x - 6.
3 2

Sol. Let p(x) = 3x3 + x2 - 22x + 9 and q(x) = 3x2 + 7x - 6.


We know if p(x) is divided by q(x) which is quadratic polynomial therefore if p(x) is not exactly divisible by
q(x) then the remainder be r(x) and degree of r(x) is less than q(x) (or Divisor)
∴ By long division method
Let we added ax + b (linear polynomial) is p(x), so that p(x) + ax + b is exactly divisible by 3x2 + 7x - 6.
Hence p(X) + ax + b = s(x) = 3x3 + x2 - 22x + 9 ax + b
= 3x3 + x2 - x(22 - a) + (9 + b)
x−2
2 3 2
3x + 7x - 6 3x + x − x( 22 − a) + 9 + b
− 3x 3 + 7 x 2 − 6x
2
− 6x + 6x − ( 22 − a)x + 9 + b
or
− 6x 2 + x( −16 + a ) + 9 + b
− 6x 2 − 14 x ±12
x( −2 + a) + ( b − 3

Hence, x(a – 2 + b – 3 = 0 . x + 0)
⇒ a-2=0&b-3=0

43
⇒ a = 2 or b = 3 Ans.
Hence, if we add ax + b or 2x + 3 in p(x) then it is exactly divisible by 3x2 + 7x - 6.
Ex.9 Using factor theorem, factories :
p(x) = 2x4 - 7x3 - 13x2 + 63x - 45
Sol. 45 ⇒ ±1,±3 ,±5,±9 ,±15,±45
if we put x = 1 in p(x)
p(1) = 2(1)4 - 7(1)3 - 13(1)2 + 63(1) - 45
2 - 7 - 13 + 63 - 45 = 65 - 65 = 0
∴ x = 1 or x - 1 is a factor of p(x).
Similarly, if we put x = 3 in p(x)
p(3) = 2(3)4 - 7(3)3 - 13(3)2 + 63(3) - 45
162 - 189 - 117 + 189 - 45 = 162 - 162 = 0
Hence, x = 3 or x - 3 = 0 is the factor of p(x).
p(x) = 2x4 - 7x3 - 13x2 + 63x - 45
∴ p(x) = 2x3(x - 1) -5x2(x - 1) - 18(x - 1) + 45(x - 1)
2x4 - 2x3(x - 1) - 5x2 - 18x2 + 18x + 45x - 54
⇒ p(x) = (x - 1)(2x3 - 5x2 - 18x + 45)
⇒ p(x) = (x - 1)(2x3 - 5x2 - 18x + 45)
⇒ p(x) = (x - 1)[2x2(x - 3) + x(x - 3) - 15(x - 3)]
⇒ p(x) = (x - 1)[2x3 - 6x2 + x2 - 3x - 15x + 45]
⇒ p(x) = (x - 1)(x - 3)(2x2 + x - 15)
⇒ p(x) = (x - 1)(x - 3)(2x2 + 6x - 5x - 15)
⇒ p(x) = (x - 1)(x - 3)[2x(x + 3) - 5(x + 3)]
⇒ p(x) = (x - 1)(x - 3)(x + 3)(2x - 5)

FACTORISATION OF A QUADRATIC POLYNOMIAL


For factorisation of a quadratic expression ax2 + bx + a where a ≠ 0, there are two method.
(a) By Method of Completion of Square :
In the form ax2 + bx + c where a ≠ 0, firstly we take ‘a’ common in the whole expression then factorise by

 b c
converting the expression a x 2 + x +  as the difference of two squares.
 a a
Ex.10 Factorise x2 - 31x + 220.
Sol. x2 - 31a + 220
2 2
31  31   31 
= x 2 − 2. .x +   −   + 220
2  2   2 
2 2
 31  961  31  81
= x −  − + 220 =  x −  −
 2  4  2  4
2
 31   9   31 9  31 9 
= x −  −   = x − +  x − − 
 2 2
    2 2  2 2

44
= (x - 11)(x - 20) Ans.

Ex.11 Factorise : - 10x2 + 31x - 24

Sol. -10x2 + 31x - 24

 31 24 
= -[10x2 - 31x + 24] = - 10 x 2 − ×+ 
 10 10 

 2 2
31  31   31  24 
= −10 x 2 − 2. .x +   −   + 
 20  20   20  10 

 2 2
31  961 24   31  1 
= −10  x −  − +  = −10  x −  − 
 20  400 10   20  400 

 2 2
31   1    31 1   31 1 
= −10  x −  −    = −10 x − +  x − −
 20   20    20 20   20 20 

 2 x − 3  5x − 8 
= −10   = −( 2 x − 3)( 5x − 8) = ( 3 − 2 x )( 5x − 8) Ans.
 2  5 

(b) By Splitting the Middle Term :

In the quadratic expression ax2 + bx + c, where a is the coefficient of x2, b is the coefficient of x and c is the

constant term. In the quadratic expression of the form x2 + bx + c, a = 1 is the multiple of x2 and another

terms are the same as above.

There are four types of quadratic expression :

(i) ax2 + bx + c (ii) ax2 - bx + c

(iii) ax2 - bx - c (iv) ax2 + bx - c

Ex.12 Factorise : 2x2 + 12 2 x + 35.

Sol. 2x2 + 12 2 x + 35

Product ac = 70 & b = 12 2

∴ Split the middle term as 7 2 & 5 2

⇒ 2x2 + 12 2 x + 35 = 2x2 + 7 2 x + 5 2 x + 35

= [ ] [
2x 2x + 7 + 5 2x + 7 ]
= [ 2 x + 5][ 2 x + 7 ] Ans.

45
Ex.13 Factorise : x2 - 14x + 24.
Sol. Product ac = 24 & b = -14
∴ Split the middle term as - 12 & - 2
⇒ x 2 - 14x + 24 = x2 - 12 - 2x + 24
⇒ x(x - 12) - 2 (x - 12) = (x - 12)(x - 2) Ans.
13 1
Ex.14 Factorise : x2 − x− .
24 12
13 1 1
Sol. x2 − x− = [ 24x 2 − 13x − 2 ]
24 12 24
Product ac = - 48 & b = - 13 ∴ We split the middle term as - 16x + 3x.
1
= [24x2 - 16x + 3x - 2]
24
1
= [8x(3x - 2) + 1(3x - 2)]
24
1
= (3x - 2)(8x + 1) Ans.
24
3 2 35
Ex.15 Factorise : x − 8x −
2 2
3 2 35 1 1
Sol. x − 8x − = ( 3x 2 − 16x − 35) = ( 3x 2 − 21x + 5x − 35)
2 2 2 2
1 1
= [ 3x( x − 7 ) + 5x( x − 7 )] = ( x − 7 )( 3x + 5) Ans.
2 2
(c) Integral Root Theorem :
If f(x) is a polynomial with integral coefficient and the leading coefficient is 1, then any integer root of f(x) is
a factor of the constant term. Thus if f(x) = x3 - 6x2 + 11x - 6 has an Integral root, then it is one of the factors
of 6 which are ±1, ± 2 , ± 3 , ± 6. .
Now Infect f(1) = (1)3 - 6(1)2 + 11(1) - 6 = 1 - 6 + 11 - 6 =0
f(2) = (2)3 - 6(2)2 + 11(2) - 6 = 8 - 24 + 22 - 6 = 0
f(3) = (3)3 - 6(3)2 + 11(3) - 6 = 27 - 54 + 33 - 6 = 0
Therefore Integral roots of f(X) are 1,2,3.

(d) Rational Root Theorem :


b b
Let be a rational fraction in lowest terms. If is a root of the polynomial f(x) =
c c
a n x n + a n − 1x n − 1 + .... + a 1x + a0 , a n ≠ 0 with integral coefficients. Then b is a factor of constant term a0 and c
is a factor of the leading coefficient a n.
b
For example : If is a rational root of the polynomial f(x) = 6x3 + 5x2 - 3x - 2, then the values of b are
c
limited to the factors of -2 which are ±1, ± 2 and the value of c are limited to the factors of 6, which are
1 1 1 2
±1, ± 2 , ± 3 , ± 6 Hence, the possible rational roots of f(x) are ± 1, ± 2 , ± , ± , ± , ± . ±1, ± 2 , ± 3, ± 6. Infect -
2 3 6 3
2 1
1 is a Integral root and ,− are the rational roots of f(x) = 6x3 + 5x2 - 3x - 2.
3 2

46
NOTE:
(i) An nth degree polynomial can have at most n real roots.
(ii) Finding a zero or root of polynomial f(x) means solving the polynomial equation f(x) = 0. It follows from
the above discussion that if f(x) = ax + b, a ≠ 0 is a linear polynomial, then it has only one root given by f(x)
= 0 i.e. f(x) = ax + b = 0
⇒ ax = - b
b
⇒ x= −
a
b
Thus a=− is the only root of f(x) = ax + b.
a
Ex.16 If f(x) = 2x3 - 13x2 + 17x + 12 then find out the value of f(-2) & f(3).
Sol. f(x) = 2x3 - 13x2 + 17x + 12
f(-2) = 2(-2)3 - 13(-2)2 + 17 (-2) + 12
= - 16 - 52 - 34 + 12 = - 90 Ans.
f(3) = 2(3)3 - 13(3)2 + 17(3) + 12
= 54 - 117 + 51 + 12 = 0 Ans.
(e) Factorisation of an Expression Reducible to A Quadratic Expression :
Ex.17 Factorise :- 8 + 9(a - b)6 - (a - b)12
Sol. -8 + 9(a - b)6 - (a - b)12
Let (a - b)6 = x
Then -8 + 9x - x2 = - (x2 - 9x + 8) = - (x2 - 8x - x + 8)
= - (x - 8)(x - 1)
= - [(a - b)6 - 8][(a - b)6 - 1]
= [1 - (a - b)6][(a - b)6 - 8]
= [(1)3 - {(a - b)2}3][{(a - b)2}3 - (2)3]
= [1 - (a - b)2][1 + (a - b)4 + (a - b)2][(a - b)2 - 2][(a - b)4 + 4 + 2(a - b)2] Ans.
Ex.18 Factorise : 6x2 - 5xy - 4y2 + x + 17y - 15
Sol. 6x2 + x[1 - 5y] - [4y2 - 17y + 15]
= 6x2 + x[1 - 5y] - [4y2 - 17y + 15]
= 6x2 + x[1 - 5y] - [4y(y - 3) -5(y - 3)]
= 6x2 + x[1 - 5y] - (4y - 5)(y - 3)
= 6x2 + 3(y - 3)x - 2(4y - 5)x - (4y -)(y - 3)
= 3x[2x + y 3] - (4y - 5)(2x + y - 3)
= (2x + y - 3)(3x - 4y + 5) Ans.

47
EXERCISE

OBJECTIVE DPP - 5.1

1. Factors of (42 - x - x2) are:

(A) (x - 7)(x - 6) (B) (x + 7)(x - 6) (C) (x + 7)(6 - x) (D) (x + 7)(x + 6)

 x 1
2. Factors of  x 2 + −  are :
 6 6

1 1 1 1
(A) (2x + 1)(3x + 1) (B) (2x + 1)(3x - 1) (C) (2x - 1)(3x - 1) (D) (2x - 1)(3x + 1)
6 6 6 6

3. Factors of polynomial x3 - 3x2 - 10x + 2x are :

(A) (x - 2)(x + 3)(x - 4) (B) (x + 2)(x + 3)(x + 4)

(C) (x + 2)(x - 3)(x - 4) (D) (x - 2)(x - 3)(x - 4)

4. If (x + a) is a factor of x2 + px + q and x2 + mx + n then the value of a is :

m−p n−q n+q m+p


(A) (B) (C) (D)
n−q m −p m+p n+q

SUBJECTIVE DPP - 5.2

1. Factorise : 8x3 + 16 - 9.

2. Factorise : x4 + x3 - 7x2 - x + 6.

3. Factorise : 9z 3 - 27z2 - 100z + 300.

4. Determine whether x - 3 is a factor of polynomial p(x) = x3 - 3x2 + 4x - 12.

5. Using factor theorem, prove that p(x) is divisible by g(x) if P(x) = 4a 4 + 5x3 - 12x2 - 11x + 5, g(x) = 4x + 5.

6. Determine if (x + 1) is a factor of x3 - x2 - (2 - 2 )x + 2.

7. x3 - 23x2 + 142x -120.

8. x3 + 13x2 + 32 x + 20.

9. 2x3 + y2 - 2y - 1.

10. 4z 3 + 20z2 + 33z + 18.

11. x4 + 5x2 + 4.

12. x3 - 10x2 - 53x - 42.

48
ANSWER KEY

(Objective DPP # 4.1)


Qus. 1 2 3 4 5 6 7 8 9 10 11

Ans. A C C D C D A B C C D

(Subjective DPP # 4.2)


1. 36 2. 1
1
3. (i) 25x2 + 40xy + 16y2 (ii)16x2 - 40xy + 25y2 (iii) 4x2 - 4 +
x2
4. 189 5. 364
5
6. (i) -281250 (ii) − (iii) -0.018
12
26 63 2
7. (i) x2 + 11x + 28 (ii) x 2 + x+1 (iii) P 4 + P −4
5 4
8. (i) 10812 (ii) 999964 (iii) 1224

9. (2x2 + 49a 2 + 14ax)(2x2 + 49a2 - 14ax)


10. (x - 1) (x + 1) (x2 + 1)(x2 + x + 1)(x2 - x + 1)(x4 - x2 + 1) 11. 3

12. (i) x6 + 7x5 - 34 + 5x2 + 2x + 4 (ii) m7 + 4m6 + 8m5 - 3m2 + 6m - 11

13. (x2 + 5x + 3)(x2 + 5x + 7) 14. (4a - 3b)3 15. (x - y) (x + y)3

(Objective DPP # 5.1)

Qus. 1 2 3 4

Ans. C B A B

(Objective DPP # 5.2)

1. (2x - 1)(4x2 + 2x + 9) 2. (x + 1)(x - 1)(x + 3)(x - 2) 3. (3z + 10)(z - 3)(3z - 10)

4. Yes 6. (x + 1) is a factor. 7. (x - 1)(x - 10)(x - 12)

8. (x + 1)(x + 2)(x + 10) 9. (y - 1)(y + 1)(2y + 1) 10. (z + 2)(2z + 3)(2z + 3)

11. (x - 1)(x + 1)(x - 2)(x + 2) 12. (x + 1)(x - 14)(x + 3)

49
COORDINATE GEOMETRY

ML - 6
CO-ORDINATE SYSTEM
In two dimensional coordinate geometry, we u se generally two types of co-ordinate system.
(i) Cartesian or Rectangular co-ordinate system.
(ii) Polar co-ordinate system.
In cartesian co-ordinate system we represent any point by ordered pair (x,y) where x and y are called X and
Y co-ordinate of that point respectively.
In polar co-ordinate system we represent any point by ordered pair (r, θ ) where ‘r’ is called radius vector
and ' θ' is called vectorial angle of that point.

CARTESIAN CO-ORDINATE SYSTEM


(a) Rectangular Co-ordinate Axes :
Let X’OX and Y’OY are two lines such that X’OX is horizontal and Y’OY is vertical lines in the same plane
and they intersect each other at O. This intersecting point is called origin. Now choose a convenient unit of
length and starting from origin as zero, mark off a number scale on the horizontal line X’OX, positive to the
right of origin O and negative to the left of origin O. Also mark off the same scale on the vertical line Y’OY,
positive upwards and negative downwards of the origin. The line X’OX is called X-axis and the line Y’OY is
known as Y-axis and the two lines taken together are called the co-ordinate axis.

50
(b) Quadrants :

The co-ordinates axes X’OX and Y’OY divide the place of graph paper into four parts XOY, X’OY, X’OY and
XOY’. These four parts are called the quadrants. The part XOY, X’OY, X’OY’ and XOY’ are known as the
first, second, third and fourth quadrant respectively.
(c) Cartesian Co-ordinates of a Point :
Let X’OX and Y’OY be the co-ordinate axis and P be any point in the plane. To find the position of P with
respect of X’OX and Y;OY, we draw two perpendiculars from P on both co-ordinate axes. Let PM and PN
be the perpendiculars on X-axis and Y-axis reservedly. The length of the line segment OM is called the x-
coordinate be the or abscissa of point P. Similarly the length of line segment ON is called they-coordinate or
ordinate of point P.
Let OM = x and ON = y. The position of the point P in the plane with respect to the coordinate axis is
represented by the ordered pair (x,y). The ordered pair (x,y) is called the coordinates of point P. “Thus, for a
given point, the abscissa and ordinate are the distance of the given point from Y-axis and X-axis
respectively”.
The above system of coordinating on ordered pair (x,y) with every point in plane is called the Rectangular
Cartesian coordinates system.

(b) Convention of Signs :


As discussed earlier that regions XOY, X’OY, X’OY’ and XOY’ are known as the first, second, third and
fourth quadrants respectively. The ray OX is taken as positive X-axis, OX’ as negative X-axis, OY as positive
Y-axis and OY’ as negative Y-axis. Thus we have,
In first quadrant : X > 0, y>0 (Positive quadrant)
In second quadrant: X < 0, Y>0
In third quadrant : X < 0, Y<0 (Negative quadrant)
In fourth quadrant: X > 0, Y<0
(e) Points on Axis :

51
In point P lies on X-axis then clearly its distance from X-axis will be zero, therefore we can say that its
coordinate will be zero. In general, if any point lies on X-axis then its y-coordinate will be zero. Similarly if
any point Q lies on Y-axis, then its distance from Y-axis will be zero therefore we can say its x-coordinate
will be zero. In general, if any point lies on Y-axis then its x-coordinate will be zero.

(f) Plotting of Points :


In order to plot the points in a plane, we may use the following algorithm m.
Step I: Draw two mutually perpendicular lines on the graph paper, one horizontal and other vertical.
Step II: Mark their intersection point as O (origin).
Step III: Choose a suitable scale on X-axis and Y-axis and mark the points on both the axis.
Step IV: Obtain the coordinates of the point which is to be plotted. Let the point be P(a,b). To plot this point
start from the origin and |a| units move along OX, OX’ according as ‘a’ is positive or negative respectively.
Suppose we arrive at point M. From point M move vertically upward or downward |b| through units
according as ‘b’ is positive or negative. The point where we arrive finally is the required point P(a,b).

ILLUSTRATIONS :

Ex.1 Plot the point (3,4) on a graph paper.


Sol. let X’IX and Y’OY be the coordinate axis. Here given point is P(3,4), first we move 3 units along OX as 3 is
positive then we arrive a point M. Now from M we move vertically upward as 4 is positive. Then we arrive
at P(3,4).

52
Ex.2 Write the quadrants for the following points.
(i) A(3,4) (ii) B(-2,3) (iii) C(-5,-2) (iv) D(4,-3) (v) E(-5,-5)
Sol. (i) Here both coordinates are positive therefore point A lies in Ist quadrant.
(ii) Here x is negative and y is positive therefore point B lies in IInd quadrant.
(iii) Here both coordinates are negative therefore point C lines in IIIrd quadrant.
(iv) Here x is positive and y is negative therefore point D lies in IVth quadrant.
(v) Point E lies in III quadrant.

Ex.3 Plot the following points on the graph paper.


(i) A(2 ,5) (ii) B(-5,-7) (iii) C(3,-2) (iv) D (0,5) (v) E(5,0)
Sol. Let XOX’ and YOY’ be the coordinate axis. Then the given points may be plotted as given below :

DISTANCE BETWEEN TWO POINTS


If there are two points A (x1, y1) and B (x2, y2) on the XY plane, the distance between them is given by AB =

d= ( x 2 − x 1 )2 + ( y 2 − y 1 )2

Ex.4 Find the distance between


(i) (5,3) and (3,2) (ii) (-1,4) and (2,-3) (iii) (a,b) and (-b,a)
Sol. Let d1, d2, d3 be the required distances. By using the formula, we have

(i) d 1 = ( 5 − 3 )2 + ( 3 − 2 )2 = 2 2 + 1 2 = 5

(ii) d 2 = ( −1 − 2 )2 + { 4 − ( −3)} 2 = ( −3)2 + 7 2 = 58

(iii) d 3 = { a + ( −b)} 2 + ( b − a )2 = (a + b)2 + (a − b)2 = 2 a 2 + 2 b2

53
EXERCISE

OBJECTIVE DPP - 6.1


1. The abscissa of a point is distance of the point from :
(A) X-axis (B) Y-axis (C) Origin (D) None of these
2. The y co-ordinate of a point is distance of that point from :
(A) X-axis (B) Y-axis (C) Origin (D) None of these
3. If both co-ordinates of any point are negative then that point will lie in :
(A) First quadrant (B) Second quadrant (C) Thirst quadrant (D) Fourth quadrant
4. If the abscissa of any point is zero then that point will lie :
(A) on X-axis (B) on Y-axis (C) at origin (D) None of these
5. The co-ordinates of one end point of a diameter of a circle are (4, -1) and coordinates of the centre of the
circle are (1 ,-3) then coordinates of the other end of the diameter are :
(A) (2,5) (B) (-2,-5) (C) (3,2) (D) (-3,-2)
6. The point (-2,-1), (1,0), (4,3) and (1,2) are the vertices of a :
(A) Rectangle (B) Parallelogram (C) Square (D) Rhombus
7. The distance of the point (3, 5) from X- axis is :

(A) 34 (B) 3 (C) 5 (D) None of these

SUBJECTIVE DPP - 6.2


1. Plot the points in the plane if its co-ordinates are given as A (5,0), B(0,3) C(7,2), D(-4,3), E(-3,-2) and F(3,-2).
2. In which quadrant do the following points lie A(2,3), B(-2,3), C(-3,-5), D(3, -1). Explain with reasons.
3. Plot the following pairs of numbers as points in the Cartesian plane.
x -3 -2 8 4 0
y 5 0 3 8 -2

4. With rectangular exes, plot the points O(0,0), A(4,0) and C(0,6). Find the coordinates of the fourth points B
such the OABC forms a rectangle.
5. Plot the points P(-3,1) and Q(2,1) in rectangular coordinate system and find all possible coordinates of other
two vertices of a square having P and Q as two adjacent vertices”.
6. Find the value of x, if the distance between the points (x, -1) and (3,2) is 5.
7. The base AB two equilateral triangles ABC and ABC’ with side 2a, lies along the x-axis such that the mid
point of AB is at origin. Find the coordinates of the vertices C and C’ of the triangles.

54
ANSWER KEY

(Objective DPP # 6.1)

Qus. 1 2 3 4 5 6 7

Ans. B A C B B B C

(Subjective DPP # 6.2)

2. A- Ist quadrant B - IInd quadrant C IIIrd quadrant D - IVth quadrant

4. (4,6) 5. (-3,6), (2,6) & (-3,-4), (2,-4) 6. 7 or -1

7. C(0, 3 a), C’(0, − 3 a)

55
LINEAR EQUATION IN
TWO VARIABLES
ML - 7
LINEAR EQUATIONS IN ONE VARIABLE
An equation of the form ax + b = 0 where a and b are real numbers and ‘x’ is a variable, is called a linear
equation in one variable.
Here ‘a’ is called coefficient of x and ‘b’ is called as a constant term. i.e. 3x + 5 = 0, 7x - 2 = 0 etc.
LINEAR EQUATION IN TWO VARIABLES
An equation of the form ax + by + c = 0 where a,b,c are real numbers and a, b ≠ 0, and x, y are variable, is
called a linear equation in two variables, here ‘a’ is called coefficient of x, ‘b’ is called coefficient of y and ‘c’
is called constant term.
Any pair of values of x and y which satisfies the equation ax + by + c = 0, is called a solution of it.
Ex.1 Prove that x = 3, y = 2 is a solution of 3x - 2y = 5.
Sol. x = 3, y = 2 is a solution of 3x - 2y = 5, because L.H.S. = 3x - 2y = 3 × 3 - 2 × 2 = 9 - 4 = 5 = R.H.S.
i.e. x = 3, y = 2 satisfied the equation 3x - 2y = 5.
∴ it is solution of the given equation.
Ex.2 Prove that x = 1, y = 1 as well as x = 2, y = 5 is a solution of 4x - y - 3 = 0.
Sol. Given eq. is 4x - y - 3 = 0 ....(i)
First we put x= 1, y = 1 in L.H.S. of eq...(i)
Here L.H.S. = 4x - y - 3 = 4 × 1 - 1 - 3 = 4 - 4 = 0 = R.H.S.
Now we put x = 2, y = 5 in eq. (i)
L.H.S. = 4x - y - 3 = 4 × 2 - 5 - 3 = 8 - 8 = 0 = R.H.S.
Since, x = 1, y = 1 and x = 2, y = 5 both pair satisfied in given equation therefore they are the solution of
given equation.
Ex.3 Determine whether the x = 2, y = -1 is a solution of equation 3x + 5y - 2 = 0.
Sol. Given eq, is 3x + 5y - 2 = 0 ....(i)
Taking L.H.S. = 3x + 5y - 2 = 3 × 2 + 5 × (-1) - 2 = 6 - 5 - 2 = 1 ≠ 0
Here L.H.S. ≠ R.H.S. therefore x = 2, y = - is not a solution of given equations.
GRAPH OF A LINEAR EQUATION
(A) in order to draw the graph of a linear equation in one variable we may follow the following algorithm.
Step I : Obtain the linear equation.

56
b  b 
Step II: If the equation is of the form ax = b, a ≠ o, then plot the point  ,0  and one more point  , α 
a  a 

 b
when α is any real number. If the equation is of the form ay = b, a ≠ 0 , then plot the point  0 ,  and
 a

 b
 β,  where β is any real number.
 a

Step III : Joint the points plotted in step II to obtain the required line.
NOTE :
If eq. is in form ax = b then we get a line parallel to Y-axis and if eg. is in form ay = b then we get a line
parallel to X-axis.
Ex.4 Draw the graph of
(i) 2x + 5 = 0 (ii) 3y - 15 = 0
Sol. (i) Graph of 2x + 5 = 0

5
On simplifying it we get 2x = - 5 ⇒ x = -
2

 5   5 
First we plot point A1  − ,0  & then we plot any other point A2  − ,2  on the graph paper, then we join
 2   2 
these two points we get required line  as shown in figure below.

(ii) Graph of 3y - 15 = 0
15
On simplifying it we get 3u = 15 ⇒ y = f = 5.
3
First we plot the point B1 (0, 5) & then we plot any other point B2 (3, 5) on the graph paper, then we join

these two points we get required line m as shown in figure.

NOTE :
A point which lies on the line is a solution of that equation. A point not lying on the line is not a solution of
the equation.

57
(B) In order to draw the graph of a linear equation ax + by + c = 0 may follow the following algorithm.
Step I : Obtain the linear equation ax + by + c = 0.

 ab + c   by + c 
Step II : Express y in terms of x i.e. y = −   or x in terms of y i.e. x = − .
 b   a 

Step III : Put any two or three values for x or y and calculate the corresponding values of y or x respectively

from the expression obtained in Step II. Let we get points as ( α 1 , β1 ), ( α 2 , β2 ), (α 3 , β3 ) .

Step IV : Plot the points ( α 1 , β 1 ), (α 2 , β 2 ), ( α 3 , β 3 ) on graph paper.

Step V : Joint the pints marked in step IV to obtain. The line obtained is the graph of the equation
ax +by + c = 0.
Ex.5 Draw the graph of the line x – 2y = 3, from the graph find the coordinate of the point when
(i) x = - 5 (ii) y = 0
Sol. Here given equation is x - 2y = 3.
x−3
Solving it for y we get 2y = x - 3 ⇒ y =
2
0 − 3 −3
Let x = 0, then y = =
2 2
3−3
x = 3, then y = =0
2
−2 − 3 −5
x = - 2, then y = = Hence we get
2 2

x 0 3 -2
y 3 0 5
− −
2 2

Clearly when x = - 5 then y = - 4 and when y = 0 then x = 3.

58
Ex.6 Draw the graphs of the lines represented by the equations x + y = 4 and 2x - y = 2 in the same graph. Also
find the coordinate of the point where the two lines intersect.
Sol. Given equations are
x + y = 4 ......(i) & 2x - y = 2 ......(ii)
(i) We have y=4-x

x 0 2 4
y 4 2 0

(ii) We have y = 2x - 2

x 1 0 3
y 0 -2 4

By drawing the lines on a graph paper, clearly we can say that P is the point of intersection where
coordinates are x = 2, y = 2

DIFFERENCE FORMS OF A LINE


(a) Slope of a Line :
If a line makes an angle θ with positive direction of x-axis then tangent of this angle is called the slope of a
line, it is denoted by m i.e. m = tan θ .
(i) Slope - intercept from is y = mx + c where m is the slope of line and c is intercept made by line with
Y-axis.

59
(ii) The equation of a line passing through origin is y = mx. Here c= 0 then the line passes always from
origin.

x y
(iii) Intercept from of line is + = 1 where a & b are intercepts on positive direction of x-axis and y-axis
a b
respectively made by line.

SOLUTION OF LINEAR EQUATION IN ONE VARIABLE


b
Let ax + b = 0 is one equation then ax + b = 0 ⇒ ax + - b ⇒ x = - is a solution.
a
x x
Ex.7 Solve : = 3+
2 3
x x x x
Sol. Given = 3+ ⇒ − = 3
2 3 2 3
3x − 2 x
⇒ =3
6
x
⇒ =3
6
⇒ x = 18 Ans.

60
SOLUTION OF LINEAR EQUATIONS IN TWO VARIABLE
(a) By Elimination of Making Equal Coefficient :
Ex.8 Solve the following equations
2x - 3y = 5
3x + 2y = 1
Sol. Given eq. are 2x - 3y = 5 .....(i)
3x + 2y = 1 ....(ii)
Multiplying 1 eg.(i) by 3 and eg. (ii) by 2 we get
6x − 9 y = 15
On subtraction _ 6x + _ 4 y = _ 2
− 9 y − 4 y = 15 − 2
⇒ -13y = 13
13
⇒ y=
− 13
⇒ y = -1
Put the value of y in eg. (i) we get
2x - (3) × (-1) = 5
2x + 3 = 5
⇒ 2x = 5 - 3
⇒ 2x = 2
⇒ x=1
∴ x = 1, y = 1 Ans.
(b) Substitution Method :
Ex.9 Solve x + 4y = 14 ....(i)
7x - 3y = ...(ii)
Sol. From equation (i) x = 14 - 4y
Substitute the value of x in equation (ii)
⇒ 7 (14 - 4y) - 3y = 5
⇒ 98 - 28y - 3y = 5
⇒ 98 - 31y = 5
⇒ 93 = 31y
93
⇒ y=
31
⇒ y=3
Now substitute value of y in equation (ii)
⇒ 7x - 3 (3) = 5
⇒ 7x - 3 (3) = 5
⇒ 7x = 14
14
⇒ x= =2
7
So, solution is x = 2 and y = 3. Ans.

61
EXERCISE

OBJECTIVE DPP # 7.1

1. Which of the following equation is not linear equation ?

2
(A) 2x + 3 = 7x - 2 (B) x + 5 = 3x - 4 (C) x2 + 3 = 5x - 3 (D) (x - 2)2 = x2 + 8
3

2. Solution of equation 3 x - 2 = 2 3 + 4 is

(A) 2( 3 − 1) (B) 2(1 − 3 ) (C) 1 + 3 (D) 2(1 + 3 )

6 x + 5 3x + 5
3. The value of x which satisfy = is
4x + 7 2 x + 6

(A) -1 (B) 1 (C) 2 (D) -2

x−a x−b x−c


4. Solution of + + = 3 is
b+c c+a a+b

(A) a + b - c (B) a - b + c (C) -a + b + c (D) a + b + c

5. A man is thrice as old as his son. After 14 years, the man will be twice as old as his son, then present age of

this son.

(A) 42 years (B) 14 years (C) 12 years (D) 36 years

6. One forth of one third of one half of a number is 12, then number is

(A) 284 (B) 286 (C) 288 (D) 290

7. A linear equation in two variables has maximum

(A) only one solution (B) two solution (C) infinite solution (D) None of these

8. Solution of the equation x - 2y = 2 is/are

(A) x = 4, y = 1 (B) x = 2, y = 0 (C) x = 6, y = 2 (D) All of these

9. The graph of line 5x + 3y = 4 cuts Y-axis at the point

 4  3 4  5 
(A)  0 ,  (B)  0 ,  (C)  ,0  (D)  ,0 
 3  4 5  4 

10. If x = 1, y = 1 is a solution of equation 9ax + 12ay = 63 then, the value of a is

(A) -3 (B) 3 (C) 7 (D) 5

62
SUBJECTIVE DPP - 7.2
Solve the following linear equations in one variable
2 x + 7 4x + 3
1. If = , find the value of x3 + x2 + x + 1.
x+2 2x − 7

2. Determine whether x = 5, y = 4 is a solution of the equation x - 2y = - 3


Solve the following linear equations in two variable.
3. 8x - 5y = 34, 3x - 2y = 13
4. 20x + 3y = 7, 8y - 15x = 5
2x 1
5. 2x - 3y - 3 = 0, + 4y + =0
3 2

6. Draw the graph of 2x + 3y = 6 and use it to find the area of triangle formed by the line and co-ordinate axis.
7. Draw the graph of the lines 4x - y = 5 and 5y - 4x = 7 on the same graph paper and find the coordinates of
their point of intersection.
8. Find two numbers such that five times the greater exceeds four times the lesser by 22 and three times the
greater together with seven times the lesser is 32.
9. Draw the graph of x - y + 1 = 0 and 3x + 2y - 12 = 0 on the same graph. Calculate the area bounded by these
lines & X-axis.
1
10. If p = 3x + 1, q = (9x + 13) and p : q = 6 : 5 then find x.
3

63
ANSWER KEY

(Objective DPP # 7.1)


Qus. 1 2 3 4 5 6 7 8 9 10
Ans. C D B D B C C D A B

(Subjective DPP # 7.2)


1
1. -104 2. Yes 3. x = 3, y = -2 4. x= ,y=1
5

21 3
5. x= ,y = − 6. Area = 3 sq. units 7. x = 2, y = 3
20 10

8. 6,2 9. 7.5 sq. units 10. -7

64
INTRODUCTION
TO EUCLID’S GEOMETRY
ML - 8

INTRODUCTION
The credit for introducing geometrical concepts goes to the distinguished Greek mathematician ‘Euclid’
who is known as the “Father of Geometry” and the word ‘geometry’ comes from the Geek words ‘geo’
which means ‘Earth’ and ‘metreon’ which means ‘measure’.

BASIC CONCEPTS IN GEOMETRY


A ‘point’, a ‘line’ and a ‘plane’ are the basic concepts to be used in geometry.
(a) Axioms :
The basic facts which are granted without proof are called axioms.
(b) Euclid’s Definitions :
(i) A point is that which has not part.
(ii) A line is breathless length.
(iii) The ends of a line segment are points.
(iv) A straight line is that which has length only.
(v) A surface is that which has length and breadth only.
(vi) The edges of surface are lines.
(vii) A plane surface is that which lies evenly with the straight lines on itself.
(c) Euclid’s Five Postulates :
(i) A straight line may be drawn from any one point to any other point.
(ii) A terminated line or a line segment can be produced infinitely.

(iii) A circle can be drawn with any centre and of any radius.
(iv) All right angles are equal to one another.
(v) If a straight line falling on two straight lines makes the exterior angles on the same side of it taken
together less than two right angles, then the two straight lines if produced infinitely meet on that side on
which the sum of angles are less than two right angles.

65
(d) Important Axioms :
(i) A line is the collection of infinite number of points.
(ii) Through a given point, an infinite lines can be drawn.

(iii) Given two distinct points, there is one and only one line that contains both the points.
(iv) If P is a point outside a line  , then one and only one line can be drawn through P which is parallel to
.

(v) Two distinct lines can not have more than one point in common.

(vi) Two lines which are both parallel to the same line, are parallel to each other.
i.e.  n , m n ⇒  m

SOME IMPORTANT DEFINITOINS


(i) Collinear points : Three or more points are said to be collinear if there is a line which contains all of
them.

(ii) Concurrent Lines : Three or more lines are said to be concurrent if there is a point which lies on all of
them.

66
(iii) Intersecting lines : Two lines are intersecting if they have a common point. The common point is called
the “point of intersection”.

(iv) Parallel lines : Two lines I and m in a plane are said to be parallel lines if they do not have a common
point.

(v) Line Segment : Given two points A and B on a line I, the connected part (segment) of the line with end
points at A and B, is called the line segment AB.

(vi) Interior point of a line segment : A point R is called an interior point of a line segment PQ if R lies
between P and Q but R is neither P nor Q.

(vii) Congruence of line segment : Two line segments AB and CD are congruent if trace copy of one can be
superposed on the other so as to cover it completely and exactly in this case we write AB ≅ CD. In other
words we can say two lines are congruent if their lengths is same.
(viii) Distance between two points : The distance between two points P and Q is the length of line segment
PQ

(ix) Ray : Directed line segment is called a ray. If AB is a ray then it is denoted by AB . Point A is called
initial point of ray.

(x) Opposite rays : Two rays AB and AC are said to be opposite rays if they are collinear and point A is the
only common point of the two rays.

1
Ex.1 If a point C lies between two points A and B such that AC = BC, then prove that AC = AB. Explain by
2
drawing the figure.
Sol. According to the given statement, the figure will be as shown alongside in which the point C lies between
two points A and B such that AC = BC.
Clearly, AC + BC = AB
⇒ AC + AC = AB [ ∵ AC = BC]
⇒ 2AC = AB
1
And, AC = AB
2

67
Ex.2 Give a definition for each of the following terms. Are there other terms that need to be defined first ? What
are they, and how might you define them ?
(i) parallel lines (i) perpendicular lines (iii) line segment (iv) radius
Sol. (i) Parallel lines : Lines which don’t intersect any where are called parallel lines.
(ii) Perpendicular lines : Two lines which are at a right angle to each other are called perpendicular lines.
(iii) Line segment : it is a terminated line.
(iv) Radius : The length of the line-segment joining the centre of a circle to any point on its circumference is
called its radius.
Ex.3 How would you rewrite Euclid’ fifth postulate so that it would be easier to understand ?
Sol. Two distinct intersecting lines cannot be parallel to the same line.
Ex.4 Does Euclid’s fifth postulate imply the existence of parallel lines ? Explain.
Sol. if a straight line  falls on two straight lines m and n such that sum of the interior angles on one side of  is
two right angles, then by Euclid’s fifth postulate the line will not meet on this side of  . Next, we know that
the sum of the interior angles on the other side of line  also be two right angles. Therefore they will not
meet on the other side. So, the lines m and n never meet and are, therefore parallel.
Theorem 1 : If  , m, n are lines in the same plane such that  intersects m and n ║ m, then  intersects n also.
Given : Three lines  , m, n in the same plane s.t.  intersects m and n ║ m.
To prove : Lines  and n are intersecting lines.

Proof : Let  and n be non intersecting lines. Then.  ║ n.


But, n ║ m [Given]
∴  ║ n and n ║ m ⇒  ║ m
⇒  and m are non-intersecting lines.
This is a contradiction to the hypothesis that  and m are intersecting lines.
So our supposition is wrong.
Hence,  intersects line n.
Theorem 2 : If lines AB, AC, AD and AE are parallel to a line  , then points A, B, C, D and E are collinear.
Given : Lines AB, AC, AD and AE are parallel to a line  .
To prove : A, B, C, D, E are collinear.
Proof : Since AB, AC, AD and AE are all parallel to a line  Therefore point A is outside  and lines AB,
AC, AD, AE are drawn through A and each line is parallel to  .
But by parallel lines axiom, one and only one line can be drawn through the point A outside it and
parallel to  .
This is possible only when A, B, C, D, and E all lie on the same line. Hence, A, B, C, D and E are collinear.

68
EXERCISE

SUBJECTIVE DPP # 8.1


1. How many lines can pass through :
(i) one point (ii) two distinct points
2. Write he largest number of points in which two distinct straight lines may intersect.
3. A, B and C are three collinear points such that point A lines between B and C. Name all the line segments
determined by these points and write the relation between them.

4. State, true of false :


(i) A point is a undefined term
(ii) A line is a defined term.
(iii) Two distinct lines always intersect at one point.
(iv) Two distinct point always determine a line.
(v) A ray can be extended infinitely on both the sides of its.
(vi) A line segments has both of its end-points fixed and so it has a definite length.
5. Name three undefined terms.
6. If AB is a line and P is a fixed point, outside AB, how many lines can be drawn through P which are :
(i) parallel to AB
(ii) Not parallel to AB
7. Out of the three lines AB, CD and EF, if AB is parallel to EF and CD is also parallel to EF, then what is the
relation between AB and CD.
8. If A, B and C three points on a line, and B lines between A and C, then prove that :
AB + BC = AC.
9. In the given figure, if AB = CD ; prove that AC = BD.

10. (i) How many lines can be drawn to pass through three given point if they are not collinear ?
(ii) How many line segments can be drawn to pass through there two given points if they are collinear

69
ANSWER KEY

(Subjective DPP # 8.1)


1. (i) Infinite (ii) Only one

2. One

3. BA, AC & BC ; BA + AC = BC

4. (i) True (ii) False


(iii) False (iv) True
(v) False (vi) True

5. Point, line and plane

6. (i) Only one (ii) Infinite

7. AB ║ CD

10. (i) Three lines (ii) one

70
LINES AND ANGLES

ML - 9

LINE
A line has length but no width and no thickness.
ANGLE
An angle is the union of two non-collinear rays with a common initial point. The common initial point is
called the ‘vertex’ of the angle and two rays are called the ‘arms’ of the angles.

REMAK :
Every angle has a measure and unit of measurement is degree.
One right angle = 900
10 = 60’ (minutes)
1’ = 60” (Seconds)
Angle addition axiom : If X is a point in the interior of ∠BAC, then m ∠BAC = m ∠BAX + m ∠XAC

(a) Types of Angles :


(i) Right angles : An angle whose measure is 90 0 is called a right angle.

71
(ii) Acute angle : An angle whose measure is less than 900 is called an acute angle.

0 0 < ∠BOA < 900

(iii) Obtuse angle : An angle whose measure is more than 900 but less than 180 0 is called an obtuse angle.

900 < ∠AOB < 180 0

(iv) Straight angle : An angle whose measure is 180 0 is called a straight angle.

(v) Reflex angle : An angle whose measure is more than 1800 is called a reflex angle.

180 0 < ∠AOB < 360 0

(vi) Complementary angles : Two angles, the sum of whose measures is 900 are called complementary
angles.

∠AOC & ∠BOC are complementary as ∠AOC + ∠BOC = 900

(vii) Supplementary angles : Two angles, the sum of whose measures is 180 0, are called the supplementary
angles.

∠AOC & ∠BOC are supplementary as their sum is 1800.

72
(viii) Angle Bisectors : A ray OX is said to be the bisector of ∠AOB, if X is a point in the interior of ∠AOB,
and ∠AOX = ∠BOX.

(ix) Adjacent angles : Two angles are called adjacent angles, it


(A) they have the same vertex,
(B) they have a common arm,
(C) non common arms are on either side of the common arm.

∠AOX and ∠BOX are adjacent angles, OX is common arm, OA and OB are non common arms and lies on
either side of OX.

(x) Linear pair of angles : Two adjacent angles are said to form a linear pair of angles, if their non common
arms are two opposite rays.

(xi) Vertically opposite angles : Two angles are called a pair of vertically opposite angles, if their arms
form two pairs of opposite rays.

∠AOC & ∠BOD from a pair of vertically opposite angles. Also ∠OD & ∠BOC form a pair of vertically
opposite angles.

73
(b) Angles Made by a Transversal with two Parallel Lines :
(i) Transversal : A line which intersects two or more give parallel lines at distinct points is called a
transversal of the given lines.

(ii) Corresponding angles : Two angles on the same side of transversal are known as the corresponding
angles if both lie either above the two lines or below the two lines, in figure ∠1 & ∠5, ∠4 & ∠8, ∠2 & ∠6, ∠3
& ∠7 are the pairs of corresponding angles.
(iii) Alternate interior angles : ∠3 & ∠5, ∠2 & ∠8, are the pairs of alternate interior angles.
(iv) Consecutive interior angles : The pair of interior angles on the same side of the transversal are called
pairs of consecutive interior angles. In figure ∠2 & ∠5, ∠3 & ∠8, are the pair of consecutive interior angles.
(v) Corresponding angles axiom :
It a transversal intersects two parallel lines, then each pair of corresponding angles are equal. Conversely, if
a transversal intersects two lines, making a pair of equal corresponding angles, then the lines are parallel.
(c) Important Facts to Remember :
(i) If a ray stands on line, then the sum of the adjacent angles so formed is 1800.
(ii) If the sum of two adjacent angles is 1800, then their non common arms are two apposite rays.
(iii) The sum of all the angles round a point is equal to 3600
(iv) If two lines intersect, then the vertically opposite angles are equal.
(v) If a transversal interests two parallel lines then the corresponding angles are equal, each pair of alternate
interior angles are equal and each pair of consecutive interior angles are supplementary.
(vi) if a transversal intersects two lines in such a way that a pair of alternet interior angles are equal, then
the two lines are parallel.
(vii) If a transversal intersects two lines in such a way that a pair of consecutive interior angles are
supplementary, then the two lines are parallel.
(viii) If two parallel lines are intersected by a transversal, the bisectors of any pair of alternate interior
angles are parallel and the bisectors of an two corresponding angles are also parallel.
(ix) If a line i s perpendicular to one or two given parallel, lines, then it is also perpendicular to the other
line.
(x) Two angles which have their arms parallel are either equal or supplementary.
(xi) Two angles whose arms are perpendicular are either equal or supplementary.

74
IMPORTANT THEOREMS
Theorem 1 : If two lines intersect each other, then the vertically opposite angles are equal.
Given : Two lines AB and CD intersecting at a point O.

To prove : (i) ∠AOC = ∠BOD


(ii) ∠BOC = ∠AOD
Proof : Since ray OD stands on AB
∴ ∠AOD + ∠DOB = 180 0 ...(i) [linear pair]
again, ray OA stands on CD
∴ ∠AOC + ∠AOD = 1800 ...(ii) [linear pair]
by (i) & (ii) we get
∠AOD + ∠DOB = ∠AOC + ∠AOD
⇒ ∠DOB = ∠AOC
⇒ ∠AOC = ∠DOB
Similarly we can prove that ∠BOC = ∠DOA Hence Proved.
Theorem 2 : If a transversal intersects two parallel lines, then each pair of alternate interior angles is equal.
Given : AB and CD are two parallel lines, Transversal I intersects AB and CD at P and Q respectively
making two pairs of alternate interior angles, ∠1, ∠2 & ∠3, ∠4.

To prove : ∠1 = ∠2 and ∠3 = ∠4
Proof : Clearly,∠2 = ∠5 [Vertically opposite angles]
And, ∠1 = ∠5 [Corresponding angles]
∴ ∠1 = ∠2
Also, ∠3 = ∠6 [Vertically opposite angles]
And, ∠4 = ∠6 [Corresponding angles]
∴ ∠3 = ∠4 Hence, Proved.

75
ILLUSTRATIONS
Ex.1 Two supplementary angles are in ratio 4 : 5, find the angles,
Sol. Let angles are 4x & 5x.
∴ Angles are supplementary
∴ 4x + 5x = 1800 ⇒ 9x = 1800

1800
⇒ x= = 200
9
∴ Angles are 4 × 200, 5 × 200 ⇒ 800 & 1000 Ans.

Ex.2 If an angle differs from its complement by 10, find the angle.
Sol. let angles is x0 then its complement is 90 - x0.
Now given x0 - (90 - x0) = 10
⇒ x0 - 90 0 + x0 = 10
⇒ 2x0 = 10 + 90 = 100

100 0
⇒ x0 = = 50 0
2
∴ Required angle is 500. Ans.
Ex.3 In figure, OP and OQ bisects ∠BOC and ∠AOC respectively. Prove that ∠POQ = 900.

Sol. ∴ OP bisects ∠BOC


1
∴ ∠POC = ∠BOC ...(i)
2
Also OQ bisects ∠AOC
1
∴ ∠COQ = ∠AOC ...(ii)
2
∴ OC stands on AB
∴ ∠AOC + ∠BOC = 180 0 [Linear pair]
1 1 1
⇒ ∠AOC + ∠BOC = × 1800
2 2 2
⇒ ∠COQ + ∠POC = 900 [Using (i) & (ii)]
⇒ ∠POQ = 90 0 [By angle sum property]
Hence Proved.

76
Ex.4 In figure, lines AB, CD and EF intersect at O. Find the measures of ∠AOC, ∠DOE and ∠BOF

Sol. Given ∠AOE = 400 & ∠BOD = 35 0


Clearly ∠AOC = ∠BOD [Vertically opposite angles]
⇒ ∠AOC = 350 Ans.
∠BOF = ∠AOE [Vertically opposite angles]
0
⇒ ∠BOF = 40 Ans.
Now, ∠AOB = 180 0 [Straight angles]
0
⇒ ∠AOC + ∠COF + ∠BOF = 180 [Angles sum property]
0 0 0
⇒ 35 + ∠COF + 40 = 180
⇒ ∠COF = 1800 - 750 = 105 0
Now, ∠DOE = ∠COF [Vertically opposite angles]
∴ ∠DOE = 1050 Ans.

Ex.5 In figure if I║m , n║p and ∠1 = 850 find ∠2

Sol. ∴ n║p and m is transversal


∴ ∠1 = ∠3 = 850 [Corresponding angles]
Also m║I & p is transversal
∴ ∠2 + ∠3 = 1800 [ ∵ Consecutive interior angles]
⇒ ∠2 + 85 0 = 1800
⇒ ∠2 + 1800 - 850
⇒ ∠2 = 95 0 Ans.

77
EXERCISE

OBJECTIVE DPP # 9.1


1. If two lines intersected by a transversal, then each pair of corresponding angles so formed is “
(A) Equal (B) Complementary (C) Supplementary (D) None of these
2. Two parallel lines have :
(A) a common point (B) two common point
(C) no any common point (D) infinite common points
0
3. An angle is 14 more than its complementary angle then angle is :
(A) 380 (B) 520 (C) 50 0 (D) none of these
4. The angle between the bisectors of two adjacent supplementary angles is :
(A) acute angle (B) right angle (C) obtuse angle (D) none of these
5. If one angle of triangle is equal to the sum of the other two then triangle is :
(A) acute a triangle (B) obtuse triangle
(C) right triangle (D) none
6. X lines in the interior of ∠BAC. If ∠BAC = 70 and ∠BAX = 42 0 then ∠XAC =
0

(A) 280 (B) 290 (C) 27 0 (D) 30 0


7. If the supplement of an angle is three times its complement, then angle is :
(A) 400 (B) 350 (C) 50 0 (D) 45 0
4a
8. Two angles whose measures are a & b are such that 2a - 3b = 600 then = ? If they form a linear pair :
5b
8 1 2
(A) 0 (B) (C) (D)
5 2 3
9. Which one of the following statements is not false :
(A) if two angles forming a linear pair, then each of these angles is of measure 900
(B) angles forming a linear pair can both be acute angles
(C) one of the angles forming a linear pair can be obtuse angle
(D) bisectors of the adjacent angles form a right angle

10. Which one of the following is correct :


(A) If two parallel lines are intersected by a transversal, then alternate angles are equal
(B) If two parallel lines are intersected by a transversal then sum of the interior angles on the same side of
transversal is 1800
(C) If two parallel lines intersected by a transversal then corresponding angles are equal
(D) All of these

78
SUBEJCTIVE DPP # 9.2

1. The supplement of an angle is one third of itself. Determine the angle and its supplement.

2. Two complementary angles are such that two times the measure of one is equal to three times measure of
the other. Find the measure of the large angle.

3. Find the complement of each of the following angles.


(A) 36040’ (B) 42025’36”

4. Write the supplementary angles of the following anglels .



0
(A) 54 28' (B) 98 0 35’ 20”

5. In figure, if ∠BOC = 7x + 200 and ∠COA = 3x, then find the value of x for which AOB becomes a straight
line.

6. In figure, if x + y = w + z then prove that AOB is a straight line.

7. If the bisectors of two adjacent angles form a right angle prove that their non common angles are in the
same straight line.

8. In figure, find ∠COD when ∠AOC + ∠BOD = 100 0.

9. In figure x : y : z = 5 : 4 : 6. if XOY is a straight line find the values of x, y and z.

79
10. In the given figure, AB is a mirror, PO is the incident ray and OR, the reflected ray. If ∠POR = 1120 find
∠POA

11. In figure, if AB║ CD ║EF and y : x = 3 : 7 find x

12. In figure if AB║CD, EF ⊥ CD and ∠GED = 1260, find ∠AGE, ∠GEF and ∠FGE.

13. ∆ ABC is an isosceles triangle in which ∠B = ∠V and L & M are points on AB & AC respectively such that
LM║BC. If ∠A = 500 find ∠LMC.

14. In figure if AB║DF, AD║FG, ∠BAC = 650, ∠ACB = 550. Find ∠FGH

15. In figure, AB║ED and ∠ABC = 30 0, ∠EDC = 700 then find x0.

80
ANSWER KEY

(Objective DPP # 9.1)

Qus. 1 2 3 4 5 6 7 8 9 10

Ans. D C B B C A D B C D

(Subjective DPP # 9.2)


1. 1350, 450 2. 540
3. (A) 53020’ (B) 47034’ 24”
4. (A) 125 032’ (B) 81024’ 40”
5. 16 0 8. 800
9. 60 0,48 0,72 0 10. 34
0
11. 126 12. 126 0,36 0,54 0
13. 1150 14. 125 0
15. 2600

81
TRIANGLES

ML - 10
TRIANGLE
A plane figure bounded by three lines in a plane is called a triangle. Every triangle have three sides and
three angels. If ABC is any triangle then AB, BC & CA are three sides and ∠A, ∠B and ∠C are three angles.

(a) Types of Triangles :


(i) On the basis of sides we have three types of triangles:
(A) Scalene triangle : A triangle whose no two sides are equal is called a scalene triangle.
(B) Isosceles triangle - A triangle having two sides equal is called an isosceles triangle.
(C) Equilateral triangle - A triangle in which all sides are equal is called an equilateral triangle.
(ii) On the basis of angles we have three types of triangles :
(A) Right triangle - A triangle in which any one angle is right angle (=900) is called right triangle.
(B) Acute triangle - A triangle in which all angles are acute (>900) is called an acute triangle.
(C) Obtuse triangle - A triangle in which any one angle is obtuse (<90 0) is called an obtuse triangle.

CONGRUENT FIGURES
The figures are called congruent if they have same shape and same size. In order words, two figures are
called congruent if they are having equal length, width and height.

Fig. (i) Fig. (ii)


In the above figures {fig. (i) and fig. (ii)} both are equal in length, width and height, so these are congruent
figures.

82
(a) Congruent Triangles :
Two triangles are congruent if and only if one of them can be made to superimposed on the other, so an to
cover it exactly.

If two triangles ∆ABC and ∆DEF are congruent then there exist a one to one correspondence between their
vertices and sides. i.e. we get following six equalities.
∠A = ∠D, ∠B = ∠E, ∠C = ∠F and AB = DE, BC = EF, AC = DF.
If two ∆ABC & ∆DEF are congruent under A ↔ D, B ↔ E, C ↔ F one to one correspondence then we
write ∆ABC ≅ ∆DEF we can not write as ∆ABC ≅ ∆DFE of ∆ABC ≅ ∆EDF or in other forms because
∆ABC ≅ ∆DFE have following one-one correspondence A ↔ D, B ↔ F, C ↔ E.
Hence we can say that “two triangles are congruent if and only if there exists a one-one correspondence
between their vertices such that the corresponding sides and the corresponding angles of the two triangles
are equal.
(b) Sufficient Conditions for Congruence of two Triangles :
(i) SAS Congruence Criterion :

Two triangles are congruent if two sides and the included angle of one are equal to the corresponding sides
and the included angle of the other triangle.
(ii) ASA Congruence Criterion :

Two triangles are congruent if two angles and the included side of one triangle are equal to the
corresponding two angles and the included side of the other triangle.
(iii) AAS Congruence Criterion :

If any two angles and a non included side of one triangle are equal to the corresponding angles and side of
another triangle, then the two triangles are congruent.

83
(iv) SSS Congruence Criterion :

Two triangles are congruent if the three sides of one triangle are equal to the corresponding three sides of
the other triangle.
(v) RHS Congruence Criterion :

Two right triangles are congruent if the hypotenuse and one side of one triangle are respectively equal to
the hypotenuse and one side of the other triangle.
(c) Congruence Relation in the Set of all Triangles :
By the definition of congruence of two triangles, we have following results.
(I) Every triangle is congruent to itself i.e. ∆ABC ≅ ∆ABC
(II) If ∆ABC ≅ ∆DEF then ∆DEF ≅ ∆ABC
(III) If ∆ABC ≅ ∆DEF and ∆DEF ≅ ∆PQR then ∆ABC ≅ ∆PQR
NOTE : If two triangles are congruent then their corresponding sides and angles are also congruent by
cpctc (corresponding parts of congruent triangles are also congruent).
Theorem-1 : Angles opposite to equal sides of an isosceles triangle are equal.
Given : ∆ABC in which AB = AC
To Prove : ∠B = ∠C
Construction : We draw the bisector AD of ∠A which meets BC in D.

Proof : In ∆ABD and ∆ACD we have


AB = AC [Given]
∠BAD = ∠CAD [ ∵ AD is bisector of ∠A]
And, AD = AD [Common side]
∴ By SAS criterion of congruence, we have
∆ABD ≅ ∆ACD
⇒ ∠B = ∠C by cpctc Hence Proved.

84
Theorem - 2: if two angles of a triangle are equal, then sides opposite to them are also equal.
Given : ∆ABC in which ∠B = ∠C
To Prove : AB = AC
Construction: We draw the bisector of ∠A which meets BC in D.

Proof : In ∆ABD and ∆ACD we have


∠B = ∠C [Given]
∠BAD = ∠CAD [ ∵ AD is bisector of ∠A]
AD = AD [Common side]
∴ By AAS criterion of congruence, we get
∆ABD ≅ ∆ACD
⇒ AB = AC [By cpctc] Hence, Proved.
Theorem-3 : if the bisector of the vertical angle bisects the base of the triangle, then the triangle is
isosceles.
Given : A ∆ABC in which AD is the bisector of ∠A meeting BC in D such that BD = CD
To Prove : ∆ACD is an isosceles triangle.
Construction : We produce AD to E such that AD = DE and join EC.

Proof : In ∆ADB and ∆EDC we have


AD = DE [By construction]
∠ADB = ∠CDE [Vertically opposite angles]
BD = DC [Given]
∴ By SAS criterion of congruence, we get
∆ADB ≅ ∆EDC ⇒ AB = EC ...(i)
And, ∠BAD = ∠CED [By cpctc]
But, ∠BAD = ∠CAD
∴ ∠CAD = ∠CED
⇒ AC = EC [Sides opposite to equal angles are equal]
⇒ AC = AB [By eg. (i)] Hence Proved.

85
Ex.1 Prove that measure of each angle of an equilateral triangle is 600.
Sol. Let ∆ABC be an equilateral triangle, then we have

AB = BC = CA ...(i)
∴ AB = BC
∴ ∠C = ∠A ...(ii) [Angles opposite to equal sides are equal]
Also, BC = CA
∴ ∠A = ∠B ...(iii) [Angles opposite to equal sides]
By (ii) & (iii) we get ∠A = ∠B = ∠C
Now in ∆ABC ∠A + ∠B + ∠C = 1800
⇒ 3∠A = 180 0 [ ∴ ∠A = ∠B = ∠C]
0
⇒ ∠A = 60 = ∠B = ∠C Hence Proved.
1
Ex.2 If D is the mid-point of the hypotenuse AC of a right triangle ABC, prove that BD = AC.
2
Sol. Let ∆ABC is a right triangle such that ∠B = 900 and D is mid point of AC then we have to prove that BD =
1
AC we produce BD to E such that BD = AC and EC.
2
Now is ∆ADB and ∆CDE we have
AD = DC [Given]
BD = DE [By construction]
And, ∠ADB = ∠CDE [Vertically opposite angles]
∴ By SAS criterion of congruence we have
∆ADB ≅ ∆CDE
⇒ EC = AB and ∠CED = ∠ABD ....(i) [By cpctc]
But ∠CED & ∠ABD are alternate interior angles
∴ CE ║ AB ⇒ ∠ABC + ∠ECB = 1800 [Consecutive interior angles]
⇒ 90 + ∠ECB = 1800
⇒ ∠ECB = 90 0
Now, In ∆ABC & ∆ECB we have
AB = EC [By (i)]
BC = BC [Common]
And, ∠ABC = ∠ECB = 900
∴ BY SAS criterion of congruence
∆ABC ≅ ∆ECB
⇒ AC = EB [By cpctc]
1 1
⇒ AC = EB
2 2
1
⇒ BD = AC Hence Proved.
2

86
Ex.3 In a right angled triangle, one acute angle is double the other. Prove that the hypotenuse is double the
smallest side.
Sol. Let ∆ABC is a right triangle such that ∠B = 900 and ∠ACB = 2∠CAB, then we have to prove AC = 2BC.
we produce CB to D such that BD = CB and join AD.
Proof : In ∆ABD and ∆ABC we have
BD = BC [By construction]
AB = AB [Common]
∠ABD = ∠ABC = 900
∴ By SAS criterion of congruence we get
∆ABD ≅ ∆ABC
⇒ AD = AC and ∠DAB = ∠CAB [By cpctc]
⇒ AD = AC and ∠DAB = x [ ∴ ∠CAB = x]
Now, ∠DAC = ∠DAB + ∠CAB = x + x = 2x
∴ ∠DAC = ∠ACD
⇒ DC = AD [Side Opposite to equal angles]
⇒ 2BC = AD [ ∵ DC = 2BC
⇒ 2BC = AC [AD = AC] Hence Proved.
Ex.4 In figure, two sides AB and BC and the median AM of a ∆ABC are respectively equal to sides DE and EF
and the median DN of ∆DEF. Prove that ∆ABC ≅ ∆DEF.

Sol. ∴ AM and DN are medians of ∆ABC & ∆DEF respectively


∴ BM = MC & EN = NF
1 1
⇒ BM = BC & EN = EF
2 2
But, BC = EF ∴ BM = EN ...(i)
In ∆ABM & ∆DEN we have
AB = DE [Given]
AM = DN [Given]
BM = EN [By (i)]
∴ By SSS criterion of congruence we have
∆ABM ≅ ∆DEN ⇒ ∠B = ∠E ...(ii) [By cpctc]
Now, In ∆ABC & ∆DEF
AB = DE [Given]
∠B = ∠E [By (ii)]
BC = EF [Given]
∴ By SAS criterion of congruence we get
∆ABC ≅ ∆DEF
Hence Proved.

87
SOME INEQUALITY RELATIONS IN A TRIANGLE

(i) If two sides of triangle are unequal, then the longer side has greater angle opposite to it. i.e. if in any
∆ABCAB > AC then ∠C > ∠B.
(ii) In a triangle the greater angle has the longer side opposite to it.
i.e. if in any ∆ABC ∠A > ∠B then BC > AC.
(iii) The sum of any two sides of a triangle is greater than the third side.
i.e. if in any ∆ABC, AB + BC > AC, BC + CA > AB and AC + AB > BC.
(iv) Of all the line segments that can be drawn to a given line, from a point, not lying on it, the
perpendicular line segment is the shortest.

P is any point not lying online  , PM ⊥ then PM < P N.


(v) The difference of any two sides of a triangle is less than the third side.
i.e. In any ∆ABC, AB - BC < AC, BC - CA <AB and AC < AB < BC.
Ex.5 In figure, PQ = PR, show that PS > PQ

Sol. In ∆PQR
∴ PQ = PR
⇒ ∠PRQ = ∠PQR ....(i) [Angles opposite to equal sides]
In ∆PSQ, SQ is produced to R
∴ Ext. ∠PQR > ∠PSQ ....(ii)
∠PRQ > ∠PSQ [By eq. (i) and (ii)]
⇒ ∠PRS > ∠PSR
⇒ PS > PR [Sides opposite to greater angles is larger]
But, PR = PQ
∴ PS > PQ Hence Proved.
Ex.6 In figure, T is a point on side QR of ∆PQR and S is a point such that RT = ST. Prove that PQ + PR > QS
Sol. In ∆PQR we have
PQ + PR > QR
⇒ PQ + PR > QT + TR
⇒ PQ + PR > QT + ST ∴ RT = ST
In ∆QST QT + ST > SQ
∴ PQ + PR > SQ Hence Proved.

88
EXERCISE

OBJECTIVE DPP # 10.1


1. In the three altitudes of a ∆ are equal then triangle is :
(A) isosceles (B) equilateral (C) right angled (D) none
2. ABCD is a square and P, Q, R are points on AB, BC and CD respectively such that AP = BQ = CR and
∠PQR = 900, then ∠QPR
(A) 450 (B) 500 (C) 600 (D) LM
3. In a ∆XYZ, LM ║ YZ and bisectors YN and ZN of ∠Y & ∠Z respectively meet at N on LM then YL + ZM =
(A) YZ (B) XY (C) XZ (D) LM
4. In a ∆PQR, PS is bisector of ∠P and ∠Q = 700 ∠R = 300, then
(A) QS > PQ > PR (B) QS < PQ < PR (C) PQ > QS > SR (D) PQ < QS < SR
5. If D is any point on the side BC of a ∆ABC, then :
(A) AB + BC + CA > 2AD (B) AB + BC + CA < 2AD
(C) AB + BC + CA > 3AD (D) None
6. For given figure, which one is correct :

(A) ∆ABC ≅ ∆DEF (B) ∆ABC ≅ ∆FED (C) ∆ABC ≅ ∆DFE (D) ∆ABC ≅ ∆EDF
7. In a right angled triangle. One acute angle is double the other then the hypotenuse is :
(A) Equal to smallest side (B) Double the smallest side
(C) Triple the smallest side (D) None of these

SUBJECTIVE DPP - 10.2


1. In the ∆ABC given below, BD bisects ∠B and is perpendicular to AC. If the lengths of the sides of the
triangle are expressed in terms of x and y as shown, find the values of x and y.

89
2. In the figure, AB = AD prove that ∠BCD is a right angle.

3. If the bisector of an angle of a triangle also bisects the opposite side, prove that the triangle is isosceles.

4. AD is meadian of ∆ABC. Prove that AB + AC > 2 AD.

5. O is any point in the interior of a triangle ABC. Prove that OB + OC < AB + AC.

6. In figure, ∆ABC is a right angled triangle at B. ADEC and BCFG are square Prove that AF = BE.

7. In figure CD is the diameter perpendicular to the chord AB of a circle with centre O. Prove that
(a) ∠CAO = ∠CBO (b) ∠AOB = 2∠ACB

8. ABCD is a square and EF║BD. E and F are the mid point of BC and DC respectively. Prove that
(a) BE = DF (b) AR bisects ∠BAD

90
9. In figure, ∆ABC is an equilateral triangle PQ║AC and AC is produced to R such that CR = PQ. Prove that
QR bisects PC.

10. In figure, the congruent parts of triangles have been indicated by line markings. Find the values of x & y.

ANSWER KEY

(Objective DPP # 10.1)

Qus. 1 2 3 4 5 6 7

Ans. B A D B A C B

(Subjective DPP # 10.2)

1. 16, 8 10. 71, 9

91
QUADRILATERIAL

ML - 11
QUADRILATERL
A quadrilateral is a closed figure obtained by joining four points (with no three points collinear) In an
order.
(I) Since, ‘quad’ means ‘four’ and ‘lateral’ is for ‘sides’ therefore ‘quadrilateral’ means ‘a figure bounded
by four sides’.
(II) Every quadrilateral has :
(A) Four vertices,
(B) Four sides
(C) Four angles and
(D) Two diagonals.
(III) A diagonals is a line segment obtained on joining the opposite vertices.

(a) Sum of the Angles of a Quadrilateral :


Consider a quadrilateral ABCD as shown alongside. Join A and C to get the diagonal AC which divides the
quadrilateral ABCD into two triangles ABC and ADC.
We know the sum of the angles of each triangle is 180 0( 2 right angles).
∴ In ∆ABC ; ∠CAB + ∠B + ∠BCA = 1800 and
In ∆ADC ; ∠DAC + ∠D + ∠DCA = 180 0
On adding, we get : (∠CAB + ∠DAC) + ∠B + ∠D + (∠BCA + ∠DCA) = 1800 + 1800
⇒ ∠A + ∠B + ∠D + ∠C = 3600
Thus, the sum of the angles of a quadrilateral is 360 0 (4-right angles).
Ex.1 The angles of a quadrilateral are in the ratio 3 : 5 : 9 : 13. Find all the angles of the quadrilateral.
Sol. Given the ratio between the angles of the quadrilateral = 3 : 5 : 9 : 13 and 3 + 5 + 9 + 13 = 30
Since, the sum of the angles of the quadrilateral = 3600
3
∴ First angle of it = × 3600 = 360,
30
5
Second angle = × 360 0 = 60 0,
30
9
Third angle = × 360 0 = 1080,
30
13
And, Fourth angle = × 3600 = 156 0
30
∴ The angles of quadrilateral are 3600, 600, 1080 and 1560.

92
ALTERNATE SOLUTION :
Let the angles be 3x, 5x, 9x and 13.
∴ 3x + 5x + 9x + 13x = 360 0
3600
⇒ 30x = 3600 and x = = 120
30
∴ 1 st angle = 3x = 2 × 12 0 = 3600
2 nd angle = 5x = × 120 = 600
3 rd angle = 9x = 9 × 12 0 = 1080
And, 4th angle = 13 × 12 0 = 1560.
Ex.2 Use the informations given in adjoining figure to calculate the value of x.
Sol. Since, EAB is a straight line.
∴ ∠DAE + ∠DAB = 1800
⇒ 730 + ∠DAB = 1800
i.e., ∠DAB = 1800 - 730 = 107 0
Since, the sum of the angles of quadrilateral ABCD is 3600
∴ 107 0 + 1050 + x + 80 0 = 3600
⇒ 292 0 + x = 360 0
⇒ x = 3600 - 292 0
⇒ x = 680 Ans.
(b) Types of Quadrilaterals :
(i) Trapezium : It is a quadrilateral in which one pair of opposite sides are parallel. In the quadrilateral
ABCD, drawn alongside, sides AB and DC are parallel, therefore it is a trapezium.

(ii) Parallelogram : It is a quadrilateral in which both the pairs of opposite sides are parallel. The adjoining
figure shows a quadrilateral ABCD in which AB is parallel to DC and AD is parallel to BC, therefore ABCD
is a parallelogram.

(iii) Rectangle : it is a quadrilateral whose each angle is 900


(A) ∠A + ∠B = 900 + 900 = 180 0 ⇒ AD ║ BC
(B) ∠B + ∠C = 900 + 900 = 1800 ⇒ AB ║ DC

Rectangle ABCD is a parallelogram Also.

93
(iv) Rhombus : It is a quadrilateral whose all the sides are equal. The adjoining figure shows a quadrilateral
ABCD in which AB = BC = CD = DA ; therefore it is a rhombus.

(v) Square : It is a quadrilateral whose all the sides are equal and each angle is 90 0. The adjoining figure
shows a quadrilateral ABCD in which AB = BC = CD = DA and ∠A = ∠B = ∠C = ∠D = 900, therefore ABCD
is a square.

(vi) Kite : It is a quadrilateral in which two pairs of adjacent sides are equal. The adjoining figure shows a
quadrilateral ABCD in which adjacent sides AB and AD are equal i.e., AB = AD and also the other pair of
adjacent sides are equal i.e., BC = CD; therefore it is a kite or kite shaped figure.

REMARK :
(i) Square, rectangle and rhombus are all parallelograms.
(ii) Kite and trapezium are not parallelograms.
(iii) A square is a rectangle.
(iv) A square is a rhombus.
(v) A parallelogram is a trapezium.

PARALLELOGRAM
A parallelogram is a quadrilateral in which both the pairs of opposite sides are parallel.
Theorem 1 : A diagonal of a parallelogram divides the parallelogram into two congruent triangles.
Given : A parallelogram ABCD.
To Prove : A diagonal divides the parallelogram into two congruent triangles
i.e., if diagonal AC is drawn then ∆ABC ≅ ∆CDA
and if diagonal BD is drawn then ∆ABD ≅ ∆CDB
Construction : Join A and C
Proof : Sine, ABCD is a parallelogram
AB ║ DC and AD ║ BC

94
In ∆ABC and ∆CDA
∠BAC = ∠DCA [Alternate angles]
∠BCA = ∠DAC [Alternate angles]
And, AC = AC [Common side]
∴ ∆ABC ≅ ∆CDA [By ASA]
Similarly, we can prove that
∆ABD ≅ ∆CDB
Theorem 2 : In a parallelogram, opposite sides are equal.
Given : A parallelogram ABCD in which AB║DC and AD║BC.

To Prove : Opposite sides are equal i.e., AB = DC and AD = BC


Construction : Join A and C
Proof : In ∆ABC and ∆CDA
∠BAC = ∠DCA [Alternate angles]
∠BCA = ∠DAC [Alternate angles]
AC = AC [Common]
∴ ∆ABC ≅ ∆CDA [By ASA]
⇒ AB = DC and AD = BC [By cpctc] Hence Proved.
Theorem 3: If each pair of opposite sides of a quadrilateral is equal, then it is a parallelogram
Given : A quadrilateral ABCD in which

To Prove: ABCD is a parallelogram i.e., AB ║ DC and AD ║ BC


Construction : Join A and C
Proof : In ∆ABC and ∆CDA
AB = DC [Given]
AD = BC [Given]
And AC = AC [Common]
∴ ∆ABC ≅ ∆CDA [By SSS]
⇒ ∠1 = ∠3 [By cpctc]
And ∠2 =∠4 [By cpctc]
But these are alternate angles and whenever alternate angles are equal, the lines are parallel.
∴ AB ║ DC and AD ║ BC
⇒ ABCD is a parallelogram. Hence Proved.

95
Theorem 4 : In a parallelogram, opposite angles are equal.
Given : A parallelogram ABCD in which AB ║ DC and AD ║ BC.
To Prove : Opposite angles are equal
i.e. ∠A = ∠C and ∠B = ∠D
Construction : Draw diagonal AC
Proof : In ∆ABC and ∆CDA :
∠BAC = ∠DCA [Alternate angles]
∠BCA = ∠DAC [Alternate angles]
AC = AC [Common]
∴ ∆ABC ≅ ∆CDA [By ASA]
⇒ ∠B = ∠D [By cpctc]
And, ∠BAD = ∠DCB i.e., ∠A = ∠C
Similarly, we can prove that ∠B = ∠D Hence Proved.
Theorem 5: If in a quadrilateral, each pair of opposite angles is equal, then it is a parallelogram.
Given : A quadrilateral ABCD in which opposite angles are equal.
i.e., ∠A = ∠C ad ∠B = ∠D
To prove : ABCD is a parallelogram i.e, AB ║ DC and AD ║ BC.
Proof : Since, the sum of the angles of quadrilateral is 3600
⇒ ∠A + ∠B + ∠C + ∠D = 3600
⇒ ∠A + ∠D + ∠A + ∠D = 360.. [∠A = ∠C and ∠B = ∠D]
0
⇒ 2 ∠A = 2 ∠D = 360
⇒ ∠A + ∠D = 1800 [Co-interior angle]
⇒ AB ║ DC
Similarly,
∠A + ∠B + ∠C + ∠D = 3600
⇒ ∠A + ∠B + ∠A + ∠B = 360 0 [∠A = ∠C and ∠B = ∠D]
0
⇒ 2 ∠A + 2 ∠V = 360
⇒ ∠A + ∠B = 180 0 [ ∵ This is sum of interior angles on the same side of transversal AB]
∴ AD ║ BC
So, AB ║ DC and AD ║ BC
⇒ ABCD is a parallelogram. Hence Proved.
Theorem 6 : The diagonal of a parallelogram bisect each other.
Given : A parallelogram ABCD. Its diagonals AC and BD intersect each other at point O.
To Prove : Diagonals AC and BD bisect each other i.e., OA = OC and OB = OD.
Proof : In ∆AOB and ∆COD
∵ AB ║ DC and BD is a transversal line.
∴ ∠ABO = ∠DCO [Alternate angles]
∴ AB ║ DC and AC is a transversal line.
∴ ∠BAO = ∠DCO [Alternate angles]
And, AB = DC
⇒ ∆AOB ≅ ∆COD [By ASA]
⇒ OA = OC and OB = OD [By cpctc] Hence Proved.

96
Theorem 7 : If the diagonals of a quadrilateral bisect each other, then it is a parallelogram.
Given : A quadrilateral ABCD whose diagonals AC and BD bisect each other at point O.
i.e., OA = OC and OB = OD
To prove : ABCD is a parallelogram
i.e., AB ║ DC and AD ║ BC.
Proof : In ∆AOB and ∆COD
OA = OC [Given]
OB = OD [Given}
And, ∠AOB = ∠COD [Vertically opposite angles]
⇒ ∆AOB ≅ ∆COD [By SAS]
⇒ ∠1 = ∠2 [By cpctc]
But these are alternate angles and whenever alternate angles are equal, the lines are parallel.
∴ AB is parallel to DC i.e., AB ║ DC
Similarly,
∆AOD ≅ ∆COB [By SAS]
⇒ ∠3 = ∠4
But these are also alternate angles ⇒ AD ║ BC
AB ║ DC and AD ║ BC ⇒ ABCD is parallelogram. Hence Proved.
Theorem 8 : A quadrilateral is a parallelogram, if a pair of opposite sides is equal and parallel.
Given : A quadrilateral ABCD in which AB ║ DC and AB = DC.
To Prove : ABCD is a parallelogram
i.e., AB ║ DC and AD ║ BC.
Construction : Join A and C.
Proof : Since AB is parallel to DC and AC is transversal
∠BAC = ∠DCA [Alternate angles]
AB = DC [Given]
And AC = AC [Common side]
⇒ ∆BAC ≅ ∆DCA [By SAS]
⇒ ∠BCA = ∠DAC [By cpctc]
But these are alternate angles and whenever alternate angles are equal, the lines are parallel.
⇒ AD ║ BC
Now, AB ║ DC (given) and AD ║ BC [Proved above]
⇒ ABCD is a parallelogram Hence Proved.
REMARKS :
In order to prove that given quadrilateral is parallelogram, we have to prove that :
(i) Opposite angles of the quadrilateral are equal, or
(ii) Diagonals of the quadrilateral bisect each other, or
(iii) A pair of opposite sides is parallel and is of equal length, or
(iv) Opposite sides are equal.
(v) Every diagonal divides the parallelogram into two congruent triangles.

97
EXERCISE

OBJECTIVE DPP # 11.1

1. In a parallelogram ABCD, ∠D = 1050, then the ∠A and ∠B will be.

(A) 1050, 75 0 (B) 750, 1050 (C) 1050, 1050 (D) 75 0, 750

2. In a parallelogram ABCD diagonals AC and BD intersects at O and AC = 12.8 cm and BD = 7.6 cm, then the

measure of OC and OD respectively equal to :

(A) 1.9 cm, 6.4 cm (B) 3.8 cm, 3.2 cm (C) 3.8 cm, 3.2 cm (D) 6.4 cm, 3.8 cm

3. Two opposite angles of a parallelogram are (3x - 2)0 and (50 - x)0 then the value of x will be :

(A) 170 (B) 160 (C) 15 0 (D) 13 0

4. When the diagonals of a parallelogram are perpendicular to each other then it is called.

(A) Square (B) Rectangle (C) Rhombus (D) Parallelogram

5. In a parallelogram ABCD, E is the mid-point of side BC. If DE and AB when produced meet at F then :

1
(A) AF = AB (B)AF = 2AB (C) AF = 4AB (D) Data Insufficient
2

6. ABCD is a rhombus with ∠ABC = 560, then the ∠ACD will be.

(A) 560 (B) 620 (C) 1240 (D) 34 0

7. In a triangle, P,Q, and R are the mid-points of the sides BC, CA and AB respectively. If AC = 16 cm, BC = 20

cm and AB = 24 cm then the perimeter of the quadrilateral ARPQ will be.

(A) 60 cm (B) 30 cm (C) 40 cm (D) None

8. LMNO is a trapezium with LM ║ NO. If P and Q are the mid-points of LO and MN respectively and LM =

5 cm and ON = 10 cm then PQ =

(A) 2.5 m (B) 5 cm (C) 7.5 cm (D) 15 cm

9. In a Isosceles trapezium ABCD if ∠A = 450 then ∠C will be.

(A) 900 (B) 135 0 (C) 90 0 (D) None

10. In a right angle triangle ABC is right angled at B. Given that AB = 9 cm, AC = 15 cm and D, E are the mid-

points of the sides AB and AC respectively, then the area of ∆ADE=

(A) 67.5 cm2 (B) 13.5 cm2 (C) 27 cm2 (D) Data insufficient

98
SUBJECTIVE DPP - 11.2

1. Find the measures of all the angles of a parallelogram , if one angle is 240 less than twice the smallest angle.

2. In the following figure, ABCD is a parallelogram in which ∠DAB = 750 and ∠DBC = 600. Find ∠COB and
∠ADB.

3. In the following figure, ABCD is a parallelogram ∠DAO = 400, ∠BAO = 350 and ∠COD = 650. Find
(i) ∠ABO

(ii) ∠ODC
(iii) ∠ACB
(iv) ∠CBD

4. In the following figure, ABCD is a parallelogram in which ∠A = 650. Find ∠B, ∠C and ∠D.

5. In the following figure, ABCD is a parallelogram in which ∠A = 600. If the bisectors of ∠A and ∠B meet at
P, prove that ∠APB = 900. Also, prove that AD = DP, PC = BC and DC = 2AD.

99
QUADRILATERAL

ML - 12
MID-POINT THEOREM
Statement : In a triangle, the line segment joining the mid-points of any two sides is parallel to the third
side and is half of it.

Given : A triangle ABC is which P is the mid-point of side AB and Q is the mid-point of side AC.
1
To Prove : P is parallel to BC and is half of it i.e., PQ ║ BC and PQ = BC
2
Construction : Produce PQ upto point R such that PQ = QR. Join T and C.
Proof : In ∆APQ and ∆CRQ : -
PQ = QR [By construction]
AQ = QC [Given]
And, ∠AQP = ∠CQR [Vertically opposite angles]
⇒ ∆APQ ≅ ∆CRQ [By SAS]
⇒ AP = CR [By cpctc]
And, ∠APQ = ∠CRQ [By cpctc]
But, ∠APQ and ∠CRQ are alternate angles and we know, whenever the alternate angles are equal, the lines
are parallel.
⇒ AP ║ CR
⇒ AB ║ CR
⇒ BP ║ CR
Given, P is mid-point of AB
⇒ AP = BP
⇒ CR = BP [As, AP = CR]
Now, BP = CR and BP ║ CR
⇒ BCRP is a parallelogram.
[When any pair of opposite sides are equal and parallel, the quadrilateral is a parallelogram]
BCRP is a parallelogram and opposite sides of a parallelogram are equal and parallel.
∴ PR = BC and PR ║ BC
Since, PQ = QR
1
⇒ PQ = PR
2

100
1
= BC [As, PR = BC]
2
Also, PQ ║ BC [As, PR ║ BC]
1
∴ PQ ║ BC and P + BC Hence Proved.
2
ALTERNATIVE METHOD :
Construction : Draw CR parallel to BA intersecting PQ produced at point R.
Proof : In ∆APQ and ∆CRQ
AQ = CQ [Given]
∠AQP = ∠RQC [Vertically opposite angles]
And ∠PAQ = ∠RCQ [Alternate angles, as AB║CR]
∆APQ ≅ ∆CRQ [By ASA]
⇒ CR = AP and QR = PQ [By cpctc]
Since, CR = AP and AP = PB
⇒ CR = PB
Also, CR ║ PB [By construction]
∴ PBCR is a parallelogram [As, opposite sides PB and CR are equal and parallel]
⇒ BC ║ PR and BC = PR
⇒ BC ║ PQ and BC = 2PQ [ ∵ PQ = QR]
1
⇒ PQ ║ BC and PQ = BC Hence Proved.
2
(a) Converse of the Mid-Point Theorem
Statement : The line drawn through the mid-point of one side of a triangle parallel to the another side
bisects the third side.

Given : A triangle ABC in which P is the mid-point of side AB nd PQ is parallel to BC.


To prove: PQ bisects the third side AB i.e., AQ = QC.
Construction : Through C, draw CR parallel to BA, which meets PQ produced at point R.
Proof : Since, PQ ║ BC i.e., PR ║ BC [Given] and CR ║ BA i.e., CR ║ BP [By construction]
∴ Opposite sides of quadrilateral PBCR are parallel.
⇒ PBCR is a parallelogram
⇒ BP = CR
Also, BP = AP [As, P is mid-point of AB]
∴ CR = AP
∴ AB ║ CR and AC is transversal, ∠PAQ = ∠RCQ [Alternate angles]
∴ AB ║ CR and PR is transversal, ∠APQ = ∠CRQ [Alternate angles]
In ∆APQ and ∆CRQ
CR = AP, ∠PAQ = ∠RCQ and ∠APQ = ∠CRQ
⇒ ∆APQ ≅ ∆CRQ [By ASA]
⇒ AAQ = QC Hence Proved.

101
Ex.1 ABCD is a rhombus and P, Q, R and S are the mid-points of the sides AB, BC, CD and DA respectively.
Prove that the quadrilateral PQRS is a rectangle.
Sol. According to the given statement, the figure will be a shown alongside; using mid-point theorem :-

1
In ∆ABC, PQ ║ AC and PQ = AC ....(i)
2
1
In ∆ADC, SR ║ AC and SR = AC ....(ii)
2
∴ P = SR and PQ ║ SR [From (i) and (ii)]
⇒ PQRS is a parallelogram.
Now, PQRS will be a rectangle if any angle of the parallelogram PWRS is 90 0
PQ ║ AC [By mid-point theorem]
QR = BD [By mid-point theorem]
But, AC ⊥ BD [Diagonals of a rhombus are perpendicular to each other]
∴ PQ ⊥ QR [Angle between two lines = angles between their parallels]
⇒ PQRS is a rectangle Hence Proved.
Ex.2 ABCD is a trapezium in which AB ║ DC, BD is a diagonal and E is the mid-point of AD. A line is drawn
through E parallel to AB intersecting BC at F (as shown). Prove that F is the mid-point of BC.
Sol. Given line EF is parallel to AB and AB ║ DC
∴ EF ║ AB ║ DC.

According to the converse of the mid-point theorem, is ∆ABD, E is the mid-point of AD.
EP is parallel to AB [As EF ║ AB]
∴ P is mid-point of side BD
[The line through the mid-point of a side of a triangle and parallel to the other side, bisects the third side]
Now, in ∆BCD, P is mid-point of BD [Proved above]
And, PF is parallel to DC [As EF ║ DC]
∴ F is mid-point of BC
[The line through the mid-point of a side of a triangle and parallel to the other side, bisects the third side]
Hence Proved.

102
REMARK :
In quadrilateral ABCD, if side AD is parallel to side BC; ABCD is a trapezium.

1
Now, P and Q are the mid-points of the non-parallel sides of the trapezium; then PQ= (AD + BC). i.e. The
2
length of the line segment joining the mid-points of the two non-parallel sides of a trapezium is always
equal to half of the sum of the lengths of its two parallel sides.
Theorem.3: If there are three or more parallel lines and the intercepts made by them on a transversal are
equal, then the corresponding intercepts on any other transversal are also equal.
Given : Three parallel lines I, m and n i.e., I ║ m ║ n. A transversal p meets these parallel lines are points A,
B and C respectively such that AB = BC. Another transversal q also meets parallel lines I, m and n at points
D, E and F respectively.

To Prove : DE = EF
Construction : Through point A, draw a line parallel to DEF; which meets BE at point P and CF and point
Q.
Proof : In ∆ACQ, B is mid-point of AC and BP is parallel to CQ and we know that the line through the mid-
point of one side of the triangle and parallel to another sides bisects the third side.
∴ AP = PQ ...(i)
When the opposite sides of a quadrilateral are parallel, it is a parallelogram and so its opposite sides are
equal.
∴ AP ║ DE and AD ║ PE ⇒ APED is a parallelogram.
⇒ AP = DE ....(ii)
And PQ ║ EF and PE ║ QF ⇒ PQFE is a parallelogram
⇒ PQ = EF ....(iii)
From above equations, we get
DE = EF Hence Proved.

103
Ex.3 In the given figure, E and F are respectively, the mid-points of non-parallel sides of a trapezium ABCD.
Prove that
(i) EF ║ AB
1
(ii) EF = (AB + DC).
2
Sol. Join BE and produce it to intersect CD produced at point P. In ∆AEB and ∆DEP, AB ║ PC and BP is
transversal
⇒ ∠ABE = ∠DPE [Alternate interior angles]
∠AEB = ∠DEP [Vertically opposite angles]
And AE = DE [E is mid - point of AD]
⇒ ∆AEB ≅ ∆DEP [By ASA]
⇒ BE = PE [By cpctc]
And AB = DP [By cpctc]
Since, the line joining the mind-points of any two sides of a triangle is parallel and half of the third side,
therefore, is ∆BPC,
E is mid-point of BP [As, BE = PE]
and F is mid-point of BC [Given]
1
⇒ EF ║ PC and EF = PC
2
1
⇒ EF ║ DC and EF = (PD + DC)
2
1
⇒ EF ║ AB and EF = (AB + DC) [As, DC ║ AB and PD = AB] Hence Proved.
2

104
EXERCISE

OBJECTIVE DPP # 12.1

1. When the opposite sides of quadrilateral are parallel to each other then it is called.
(A) Square (B) Parallelogram (C) Trapezium (D) Rhombus
2. In a ∆ABC, D, E and F are respectively, the mid-points of BC, CA and AB. If the lengths of side AB, BC and
CA are 17 cm, 18 cm and 19 cm respectively, then the perimeter of ∆DEF equal to :
(A) 54 cm (B) 18 cm (C) 27 cm (D) 13.5 cm
3. When only one pair of opposite sides of a quadrilateral parallel to each other it is called.
(A) Square (B) Rhombus (C) Parallelogram (D) Trapezium
4. When the diagonals of a parallelogram are equal but not perpendicular to each other it is called a.
(A) Square (B) Rectangle (C) Rhombus (D) Parallelogram
5. When each angle of a rhombus equal to 90.0, it is called a.
(A) Square (B) Rectangle (C) Trapezium (D) Parallelogram
6. In the adjoining figure, AP and BP are angle bisectors of ∠A and ∠B which meets at P on the parallelogram
ABCD. Then 2∠APB =

(A) ∠C + ∠D (B) ∠A + ∠C (C) ∠B + ∠D (D) 2∠C


7. In a quadrilateral ABCD, AO & DO are angle bisectors of ∠A and ∠D and given that ∠C = 1050, ∠B = 700
then the ∠AOD is :
(A) 67.5 0 (B) 77.50 (C) 87.50 (D) 99.75 0
8. In a parallelogram the sum of the angle bisectors of two adjacent angle is :
(A) 300 (B) 450 (C) 600 (D) 900
9. In the adjoining parallelogram ABCD, the angles x and y are :

(A) 600, 300 (B) 300, 60 0 (C) 450, 45 0 (D) 900, 90 0

105
10. From the figure find the value of ∠SQP and ∠QSP of parallelogram PQRS.

(A) 600, 500 (B) 600, 45 0 (C) 700, 35 0 (D) 350, 70 0

SUBJECTIVE DPP 12.2

1. Prove that the line joining the mid-points of the diagonals of a trapezium is parallel to each to the parallel
sides and is equal to half of the difference of these sides.
1 1
2. ABCD is a parallelogram. P is a point on AD such that AP = AD. Q is a point on BC such that CQ = BC.
3 3
Prove that AQCP is a parallelogram.
3. In the following figure, AD is a median and DE ║ AB. Prove that BE is a median.

4. Prove that “If a diagonal of a parallelogram bisects one of the angles of the parallelogram, it also bisects the
second angle and then the two diagonals are perpendicular to each other.
5. Prove that the figure formed by joining the mid-points of the consecutive sides of a quadrilateral is a
parallelogram.
6. In a parallelogram ABCD, the bisector of ∠A also bisects BC at P. Prove that AD = 2AB.
7. The diagonals of parallelogram ABCD intersect at O. A line through O intersects AB at X and DC at Y.
Prove that OX = OY.
8. Show that the quadrilateral formed by joining the mid points of the sides of square is also a square.
9. ABCD is a trapezium in which side AB is parallel to side DC and E is the mid-point of side AD. If F is a
1
point on side BC such that segment EF is parallel to side DC. Prove that EF = (AB + DC).
2
10. In ∆ABC, AD is the median through A and E is the mid-point of AD. BE produced meets AC in F. Prove
1
that AF= AC.
3

106
ANSWER KEY

(Objective DPP # 11.1)

Qus. 1 2 3 4 5 6 7 8 9 10

Ans. B D D C B B C C B C

(Subjective DPP # 11.2)


0 0 0 0
1. 68 , 12 , 68 , 112 2. 45 0 & 600

3. (i) 800 (ii) 800 (iii) 40 0 (iv) 250

4. 1150, 650 and 1150

(Objective DPP # 12.1)

Qus. 1 2 3 4 5 6 7 8 9 10

Ans. B C D B A A C D A A

107
AREA OF PARALLELOGRAMS
AND TRIANGLE

ML - 13
POLYGONAL REGION
Polygon region can be expressed as the union of a finite number of triangular regions in a plane such that if
two of these intersect, their intersection is either a point or a line segment. It is the shaded portion including
its sides as shown in the figure.

(a) Area Axioms :


Every polygonal region R has an area, measure in square unit and denoted by ar(R).
(i) Congruent area axiom : if R1 and R2 be two regions such that R1 ≅ R2 then ar(R1) = ar (R2).
(ii) Area monotone axiom : If R1 ⊂ R2, then are (R1) ≤ ar(R2).
(iii) Area addition axiom : If R1 are two polygonal regions, whose intersection is a finite number of points
and line segments and R = R1 ∪ R2, then ar (R) = ar(R1) + ar(R2).
(iv) Rectangular area axiom : If AB = a metre and AD = b metre then,
ar (Rectangular region ABCD) = ab sq. m.
(b) Unit of Area :
There is a standard square region of side 1 metre, called a square metre, which is the unit of area measure.
The area of a polygonal region is square metres (sq. m or m2) is a positive real number
AREA OF A PARALLELOGRAM
(a) Base and Altitude of a Parallelogram :
(i) Base : Any side of parallelogram can be called its base.
(ii) Altitude : The length of the line segment which is perpendicular to the base from the opposite side is
called the altitude or height of the parallelogram corresponding to the given base.

108
In the Adjoining Figure
gm
(i) DL is the altitude of ABCD, corresponding to the base AB.

gm
(ii) DM is the altitude of ABCD, corresponding to the base BC.

Theorem -1 A diagonal of parallelogram divides it into two triangles of equal area.


Given : A parallelogram ABCD whose one of the diagonals is BD.
To prove : ar (∆ABD) = ar (∆CDB).
Proof : In ∆ABD and ∆CDB.
gm
AB = DC [Opp. sides of a ]

gm
AD = BC [Opp. sides of a ]

BD = BD [Common side]
∴ ∆ABD ≅ ∆CDB [By SSS]
∴ ar (∆ABD) = ar(∆CDB) [Congruent area axiom] Hence Proved.
Theorem -2: Parallelograms on the same base or equal base and between the same parallels are equal in
area.

gm
Given : Two ABCD and ABEF on the same base AB and between the same parallels AB and FC.

gm gm
To Prove : ar( ABCD) = ar( ABEF)

Proof : In ∆ADF and ∆BCE, we have


gm
AD = BC [Opposite sides of a ]

gm
AF = BE [Opposite sides of a ]

∠DAF = ∠CBE [ ∵ AD ║ BC and AF ║ BE]


[Angle between AD and AF = angle between BC and BE]
∴ ∆ADF ≅ ∆BCE [By SAS]
∴ ar(∆ADF) = ar(∆BCE) ....(i)
gm
∴ ar( ABCD) = ar(ABED) + ar(∆BCE)

= ar(ABED) + ar(∆ADF) [Using (i)]


gm
= ar( ABEF).

gm gm
Hence, ar( ABCD) = ar( ABEF). Hence Proved.

NOTE : A rectangle is also parallelogram.


Theorem -3: The are of parallelogram is the product of its base and the corresponding altitude.

109
gm
Given : A ABCD in which AB is the base and AL is the corresponding height.
gm
To prove : Area ( ABCD) = AB × AL.
Construction : Draw BM ⊥ DC so that rectangle ABML is formed.
gm
Proof : ABCD and rectangle ABML are on the same base AB and between the same parallel lines AB
and LC.
gm
∴ ar( ABCD) = ar(rectangle ABML) = AB × AL.
gm
∴ area of a = base × height. Hence Proved.
Theorem-4 : Parallelograms on equal bases and between the same parallels are equal in area.

gm
Given : Two ABCD and PQRS with equal base AB and PQ and between the same parallels, AQ and
DR.
gm gm
To prove: ar( ABCD) = ar( PQRS).
Construction : Draw AL ⊥ DR and PM ⊥ DR.
Proof : AB ║ DR, AL ⊥ DR and PM ⊥ Dr
∴ AL = PM.
gm
∴ ar( ABCD) = AB × AL
= PQ × PM [ ∵ AB = PQ and AL = PM]
gm
= a( PQRS). Hence Proved.
ILLUSTRATIONS :
Ex.1 In a parallelogram ABCD, AB = 8 cm. The altitudes corresponding to sides AB and AD are respectively 4 m
and 5 cm. Find AD.
Sol. We know that, Area of a parallelogram = Base × Corresponding altitude

∴ Area of parallelogram ABCD = AD × BN = AB × DM


⇒ AD × 5 = 8 × 4
8× 4
⇒ AD =
5
= 6.4 cm. Ans.
Ex.2 In figure, ABCD is a parallelogram, AE ⊥ DC and CF ⊥ AD. If AB = 16 cm, AE = 8 cm and CF = 10 cm find
AD.

110
Sol. We have AB = 16 cm, AE = 8 cm CF = 10 cm.
We know that are of parallelogram = Base × Height [Base = CD, height = AE]
2
ABCD = CD × AE = 16 × 8 = 128 cm
Again, Area of parallelogram = Base × Height = AD × CF [Base = AD, height = CF]
128 = AD × 10
128
⇒ AD = =12.8 cm Ans.
10
Ex.3 ABCD is a quadrilateral and BD is one of its diagonal as shown in the figure. Show that the quadrilateral
ABCD is a parallelogram and find its area.
Sol. From figure, the transversal DB is intersecting a pair of lines DC and AB such that
∠CDB = ∠ABD = 90 0.
Hence these angles from a pair of alternate equal angles.
∴ DC ║ AB.
Also DC = AB = 2.5 units.
∴ Quadrilateral ABCD is a parallelogram.
Now, area of parallelogram ABCD
= Base × Corresponding altitude
= 2.5 × 4
= 10 sq. units Ans.
Ex.4 The diagonals of a parallelogram ABCD intersect in O. A line through O meets AB is X and the opposite
1
side CD in Y. Show that ar (quadrilateral AXYD) = far(parallelogram ABCD).
2
Sol. ∴ AC is a diagonal of the parallelogram ABCD.
1
ar(∆ACD) = ar(ABCD) ...(i)
2
Now, in ∆s AOX and COY,
AO = CO
∵ Diagonals of parallelogram bisect each other.
∠AOX = ∠COY [Vert. opp. ∠s]
∠OAX = ∠OCY [Alt. Int. ∠s]
∴ AB ║ DC and transversal AC intersects them
∴ ∆AOX ≅ ∆COY [ASA]
∴ ar(∆AOX) = ar(∆COY) ....(ii)
Adding ar(quad. AOYD) to both sides of (ii), we get
ar(quad. AOYD) + ar(∆AOX) = ar(quad. AOYD) + ar(∆COY)
1
⇒ ar(quad. AXYD) = ar(∆ACD) = ar(║gm ABCD) (using (i)) Hence Proved.
2
AREA OF A TRIANGLE
Theorem-5 : Two triangles on the same base (or equal bases) and between the same parallels are equal in
area.
Given : Two triangles ABC and PCs on the same base BC and between the same parallel lines BC and AP.
To prove : ar(∆ABC) = ar(∆PBC)

111
Construction : Through B, draw BD ║ CA intersecting PA produced in D and through C, draw CQ ║ BP,
intersecting line AP in Q.
Proof : We have,
BD ║ CA [By construction]
And, BC ║ DA [Given]
∴ Quad. BCAD is a parallelogram.
Similarly, Quad. BCQP is a parallelogram.
Now, parallelogram BCQP and BCAD are on the same base BC, and between the same parallels.
gm gm
∴ ar( BCQP) = ar( BCAD) ....(i)

We know that the diagonals of a parallelogram divides it into two triangles of equal area.
1 gm
∴ ar(∆PBC= ar( BCQP) ....(ii)
2
1 gm
And ar(∆ABC) = ar( BCAD) ....(iii)
2
gm gm
Now, ar( BCQP) = ar( BCAD) [From (i)]

1 gm 1 gm
⇒ ar( BCAD) = ar( BCQP)
2 2
Hence, ar(∆ABC) = ar(∆PBC) [Using (ii) and (iii)]
Hence Proved.
Theorem-6 : The area of a trapezium is half the product of its height and the sum of the parallel sides.

Given : Trapezium ABCD in which AB ║ DC, AL ⊥ DC, CN ⊥ AB and AL = CN = h (say)


AB = a, DC = b.
1
To prove : ar(trap. ABCD) = h × (a + b).
2
Construction : Join AC.
Proof : AC is a diagonal of quad. ABCD.
1 1 1
∴ ar(trap. ABCD) = ar(∆ABC) + ar(∆ACD) = h × a + h × b= h(a + b). Hence Proved.
2 2 2
Theorem -7: Triangles having equal areas and having one side of the triangle equal to corresponding
side of the other, have their corresponding altitudes equal/
Given : Two triangles ABC and PQR such that (i) ar (∆ABC) = ar(∆PQR) and (ii) AB = PQ.
CN and RT and the altitude corresponding to AB and PQ respectively of the two triangles.

112
To prove : CR = RT.
Proof : In ∆ABC, CN is the altitude corresponding to the side AB.
1
ar(∆ABC) = AB × CN ....(i)
2

1
Similarly, ar(∆PQR) = PQ × RT ....(ii)
2
Since ar(∆ABC) =ar(∆PQR) [Given]
1 1
∴ AB × CN = PQ × RT
2 2
Also, AB = PQ [Given]
CN = RT Hence Proved.
Ex.5 In figure, E is any point on median AD of a ∆ABC. Show that ar(ABE) = ar(ACE).

Sol. Construction : From A draw AG ⊥ BC and from E draw EF ⊥ BC.


BD × AG
Proof : ar(∆ABD) =
2
DC × G
ar(∆ADC) =
2
But, BD = DC [ ∴ D is the mid-point of BC, AD being the median]
ar(∆ABD) = ar(∆ADC) ...(i)
BD × EF
Again, ar(∆EBD)=
2
DC × EF
ar(∆EDC) =
2
But, BD = DC
∴ ar(∆EBD) = ar(∆EDC) ...(ii)
Subtracting (ii) from (i), we get
ar(∆ABD) - ar(∆EBD) = ar(∆ADC) - ar(∆EDC)
⇒ ar(∆ABE) = ar(∆ACE). Hence Proved.
Ex.6 Triangles ABC and DBC are on the same base BC; with A, D on opposite sides of the line BC, such that
ar(∆ABC) = ar(∆DBC). Show that BC bisects AD.

113
Sol. Construction : Draw AL ⊥ BC and DM ⊥ BC.
Proof : ar(∆ABC) = ar(∆DBC) [Given]
BC × AL BC × DM
⇒ =
2 2
⇒ AL = DM ....(i)
Now in ∆s OAL and OMD
AL = DM [From (i)]
⇒ ∠ALO = ∠DMO [Each = 900]
⇒ ∠AOL = ∠MOD [Vert. opp. ∠s]
⇒ ∠OAL = ∠ODM [Third angles of the triangles]
∴ ∆OAL ≅ ∆OMD [By ASA]
∴ OA = OD [By cpctc]
i.e., BC bisects AD. Hence Proved.
Ex.7 ABC is a triangle in which D is the mid-point of BC and E is the mid-point of AD. Prove that the area of
1
∆BED= area of ∆ABC.
4
Sol. Given : A ∆ABC in which D is the mid-point of BC and E is the mid-point of AD.
1
To prove: ar(∆BED) = ar(∆ABC).
4

Proof : ∵ AD is a median of ∆ABC.


1
∴ ar(∆ABD) = ar(∆ADC) = ar(∆ABC) .....(i)
2
1
[ ∴ Median of a triangle divides it into two triangles of equal area) = ar(∆ABC)
2
Again,
∵ BE is a median of ∆ABD,
1
∴ ar(∆BEA) = ar(∆BED) = ar(∆ABD)
2
[ ∴ Median of a triangle divides it into two triangles of equal area]
1 1 1
And ar(∆ABD) = × ar(∆ABC) [From (i)]
2 2 2
1
∴ ar(∆BED) = ar(∆ABC). Hence Proved.
4

114
1
Ex.8 if the medians of a ∆ABC intersect at G, show that ar(∆AGB) = ar(∆BGC) = ar(∆ABC).
3
Sol. Given : A ∆ABC its medians AD, BE and CF intersect at G.
1
To prove : ar(∆AGB) = ar(∆AGC) = ar(∆BGC) = ar(∆ABC).
3
Proof : A median of triangle divides it into two triangles of equal area.
In ∆ABC, AD is the median.
∴ ar(∆ABD) = ar(∆ACD) ...(i)
In ∆GBC, GD is the median.
∴ ar(∆GBD) = ar(∆GCD) ...(ii)
From (i) and (ii), we get
ar(∆ABD) - ar(∆GBD) = ar(∆ACD) -ar(∆GCD)
∴ a(∆AGB) = ar(∆AGC).
Similarly,
ar(∆AGB) =ar(∆AGC) =ar(∆BGC) ....(iii)
But, ar(ABC) = ar(∆AGB) + ar(∆AGC) + ar(∆BGC)
= 3 ar(∆AGB) [Using (iii)]
1
∴ ar(∆AGB) = ar(∆ABC).
3
1
Hence, ar(∆AGB) = ar(∆AGC) = ar∆(BGC) = ar(∆ABC). Hence proved.
3
Ex.9 D,E and F are respectively the mid points of the sides BC, CA and AB of a ∆ABC. Show that
(i) BDEF is parallelogram
gm 1
(ii) ar( BDEF) = ar(∆ABC)
2
1
(iii) ar(∆DEF) = ar(∆ABC)
4
Sol. Given : A ∆ABC in which D,E,F are the mid-point of the side BC, CA and AB respectively.
To prove:
(i) Quadrilateral BDEF is parallelogram.
gm 1
(ii) ar( BDEF) = ar(∆ABC).
2
1
(iii) ar(∆DEF) = ar(∆ABC).
4
Proof:
(i) In ∆ABC,
∴ F is the mid-point of side AB and E is the mid point of side AC.
∴ EF ║ BD
[ ∵ Line joining the mid-points of any two sides of a ∆ is parallel to the third side.]
Similarly,
ED ║ FB.
Hence, BDEF is a parallelogram. Hence Proved.
(ii) Similarly, we can prove that AFDE and FDCE are parallelograms.

115
∴ FD is diagonals of parallelogram BDEF.
∴ ar(∆FBD) = ar(∆DEF) ...(i)
Similarly,
ar(∆FAE) = ar(∆DEF) . ..(ii)
And ar(∆DCE) = ar(∆DEF) ...(iii)
From above equations, we have
ar(∆FBD) = ar(∆FAE) = ar(∆DCE) = ar(∆DEF)
And ar(∆FBD) + ar(∆DCE) + ar(∆DEF) + ar(∆FAE) = ar(∆ABC)
⇒ 2[ar(∆FBD) + ar(∆DEF)] = ar(∆AC) [By using (i), (ii) and (iii)]
⇒ 2[ar(║gmBDEF)] = ar(∆ABC)
1
⇒ ar(║gm BDEF) = ar(ABC)
2

(iii) Since, ∆ABC is divided into four non-overlapping triangles FBD, FAE, DCE and DEF.
∴ ar(∆ABC) = ar(∆FBD) + ar(∆FAE) + ar(∆DCE) + ar(∆DEF)
⇒ ar(∆ABC) = 4 ar(∆DEF) [Using (i), (ii) and (iii)]
1
⇒ ar(∆DEF) = ar(∆ABC). Hence Proved.
2
3 2
Ex.10 Prove that the area of an equilateral triangle is equal to a , where a is the side of the triangle.
4
Sol. Draw AD ⊥ BC
⇒ ∆ABD ≅ ∆ACD [Br R.H.S.]
∴ BD = DC [By cpctc]
∴ BC = a
a
∴ BD = DC =
2
In right angled ∆ABD
2
a a 2 3a 2
AD2 = AB2 - BD2 = a2 -   = a2 - =
2 4 4

3a
⇒ AD =
2
1 1 3a 3a 2
Area of ∆ABC = BC × AD = a = . Hence Proved.
2 2 2 4

Ex.11 In figure, P is a point in the interior of rectangle ABCD. Show that


1
(i) ar(∆APB) + ar(∆PCD) = ar(rect. ABCD)
2
(ii) ar(APD) + ar(PBC) = ar(APB) + ar(PCD)

116
Sol. Given : A rect. ABCD and P is a point inside it. PA, PB, PC and PD have been joined.
To prove :
1
(i) ar(∆APB) + ar(∆PCD) = ar(rect. ABCD)
2
(ii) ar(∆APD) + ar(∆BPC) = ar(∆APB) + ar(∆CPD).
Construction : Draw EPF ║ AB and LPM ║ AD.
Proof : (i) EPF ║ AB and DA cuts them,
∴ ∠DEP = ∠EAB = 900 [Corresponding angles]
∴ PE ⊥ AD.
Similarly, PR ⊥ BC; PL ⊥ AB and PM ⊥ DC.
∴ ar(∆APD) + ar(∆BPC)

1  1  1
=  × AD × PE  + ar × BC × PF  = AD × ( PE + PF) [∴BC = AD]
2  2  2
1 1
= × AD × EF = × AD × AB [∴ EF = AB]
2 2
1
= × (rectangle ABCD).
2
(ii) ar (∆APB) + ar(PCD)

1  1  1
=  × AB × PL  +  × DC × PM  = × AB × (PL + PM ) [∴EF = AB]
2  2  2
1 1
= × AB × LM = × AB × AD [ ∵ LM = AD]
2 2
1
= × ar(rect. ABCD).
2
ar(∆APD) + ar(PBC) = ar(∆APB) + ar(PCD) Hence Proved.
Ex.12 Diagonals AC and BD of a quadrilateral ABCD intersect each other at P. Show that :
ar(APB) + ar(CPD) = ar(APD) × ar(BPC)
Sol. Draw perpendiculars AF and CE on BD.

1  1 
ar(APB) × ar(CPD) =  × PB × AF  ×  × PD × CE  ....(i)
2  2 

1  1 
ar(APD) × ar(BPC) =  × PD × AF  ×  × BP × CE  ....(ii)
2  2 
From above equations, we get
ar(APB) × ar(CPD) = ar(APD) × ar(BPC) Hence Proved.

117
EXERCISE
OBJECTIVE DPP - 13.1
1. The sides BA and DC of the parallelogram ABCD are produced as shown in the figure then
(A) a + x = b + y (B) a + y = b + a
(C) a + b = x + y (D) a - b = x - y

2. The sum of the interior angles of polygon is three times the sum of its exterior angles. Then numbers of
sides in polygon is
(A) 6 (B) 7 (C) 8 (D) 9
3. In the adjoining figure, AP and BP are angle bisector of ∠A and ∠B which meet at a point P of the
parallelogram ABCD. Then 2∠APB =
(A) ∠A + ∠B (B) ∠A + ∠C
(C) ∠B + ∠D (D) ∠C + ∠D
4. In a parallelogram the sum of the angle bisector of two adjacent angles is
(A) 300 (B) 45 0
(C) 600 (D) 900
5. In a parallelogram ABCD ∠D = 600 then the measurement of ∠A
(A) 1200 (B) 650 (C) 90 0 (D) 75 0
6. In the adjoining figure ABCD, the angles x and y are
(A) 600, 300 (B) 30 0, 600
(C) 450, 450 (D) 900 , 90 0
7. From the figure parallelogram PQRS, the values of ∠SQP and ∠QSP are are
(A) 450, 600 (B) 60 0, 450
(C) 700, 350 (D) 350, 70

8. In parallelogram ABCD, AB = 12 cm. The altitudes corresponding to the sides AB and AD are respectively 9
cm and 11 cm. Find AD.
108 108
(A) cm (B) cm
11 10
99 108
(C) cm (D) cm
10 17
9. In ∆ABC, AD is a median and P is a point is AD such that AP : PD = 1 : 2 then the area of ∆ABP =
1 2 1 1
(A) × Area of ∆ABC (B) × Area of ∆ABC (C) × Area of ∆ABC (D) × Area of ∆ABC
2 3 3 6
10. In ∆ABC if D is a point in BC and divides it the ratio 3 : 5 i.e., if BD : DC = 3 : 5 then, ar (∆ADC) : ar(∆ABC)
=?
(A) 3 : 5 (B) 3 : 8 (C) 5 : 8 (D) 8 : 3

118
SUBJECTIVE DPP - 13.2
1. If each diagonal of a quadrilateral separates into two triangles of equal area, then show that the
quadrilateral is a parallelogram.
2. In the adjoining figure, PQRS and PABC are two parallelograms of equal area. Prove that QC ║ BR.

3. In the figure ABCD is rectangle inscribed in a quadrant of a circle of radius 10 cm. If AD = 2 5 cm. Find

the area of the rectangle.

4. P and Q are any two points lying on the sides DC and AD respectively of parallelogram ABCD. Prove that
: ar (∆APB) = ar(∆BQC).
5. In the figure, given alongside, PQRS and ABRS are parallelograms and X is any point on side BR. Prove that
:
(i) ar(PQRS) = ar(ABRS)
1
(ii) ar(AXS) = ar(PQRS)
2

6. Find the area a rhombus, the lengths of whose diagonals are 16 cm and 24 cm respectively.
7. Find the area of trapezium whose parallel sides are 8 cm and 6 cm respectively and the distance between
these sides is 8 cm.
8. (i) Calculate the area of quad. ABCD, given in fig. (i)
(ii) Calculate the area of trap. PQRS, given in fig. (ii).

119
(i) (ii)
9. In figure, ABCD is a trapezium in which AB ║ DC; AB = 7 cm; AD = BC = 5 cm and the distance between
AB and DC is 4 cm.

Find the length of DC and hence, find the area of trap. ABCD.
10. BD is one of the diagonals of quadrilateral ABCD. If AL ⊥ BD and CM ⊥ BD, show that : ar(quadrilateral
1
ABCD) = × BC × (AL + CM).
2

11. In the figure, ABCD is a quadrilateral in which diag. BD = 20 cm. If AL ⊥ BD and CM ⊥ BD, such that :
AL = 10 cm and CM = 5 cm, find the area of quadrilateral ABCD.

12. In fig. ABCD is a trapezium in which AB ║ DC and DC = 40 cm and AB = 60 cm. If X and Y are,
respectively, the mid - points of AD and BC, prove that
(i) XY = 50 cm
(ii) DCYX is a trapezium
9
(iii) Area (trapezium DCYX) = Area (trapezium XYBA)
11
13. Show that a median of a triangle divides it into two triangles of equal area.
14. In the figure, given alongside, D and E are two points on BC such that BD = DE = EC. Prove that : ar(ABD)
= ar(ADE) = ar(AEC)

120
15. In triangle ABC, if a point D divides BC in the ratio 2 : 5, show that : ar(∆ABD) : ar(∆ACD) = 2 : 5.

ANSWER KEY

(Objective DPP # 13.1)

Qus. 1 2 3 4 5 6 7 8 9 10

Ans. C C D D A A A A D C

(Subjective DPP # 13.2)

3. 40 cm2 6. 192 cm2


7. 56 cm2 8. (i) 114 cm2 (ii) 195 cm2
9. 40 cm2 11. 150 cm2

121
CIRCLE

ML - 14
DEFINITIONS
(A) Circle :
The collection of all the points in a plane, which are at a fixed distance from a fixed point in the plane, is
called a circle.
The fixed point is called the centre of the circle and the fixed distance is called the radius of the circle.

In figure, O is the centre and the length OP is the radius of the circle. So the line segment joining the centre
and any point on the circle is called a radius of the circle.
(b) Interior and Exterior of a Circle :
A circle divides the plane on which it lies into three parts. They are
(i) inside the circle (or interior of the circle)
(ii) the circle nd
(iii) outside the circle (or exterior of the circle.)

The circle and its interior make up the circular region.


(c) Chord :
If we take two points P and Q on a circle, then the line segment PQ is called a chord of the circle.

122
(d) Diameter:
The chord which passes through the centre of the circle, is called a diameter of the circle.

A diameter is the longest chord and all diameter have the same length, which is equal to two times the
radius. In figure, AOB is a diameter of circle.
(e) Arc :
A piece of a circle between two points is called an arc. If we look at the pieces of the circle between two
points P and Q in figure, we find that there are two pieces, one longer and the other smaller. The longer one
is called the major arc PQ and the shorter one is called the minor arc PQ. The minor arc PQ is also denoted
by PQ and the major arc PQ by PRQ, where R is some point on the arc between P and Q. Unless otherwise
states, arc PQ or PQ stands for minor arc PQ. When P and Q are ends of a diameter, then both arcs are equal
and each is called a semi circle.

(f) Circumference:
The length of the complete circle is called its circumference.
(g) Segment :
The region between a chord and either of its arcs is called a segment of the circular region or simply a
segment of the circle. There are two types of segments also, which are the major segment and the minor
segment (as in figure).

123
(h) Sector :
The region between an arc and the two radii, joining the centre to the end points of the arc is called a sector.
Like segments, we find that the minor arc corresponds to the minor sector and the major arc corresponds to
the major sector. In figure, the region OPQ in the minor sector and the remaining part of the circular region
is the major sector. When two arcs are equal, then both segments and both sectors become the same and
each is known as a semicircular region.

Theorem-1 : Equal chords of a circle subtend equal angles at the centre.


Given : AB and CD are the two equal chords of a circle with centre O.
To Prove : ∠AOB = ∠COD.
Proof : In ∆AOB and ∆COD,
OA = OC [Radii of a circle]
OB = OD [Radii of a circle]
AB = CD [Given]
∴ ∆AOB ≅ ∆COD [By SSS]
∴ ∠AO B = ∠COD. [By cpctc]
Converse of above Theorem :
In the angles subtended by the chords of a circle at the centre are equal, then the chords are equal.

Given : ∠AOB and ∠POQ are two equal angles subtended by chords AB and PQ of a circle at its centre O.
To Prove : AB = PQ

Proof : In ∆AOB and ∆POQ,


OA = OP [Radii of a circle]
OB = OQ [Radii of a circle]
∠AOB = ∠POQ [Given]

∴ ∆AOB ≅ ∆POQ [By SAS]

124
∴ AB = PQ [By cpctc] Hence Proved.
Theorem-2 : The perpendicular from the centre of a circle to a chord bisects the chord.

Given : A circle with centre O. AB is a chord of this circle. OM ⊥ AB.


To Prove : MA = MB.
Construction : Join OA and OB.
Proof : In right triangles OMA and OMB,
OA = OB [Radii of a circle]
OM = OM [Common]
∠OMA = ∠OMB [900 each]
∴ ∆OMA ≅ ∆OMB [By RHS]
∴ MA = MB [By cpctc] Hence Proved.
Converse of above Theorem :
The line drawn through the centre of a circle to bisect a chord a perpendicular to the chord.
Given : A circle with centre O. AB is a chord of this circle whose mid-point is M.
To Prove : OM ⊥ AB.
Construction : Join OA and OB.
Proof : In ∆OMA and ∆OMB.
MA = MB [Given]
OM = OM [Common]
OA = OB [Radii of a circle]
∴ ∆OMA ≅ ∆OMB [By SSS]
∴ ∠AMO = ∠BMO [By cpctc]
But ∠AMO + ∠BMO = 1800 [Linear pair axiom]
∴ ∠AMO = ∠BMO = 900
⇒ OM ⊥ AB.
Theorem-3 : There is one and only one circle passing through three given non-collinear points.
Proof : Let us take three points A, B and C, which are not on the same line, or in other words, they are not
collinear [as in figure]. Draw perpendicular bisectors of AB and BC say, PQ and RS respectively. Let these
perpendicular bistros intersect at one point O. (Note that PQ and RS will intersect because they are not
parallel) [as in figure].

∴ O lies on the perpendicular bisector PQ of AB.


∴ OA = OB
[ ∵ Every point on the perpendicular bisector of a line segment is equidistant from its end points]

125
Similarly,
∴ O lies on the perpendicular bisector RS of BC.
∴ OB = OC
[ ∵ Every point on the perpendicular bisector of a line segment is equidistant from its end points]
So, OA = OB = OC
i.e., the points A, B and C are at equal distances from the point O.
So, if we draw a circle with centre O and radius OA it will also pass through B and C. This shows that there
is a circle passing through the three points A, B and C. We know that two lines (perpendicular bisectors)
can intersect at only one point, so we can draw only one circle with radius OA. In other words, there is a
unique circle passing through A, B and C. Hence Proved.
REMARK :
If ABC is a triangle, then by above theorem, there is a unique circle passing through the three vertices A, B
and C of the triangle. This circle the circumcircle of the ∆ABC. Its centre and radius are called respectively
the circumcentre and the circumradius of the triangle.
Ex.1 In figure, AB = CB and O is the centre of the circle. Prove that BO bisects ∠ABC.
Sol. Given : In figure, AB = CB and O is the centre of the circle.
To Prove : BO bisects ∠ABC.
Construction : Join OA and OC.

Proof : In ∆OAB and ∆OCB,


OA = OC [Radii of the same circle]
AB = CB [Given]
OB = OB [Common]
∴ ∆OAB ≅ ∆OCB [By SSS]
∴ ∠ABO = ∠CBO [By cpctc]
⇒ BO bisects ∠ABC. Hence Proved.
Ex.2 Two circles with centres A and B intersect at C and D. Prove that ∠ACB = ∠ADB.
Sol. Given : Two circles with centres A and B intersect at C and D.
To Prove : ∠ACB = ∠ADB.
Construction : Join AC, AD, BC, BD and AB.
Proof : In ∆ACB an ∆ADB,
AC = AD [Radii of the same circle]
BC = BD [Radii of the same circle]
AB = AB [Common]
∴ ∆ACB ≅ ∆ADB [By SSS]
∴ ∠ACB = ∠ADB. [By cpctc] Hence Proved.

E.3 In figure, AB ≅ ACand O is the centre of the circle. Prove that OA is the perpendicular bisector of BC.

126
Sol. Given : In figure, AB ≅ AC and O is the centre of the circle.
To Prove : OA is the perpendicular bisector of BC.
Construction : Join OB and OC.
Proof :
∴ AB ≅ AC [Given]
∴ chord AB = chord AC.
[ ∵ If two arcs of a circle are congruent, then their corresponding chords are equal.]
∴ ∠AOB = ∠AOC ....(i) [ ∵ Equal chords of a circle subtend equal angles at the centre]
In ∆OBC and ∆OCD,
∠DOB = ∠DOC [From (1)]
OB = OC [Radii of the same circle]
OD = OD [Common]
∴ ∆OBD ≅ ∆OCD [By SAS]
∴ ∠ODB = ∠ODC ....(ii) [By cpctc]
And BD = CD ...(ii) [By cpctc]
0
But ∠BDC = 180
∴ ∠ODB + ∠ODC = 1800
⇒ ∠ODB + ∠ODB = 1800 [From equation (ii)]
0
⇒ 2 ∠ODB = 180
⇒ ∠ODB = 900
∴ ∠ODB = ∠ODC = 90 0 ....(iv) [From (ii)]
So, by (iii) and (iv), OA is the perpendicular bisector of BC. Hence Proved.
Ex.4 Prove that the line joining the mid-points of the two parallel chords of a circle passes through the centre of
the circle.
Sol. Let AB and CD be two parallel chords of a circle whose centre is O.
Let l and M be the mid-points of the chords AB and CD respectively. Join PL and OM.
Draw OX ║ AB or CD.

∴ L is the mid-point of the chord AB and O is the centre of the circle


∴ ∠OLB = 900
[ ∵ The perpendicular drawn from the centre of a circle to chord bisects the chord]
But, OX ║ AB
∴ ∠LOX = 90 0......(i)
[ ∵ Sum of the consecutive interior angles on the same side of a transversal is 1800]
∴ M is the mid-point of the chord CD and O is the centre of the circle.
∴ ∠OMD = 90 0
[ ∵ The perpendicular drawn from the centre of a circle to a chord bisects the chord]

But OX ║ CD ....(ii)

127
[ ∵ Sum of the consecutive interior angles on the same side of a transversal is 1800]
∴ ∠MOX = 900
From above equations, we get
∠LOX + ∠MOX = 90 0 + 90 0 = 1800
⇒ ∠LOM = 1800
⇒ LM is a straight line passing through the centre of the circle. Hence Proved.
Ex.5  is a line which intersects two concentric circle (i.e., circles with the same centre) with common centre O at
A, B, C and D (as in figure). Prove that AB = CD.
Sol. Given :  is a line which intersects two concentric circles (i.e., circles with the same centre) with common
centre O at A, B, C and D.
To Prove : AB = CD.
Construction : Draw OE ⊥ 
Proof :
∴ The perpendicular drawn from the centre of a circle to a chord bisects the chord
∴ AE = ED .....(i)
And BE = EC ....(ii)
Subtracting (ii) from (i), we get
AE - BE = ED - EC
⇒ AB = CD. Hence Proved.
Ex.6 PQ and RS are two parallel chords of a circle whose centre is O and radius is 10 cm. If PQ = 16 cm and RS =
12 cm, find the distance between PQ and RS, if they lie.
(i) on the same side of the centre O.
(ii) on opposite sides of the centre O.
Sol. (i) Draw the perpendicular bisectors OL and OM of PQ and RS respectively.
∴ PQ ║ RS
∴ OL and OM are in the same line.
⇒ O, L and M are collinear.
Join OP and OR.
In right triangle OLP,
OP2 = OL2 + PL2 [By Pythagoras Theorem]
2
1 
⇒ (10)2 = OL2 +  × pq 
 2 
[ ∴ The perpendicular drawn from the centre of a circle to a chord bisects the chord]
2
1 
⇒ 100 = OL2 +  × 16 
 2 
⇒ 100 = OL2 + (8)2
⇒ 100 = OL2 + 64
⇒ OL2 = 100 - 64
⇒ OL2 = 36 = (6)2
⇒ OL = 6 cm

128
In right triangle OMR,
OR2 = OM2 + RM2 [By Pythagoras Theorem]
2
1 
⇒ OR2 = OM2 +  × RS 
2 
[ ∵ The perpendicular drawn from the centre of a circle to a chord bisects the chord]
2
1 
⇒ (10)2 = OM2 +  × 12 
2 
⇒ (10)2 = OM2 + (6)2
⇒ OM2 = (10)2 - (6)2 = (10 - 6)(10 + 6) = (4)(16) = 64 = (8)2
⇒ OM = 8 cm
∴ LM = OM - OL = 8 - 6 = 2 cm
Hence, the distance between PQ and RS, if they lie on he same side of the centre O, is 2 cm.
(ii) Draw the perpendicular bisectors OL and OM of PQ and RS respectively.
∴ PQ ║ RS
∴ OL and OM are in the same line
⇒ L, O and M are collinear.
Join OP nd OR.
In right triangle OLP,
OP2 = OL2 + PL2 [By Pythagoras Theorem]
2
1 
⇒ OP2 = OL2 +  × pQ 
2 
[ ∵ The perpendicular drawn from the centre of a circle to a chord bisects the chord]
2
1 
⇒ (10))2 = OL2 +  × 16 
 2 
⇒ 100 = OL2 + (8)2
⇒ 100 = OL2 + 64
⇒ OL2 = 100 - 64
⇒ OL2 = 36 = (6)2
⇒ OL = 6 cm
In right triangle OMR,
OR2 = OM2 + RM2 [By Pythagoras Theorem]
2
1 
⇒ OR2 = OM2 +  × 12 
 2 
[ ∵ The perpendicular drawn from the centre of a circle to a chord bisects the chord]
2
1 
⇒ (10)2 = OM2 +  × RS 
2 
⇒ (10)2 = OM2 + (6)2
⇒ OM2 = (10)2 - (6)2 = (10 - 6)(10 + 6) = (4)(16) = 64 = (8)2
⇒ OM = 8 cm
∴ LM = OL + OM = 6 + 8 = 14 cm
Hence, the distance between PQ and RS, if they lie on the opposite side of the centre O, is 14 cm.

129
Theorem-4 : Equal chords of a circle (or of congruent circles) are equidistant from the centre (or centres).

Given : A circle have two equal chords AB & CD. .e. AB = CD and OM ⊥ AB, ON ⊥ CD
To Prove : OM = ON
Construction : Join OB & OD
Proof : AB = CD (Given)
[ ∵ The perpendicular drawn from the centre of a circle to bisect the chord.]
1 1
∴ AB = CD
2 2
⇒ BM = DN
In ∆OMB & ∆OND
∠OMB = ∠OND = 90 0 [Given]
OB = OD [Radii of same circle]
Side BM = Side DN [Proved above]
∴ ∆OMB ≅ ∆OND [By R.H.S.]
∴ OM = ON [By cpctc] Hence Proved.
REMARK :
Chords equidistant from the centre of a circle are equal in length.

Ex.7 AB and CD are equal chords of a circle whose centre is O. When produced, these chords meet at E. Prove
that EB = ED.
Sol. Given : AB and CD are equal chords of a circle whose centre is O. When produced, these chords meet at E.
To Prove : EB = ED.
Construction : From O draw OP ⊥ AB and OQ ⊥ CD. Join OE.
Proof : ∴ AB = CD [Given]
∴ OP = OQ [ ∴ Equal chords of a circle are equidistant from the centre]
Now in right tingles OPE and OQE,
OE = OE [Common]
Side OP = Side OQ [Proved above]
∴ ∆OPE ≅ ∆OQE [By RHS]
∴ OE = QE [By cpctc]
1 1
⇒ PE - AB = QE - CD [ ∴ AB = CD (Given)]
2 2
⇒ PE - PB = QE - QD
⇒ EB = ED. Hence Proved.

130
Ex.8 Bisector AD of ∠BAC of ∆ABC passed through the centre O of the circumcircle of ∆ABC. Prove that AB =
AC.
Sol. Given : Bisector AD of ∠BAC of ∆ABC passed through the centre O of the circumcircle of ∆ABC,
To Prove : AB = AC.
Construction : Draw OP ⊥ AB and OQ ⊥ AC.
Proof :

In ∆APO and ∆AQO,


∠OPA = ∠OQA [Each = 90 0 (by construction)]
∠OAP = ∠OAQ [Given]
OA = OA [Common]
∴ ∆APO ≅ ∆AQO [By ASS cong. prog.]
∴ OP = OQ [By cpctc]
∴ AB = AC. [ ∵ Chords equidistant from the centre are equal] Hence Proved.
Ex.9 AB and CD are the chords of a circle whose centre is O. They intersect each other at P. If PO be the bisector
of ∠APD, prove that AB = CD.
OR
In the given figure, O is the centre of the circle and PO bisect the angle APD. prove that AB = CD.
Sol. Given : AB and CD are the chords of a circle whose centre is O. They interest each other at P. PO is the
bisector of ∠APD.
To Prove : AB = CD.
Construction : Draw OR ⊥ AB and OQ ⊥ CD.
Proof : In ∆OPR and ∆OPQ,
∠OPR = ∠OPQ [Given]
OP = OP [Common]
And ∠ORP = ∠OQP [Each = 90 0]
∴ ∆ORP ≅ ∆OPQ [By AAS]
∴ OR = OQ [By cpctc]
∴ AB = CD [ ∵ Chords of a circle which are equidistant from the centre are equal]
REMARK :
Angle Subtended by an Arc of a Circle :
In figure, the angle subtended by the minor arc PQ at O is ∠POQ and the angle subtended by the major arc
PQ at O is reflex angle ∠POQ.

131
EXERCISE
OBJECTIVE DPP # 14.1

1. If two circular wheels rotate on a horizontal road then locus of their centres will be
(A) Circles (B) Rectangle (C) Two straight line (D) Parallelogram
2. In a plane locus of a centre of circle of radius r, which passes through a fixed point
(A) rectangle (B) A circle (C) A straight line (D) Two straight line
3. In a circle of radius 10 cm, the length of chord whose distance is 6 cm from the centre is
(A) 4 cm (B) 5 cm (C) 8 cm (D) 16 cm
4. If a chord a length 8 cm is situated at a distance of 3 cm form centre, then the diameter of circle is :
(A) 11 cm (B) 10 m (C) 12 cm (D) 15 cm
5. In a circle the lengths of chords which are situated at a equal distance from centre are :
(A) double (B) four times (C) equal (D) three times

SUBJECTIVE DPP # 14.2

1. The radius of a circle is 13 cm and the length of one of its chords is 10 cm . Find the distance of the chord
from the centre.
2. Show is the figure, O is the centre of the circle of radius 5 cm. OP ⊥ AB, OQ ℜ CD, AB ║ CD, AB = 6 cm
and CD = 8 cm. Determine PQ.

3. AB and CD are two parallel chords of a circle such that AB = 10 cm and CD 24 cm. If the chords are on the
opposite side of the centre and the distance between is 17 cm, Find the radius of the circle.
4. In a circle of radius 5 cm, AB and AC are two chords such that AB = AC = 6 cm. Find the length of the
chord BC.
5. AB and CD are two parallel chords of a circle whose diameter is AC. Prove that AB = CD.
6. Two circles of radii 10 cm and 8 cm interest and the length of the common chord is 12 cm. Find the distance
between their centries.
7. Two circles with centre A and B and of radii 5 cm and 3 cm touch each other internally. If the perpendicular
bisector of segment AB meet the bibber circle is P and Q, find the length of PQ.

132
CIRCLE

ML - 15

SOME IMPORTANT THEOREMS


Theorem-1 : Equal chords of a circle subtend equal angles at the centre.
Given : A circle with centre O in which chord PQ = chord RS.
To Prove : ∠POQ = ∠ROS.
Proof : In ∆POQ and ∆ROS,
OP = OR [Radii of the same circle]
OQ = OS [Radii of the same circle]
PW = RS [Given]
⇒ ∆POQ = ∆ROS [By SSS]
⇒ ∠POQ = ∠ROS [By cpctc] Hence Proved.
Theorem-2 : If the angles subtended by the chords at the centre (of a circle) are equal then the chords are
equal.
Given : A circle with centre O . Chords PQ and RS subtend equal angles at the enter of the circle.
i.e. ∠POQ = ∠ROS
To Prove : Chord PQ = chord RS.
Proof : In ∆POQ and ∆ROS,
∠POQ = ∠ROS [Given]
OP = OR [Radii of the same circle]
OQ = OS [Radii of the same circle]
⇒ ∆POQ ≅ ∆ROS [By SSS]
⇒ chord PQ = chord RS [By cpctc] Hence Proved.
Corollary-1 : Two arc of a circle are congruent, if the angles subtended by them at the centre are equal.
Corollary 2 : If two arcs of a circle are equal, they subtend equal angles at the centre.
Corollary 3 : If two arc of a circle are congruent (equal), their corresponding chords are equal.
Corollary 4: If two chords of a circle are equal, their corresponding arc are also equal.
∠AOB = ∠COD
∴ Chord AB = Chord CD
∴ Arc APB = Arc COD.

133
Theorem-3 : The angle subtended by an arc at the centre is double the angle subtended by it at any point
on the remaining part of the circle.
Given : An arc PQ of a circle subtending angles POQ at the centre O and PAQ at a point A on the
remaining part of the circle.
To Prove : ∠POQ = 2∠PAQ.
Construction : Join AO and extend it to a point B.

(A) (B) (C)


Proof : There arises three cases :
(A) are PQ is minor
(B) arc PQ s a semi - circle
(C) arc PQ is major.
In all the cases,
∠BOQ = ∠OAQ + ∠AQO ....(i)
[ ∵ An exterior angle of triangle is equal to the sum of the two interior opposite angles]
In OAQ,
OA = OQ [Radii of a circle]
∴ ∠OAQ = ∠OQA ...(ii) [Angles opposite equal of a triangle are equal]
(i) and (ii), give,
∠BOQ = 2∠OAQ ....(iii)
Similarly,
∠BOP = 2∠OAP ....(iv)
Adding (iii) and (iv), we get
∠BOP + ∠BOQ = 2(∠OAP + ∠OAQ)
⇒ ∠POQ = 2∠PA. ....(v)
NOTE : For the case (C), where PQ is the major arc, (v) is replaced by reflex angles.
Thus, ∠POQ = 2∠PAQ.
Theorem- 4 : Angles in the same segment of a circle are equal.
Proof : Let P and Q be any two points on a circle to form a chord PQ, A and C any other points on the
remaining part of the circle and O be the centre of the circle. Then,
∠POQ = 2∠PAQ ...(i)
And ∠POQ = 2∠PCQ ...(ii)
From above equations, we get
2∠PAQ = 2∠PCQ
⇒ ∠PAQ = ∠PCQ Hence Proved

134
Theorem-5 : Angle in the semicircle is a right angle.
Proof : ∠PAQ is an angle in the segment, which is a semicircle.
1 1
∴ ∠PAQ = ∠PAO = × 180 0 = 90 0
2 2
[∴ ∠PQR is straight line angle or ∠PQR = 1800]
If we take any other point C on the semicircle, then again we get
1 1
∠PCQ = ∠POQ = × 180 0 = 90 0 Hence Proved.
2 2

Theorem-6: If a line segment joining two points subtend equal angles at two other points lying on the
same side of the lien containing the line segment the four points lie on a circle (i.e., they are concyclic).
Given : AB is a line segment, which subtends equal angles at two points C and D. i.e., ∠ACB = ∠ADB.
To Prove : The points A, B, C and D lie on a circle.
Proof : Let us draw a circle through the points A, C and B.
Suppose it does not pass through the point D.
Then it will intersect AD (or extended AD) at a point, say E (or E’).
If points A,C,E and B lie on a circle,
∠ACD = ∠AEB [∴ Angles in the same segment of circle are equal]
But it is given that ∠ACB = ∠ADB
Therefore, ∠AEB = ∠ADB
This is possible only when E coincides with D. [As otherwise ∠AEB > ∠ADB]
Similarly, E’ should also coincide with D. So A, B, C and D are concyclic Hence Proved.
CYCLIC QUADRILATERAL
A quadrilateral ABCD is called cyclic if all the four vertices of it lie on a circle.

Theorem-7 : The sum of either pair of opposite angles of a cyclic quadrilateral is 1800
Given : A cyclic quadrilateral ABCD.
To Prove : ∠A + ∠C = ∠B + ∠D = 180 0
Construction : Join AC and BD.
Proof : ∠ACB = ∠ADB [Angles of same segment]
And ∠BAC = ∠BDC [Angles of same segment]
∴ ∠ACB + ∠BAC = ∠ADB + ∠BDC =∠ADC.
Adding ∠ABC to both sides, we get
∠ACB + ∠BAC + ∠ABC =∠ADC + ∠ABC.
The left side being the sum of three angles of ∆ABC is equal to 180 0.
∴ ∠ADC + ∠ABC = 180 0
i.e., ∠D + ∠B = 180 0
∴ ∠A + ∠C = 3600 -(∠B + ∠D) = 1800 [∴ ∠A + ∠B + ∠C + ∠D = 3600] Hence Proved.

135
Corollary : If the sum of a pair of opposite angles of a quadrilateral is 1800, then quadrilateral is cyclic.
Ex.1 In figure, ∠ABC = 690, ∠ACB = 310, find ∠BDC.
Sol. In ∆ABC.
∠BAC + ∠ABC + ∠ACB = 180 0
[Sum of all the angles of a triangle is 180 0]
⇒ ∠BAC + 69 0 + 31 0 = 1800
⇒ ∠BAC + 1000 = 180 0
⇒ ∠BAC = 1800 - 100 0 = 80 0
Now, ∠BDC = ∠BAC = 800. Ans. [Angles in the same segment of a circle are equal]
Ex.2 ABCD is a cyclic quadrilateral whose diagonals intersect at a point E. If ∠DBC = 700, ∠BAC is 30 0, find
∠BCD. Further, if B = BC, find ∠ECD.

Sol. ∠CDB = ∠BAC = 300 ...(i) [Angles in the same segment of a circle are equal]
0
∠DBC = 70 ....(ii)
In ∆BCD,
∠BCD + ∠DBC + ∠CDB = 1800 [Sum of all he angles of a triangle is 1800]
0 0
⇒ ∠BCD + 70 + 30.0 = 180 [Using (i) and (ii)
0 0
⇒ ∠BCD + 100 = 180
⇒ ∠BCD = 1800 - 1000
⇒ ∠BCD = 800 ...(iii)
In ∆ABC,
AB = BC
∴ ∠BCA = ∠BAC = 300 ...(iv) [Angles opposite to equal sides of a triangle are equal]
0
Now, ∠BCD = 80 [From (iii)]
⇒ ∠BCA + ∠ECD = 800
⇒ 300 + ∠ECD = 800
⇒ ∠ECD = 800 - 300
⇒ ∠ECD = 500

Ex.3 If the nonparallel side of a trapezium are equal, prove that it is cyclic.
Sol. Given : ABCD is a trapezium whose two non-parallel sides AB and BC are equal.
To Prove : Trapezium ABCD is a cyclic.
Construction : Draw BE ║ AD.
Proof : ∴ AB ║ DE [Given]
AD ║ BE [By construction]
∴ Quadrilateral ABCD is a parallelogram.
∴ ∠BAD = ∠BED ....(i) [Opp. angles of a║gm]
And, AD = BE ....(ii) [Opp. sides of a ║gm]
But AD = BC ...(iii) [Given]

136
From (ii) and (iii),
BE = BC
∴ ∠BEC = ∠BCE ....(iv) [Angles opposite to equal sides]
0
∠BEC + ∠BED = 180 [Linear Pair Axiom]
⇒ ∠BCE + ∠BAD = 180 0 [From (iv) and (i)]
⇒ Trapezium ABCD is cyclic.
[∴ If a pair of opposite angles of a quadrilateral 1800, then the quadrilateral is cyclic] Hence Proved.
Ex.4 Prove that a cyclic parallelogram is a rectangle.
Sol. Given : ABCD is a cyclic parallelogram.
To Prove : ABCD is a rectangle.
Proof : ∴ ABCD is a cyclic quadrilateral
∴ ∠1 + ∠2 = 1800 ....(i)
[∴ Opposite angles of a cyclic quadrilateral are supplementary]
∴ ABCD is a parallelogram
∴ ∠1 = ∠2 ...(ii) [Opp. angles of a ║ gm]
From (i) and (ii),
∠1 = ∠2 = 900
∴ ║gm ABCD is a rectangle. Hence Proved.
Ex.5 Bisectors of angles A, B and C of a triangle ABC intersect its circumcircle at D, E and F respectively.
1 1 ∠C
Prove that the angles of the triangle DEF are 900 − A ,900 − B and 90 0 − .
2 2 2
Sol. Given : Bisectors of angles A, B and C of a triangle ABC intersect its circumcircle at D, E and F respectively.

∠A ∠B C
To Prove : The angles of the ∆DEF are 90 0 − ,90 0 − and 90 0 − respectively.
2 2 2
Construction : Join DE, EF and FD.
Proof : ∠FDE = ∠FDA + ∠EDA = ∠FCA + ∠EBA [∴ Angles in the same segment are equal ]
1 1
= ∠C + ∠B
2 2
∠C + ∠B 1800 − ∠A
⇒ ∠D = = [∴ In ∆ABC, ∠A + ∠B + ∠C = 1800]
2 2
∠A
⇒ ∠D = 90 0 -
2
Similarly, we can show that
∠B
∠E = 900 -
2
∠C
And ∠F = 90 0 - . Hence Proved.
2

137
Ex.6 Find the area of a triangle, the radius of whose circumcircle is 3 cm and the length of the altitude drawn
from the opposite vertex to the hypotenuse is 2 cm.
Sol. We know that the hypotenuse of a right angled triangle is the diameter of its circumcircle.
∴ BC = 2OB = 2 × 3 = 6 cm
Let, AD ⊥ BC
AD = 2 cm [Given]
1
∴ Area of ∆ABC = (BC)(AD)
2
1
= (6)(2)
2
= 6 cm2. Ans.
Ex.7 In figure, PQ is a diameter of a circle with centre O. IF ∠PQR = 650, ∠SPR = 400, ∠PQ M = 50 0, find ∠QPR,
∠PRS and ∠QPM.
Sol. (i) ∠QPR
∴ PQ is a diameter
∴ ∠PRQ = 90 0 [Angle in a semi-circle is 900]
In ∆PQR,
∠QPR + ∠PRQ + ∠PQR = 1800 [Angle Sum Property of a triangle]
⇒ ∠QPR + 90 0 + 65 0 = 1800
⇒ ∠QPR + 1550 = 180 0
⇒ ∠QPR = 1800 - 155 0
⇒ ∠QPR = 25 0.
(ii) ∠PRS
∴ PQRS is a cyclic quadrilateral
∴ ∠PSR + ∠PQR = 1800 [∴ Opposite angles of a cyclic quadrilateral are supplementary]
0 0
⇒ ∠PSR + 65 = 180
⇒ ∠PSR = 1800 - 650
⇒ ∠PSR = 1150
In ∆PSR,
∠PSR + ∠SPR + ∠PRS = 180 0 [Angles Sum Property of a triangle]
⇒ 115 0 + 40 0 + ∠PRS = 1800
⇒ 115 0 + ∠PRS = 1800
⇒ ∠PRS = 1800 - 1550
⇒ ∠PRS = 250
(iii) ∠QPM
∴ PQ is a diameter
∴ ∠PMQ = 90 0 [ ∵ Angle in a semi - circle is 900]
In ∆PMQ,
∠PMQ + ∠PQM + ∠QPM = 1800 [Angle sum Property of a triangle]
⇒ 900 + 500 + ∠QPM = 1800
⇒ 140 0 + ∠QPM = 180 0
⇒ ∠QPM = 1800 - 140 0
⇒ ∠QPM = 40 0.

138
Ex.8 In figure, O is the centre of the circle. Prove that
∠x + ∠y = ∠z.
1 1
Sol. ∠EBF = ∠EOF = ∠z [ ∵ Angle subtended by an arc of a circle at the centre in twice the angle
2 2
subtended by it at any point of the remaining part of the circle]
1
∴ ∠ABF = 1800 - ∠z ...(i) [Linear Pair Axiom]
2
1 1
∠EDF = ∠EOF = ∠z
2 2

[ ∵ Angle subtend by any arc of a circle at the centre is twice the angle subtended by it at any point of the
remaining part of the circle]
1
∴ ∠ADE = 1800 - ∠z ....(ii) [Linear Pair Axiom]
2
∠BCD = ∠ECF = ∠y [Vert. Opp. Angle]
∠BAD = ∠x
In quadrilateral ABCD
∠ABC + ∠BCD + ∠CDA + ∠BAD = 2 × 1800 [ Angle Sum Property of a quadrilateral]
1 1
⇒ 180 0 - ∠z + ∠y + 1800 - ∠z + ∠x = 2 × 1800
2 2
⇒ ∠x + ∠y = ∠z Hence Proved.
Ex.9 AB is a diameter of the circle with centre O and chord CD is equal to radius OC, AC and BD produced meet
at P. Prove that ∠CPD = 60 0.
Sol. Given : AB is a diameter of the circle with centre O and chord CD is equal to radius OC. AC and BD
produced meet at P.
To Prove : ∠CPD = 600
Construction : Join AD.
Proof : In ∆OCD,
OC = OD ...(i) [Radii of the same circle]
OC = CD ....(ii) [Given]
From (i) and (ii),
OC = OD = CD
∴ ∆OCD is equilateral
∴ ∠COD = 600
1 1
∴ ∠CAD = ∠COD = ∠(60 0) = 300
2 2
[ ∵ Angle subtended by any arc of a circle at the centre is twice the angle subtended by it at any point of the
reaming part of the circle]
⇒ ∠PAD = 300 .....(iii)
0
And, ∠ADB = 90 .....(iv) [Angle in a semi-circle]
⇒ ∠ADB + ∠ADP = 1800 [Linear Pair Axiom]
0 0
⇒ 90 + ∠ADP = 180 [From (iv)]
⇒ ∠ADP = 900 ....(v)

139
In ∆DP,
∠ADP + ∠PAD + ∠ADP = 1800 [ ∵ The sum of the three angles of a triangles is 180 0]
⇒ ∠APD + 300 + 900 = 1800 [From (iii) and (v)]
0 0
⇒ ∠APD + 120 = 180
⇒ ∠APD = 1800 - 1200 = 60 0
⇒ ∠CPD = 600. Hence Proved.

Ex.10 Prove that the quadrilateral formed by angle bisectors of a cyclic quadrilateral is also cyclic.
Sol. Given : ABCD is a cyclic quadrilateral. Its angle bisectors from a quadrilateral PQRS.
To Prove : PQRS is a cyclic quadrilateral.

Proof : ∠1 + ∠2 + ∠3 = 1800 ...(i) [ ∵ Sum of the angles of a ∆ is 1800]


0
∠4 + ∠5 + ∠ 6 = 180 ...(ii) [ ∵ Su m of the angles of a ∆ is 180 0]
∴ ∠1 + ∠2 + ∠3 + ∠4 + ∠5 + ∠6 =360.. ....(iii) [Adding (i) and (ii)]
1
But ∠2 + ∠3 + ∠6 + ∠5 = [∠A + ∠B + ∠C + ∠D]
2
1
= . 360 0 = 1800 [ ∵ Sum of the angles of quadrilateral is 3600]
2
∴ ∠1 + ∠4 = 3600 - (∠2 + ∠3 + ∠6 + ∠5)
∴ PQRS is a cyclic quadrilateral.
[ ∵ If the sum of any pair of opposite angles of a quadrilateral is 180 0, then the quadrilateral is a cyclic]
Hence Proved.
Ex.11 Prove that the angle bisectors of the angles formed by producing opposite sides of a cyclic quadrilateral
(Provided they are not parallel) intersect a right angle.
Sol. Given : ABCD is a cyclic quadrilateral. Its opposite sides DA and CB are produced to meet at P and
opposite sides AB and DC are produced to meet at Q. The bisectors of ∠P and ∠Q meet is F.
To Prove : ∠PFQ = 900.
Construction : Produce PF to meet DC is G.
Proof : In ∆PEB,
∠5 = ∠2 + ∠6 .....(i)
[ ∵ Exterior angle of a triangle is equal to the sum of interior opposite angles]
But ∠2 = ∠1
And, ∠6 = ∠D [ ∵ In a cyclic quadrilateral, exterior angle = interior opposite angle]
∴ ∠5 = ∠1 + ∠D ....(ii) [From (i)]
Now in ∆PDG,
∠7 = ∠1 + ∠D ...(iii)
[ ∵ Exterior angle of a triangle is equal to the sum of interior opposite angles]
Frim (ii) and (iii), we have
∠5 = ∠7

140
Now, in ∆ QEF and ∆ QGF, [Proved above]
∠5 = ∠7 [Common side]
QF = QF [Given]
∠3 = ∠4 [AAS criterion]
∴ ∆QEF ≅ ∆QGE [By cpctc]
∴ ∠8 = ∠9
But ∠8 + ∠9 = 1800
∴ ∠8 = ∠9 = 900 [Linear Pair Axiom]
∴ ∠PFQ = 900 Hence Proved.
Ex.12 Two concentric circles with centre O have A, B, C, D as the points of intersection with the line  as shown
in the figure. If AD = 12 cm and BC = 8 cm, find the length of AB, CD, AC and BD.
Sol. Since OM ⊥ BC, a chord of the circle,
∴ is bisects BC.
1 1
∴ BM = CM = (BC) = (8) = 4 cm
2 2
Since, OM ⊥ AD, a chord of the circle,
∴ it bisects AD.
1 1
∴ AM = AD = AD = (8) = 4 cm
2 2
Since, OM ⊥ CD, a chord of the circle,
∴ it bisects AD.
1 1
∴ AM = MD = AD = (12) = 6 cm
2 2
Now, AB = AM - BM = 6 - 4 = 2 cm
CD = MD - MD = 6 - 4 = 2 cm
AC = AM + MC = 6 + 4 = 10 cm
BD = BM + MD = 4 + 6 = 10 cm
Ex.13 OABC is a rhombus whose three vertices, A B and C lie on a circle with centre O. If the radius of the circle is
10 cm. Find the area of the rhombus.
Sol. Since OABC is a rhombus
∴ OA = AB = BC = OC = 10 cm
1 1
Now, OD ⊥ BC ⇒ CD = BC = (10) = 5 cm
2 2
∴ By Pythagoras theorem,
OC2 = OD2 + DC2
⇒ OD2 = OC2 - DC2 = (10)2 - (5)2 = 100 - 25 = 75
⇒ OD = 75 = 5 3
1 1
∴ Area (∆OBC) = BC × OD = (10) × 5 3 = 25 3 sq. cm.
2 2

Ex.14 Chords AB and CD of a circle with centre O, intersect at a point E. If OE objects ∠AED. Prove that AB = CD.

Sol. In ∆OLE and ∆OME


∠OLE = ∠OME [900 each]
∠LEO = ∠MEO [Given]
And OE = OE [Common]
∴ ∆OLE ≅ ∆OME [By AAS Criteria]
⇒ OL = OM [By cpctc]

This chords AB and CD are equidistant from centre. But we know that only equal chords are equidistant
from centre.
⇒ AB = DC

141
Ex.15 In the given figure. AB is the chord of a circle with centre O. AB is produced to C such that BC = OB. CO is
joined and produced to meet the circle in D. If ∠ACD = y0 and ∠AOD = x0, prove that x0 = 3y0.
Sol. Since BC = OB [Given]

∴ ∠OCB = ∠BOC = y0 [ ∵ Angles opposite to equal sides are equal]


∠OBA = ∠BOC + ∠OCB = y0 + y0 = 2y0.
[ ∵ Exterior angle of a ∆ is equal to the sum of the opposite interior angles]
Also OA = OB [Radii of the same circle]
0
∠OAB = ∠OBA = 2y [Angles opposite to equal sides of a triangle are equal]
∠AOD = ∠OAC + ∠OCA
= 2y0 + y0 = 3y0
[ ∵ Exterior angle of a ∆ is equal to the sum of the opposite interior angles]
Hence x0 = 3y0 Hence Proved.
Ex.16 In the given figure, the chord ED is parallel to the diameter AC. Find ∠CED.
Sol. ∠CBE = ∠1 [∠s in the same segment]
∠1 = 50 0 ....(i) [ ∵ ∠CBE = 500]
∠AEC = 900 ....(ii) [Semicircle Angle is right angle]
Now, in ∆AEC,

∠1 + ∠AEC + ∠2 = 180 0 [ ∵ Sum of angles of a ∆ = 180 0]


∴ 500 + 900 + ∠2 = 1800
⇒ ∠2 = 1800 - 1400 = 400
Thus ∠2 = 400 ....(iii)
Also, ED ║ AC [Given]
∴ ∠@ = ∠3 [Alternate angles]
0 0
∴ 40 = ∠3 i.e., ∠3 = 40
Hence ∠CED = 400Ans.

142
Ex.17 ABCD is a parallelogram. The circle through A, B, C intersects CD (produced if necessary) at E. Prove that
AD = AE.
Sol. Given : ABCD is a parallelogram. The circle through A, B, C intersects CD, when produced in E.
To prove : AE = AD.
Proof : Since ABCE is a cyclic quadrilateral
∴ ∠1 + ∠2 = 1800 ....(i) [opposite angles of a cyclio quadrilateral are supplementary]

Also ∠3 + ∠4 = 180 0 [linear pair] ....(ii)


From (i) and (ii), we get ∠1 + ∠2 = ∠3 + ∠4 ....(iii)
But ∠2 = ∠3 ....(iv)
∴ From (iii) and (iv), we get ∠1 = ∠4
Now in ∆ADE, since ∠1 = ∠4
AD = AE [Sides opp. to equal angles of a triangle are equal]

Hence Proved.

EXERCISE

OBJECTIVE DPP # 15.1

1. I the given circle ABCD, O is the centre and ∠BDE = 420. The ∠ACB is equal to :
(A) 480
(B) 450
(C) 420
(C) 600

2. In the diagram, O is the centre of the circle. The angles CBD is equal to :
(A) 250
(B) 500
(C) 400
(D) 1300

3. In the given figure, ∠CAB = 800, ∠ABC = 400. The sum of ∠DAB + ∠ABD is equal to :
(A) 800
(B) 100 0
(C) 120 0
(D) 1400

143
4. In the given figure, if C is the centre of the circle and ∠PC = 250 and ∠PRC = 15 0, then ∠QCR is equal to :
(A) 400
(B) 600
(C) 800
(D) 1200
5. In a cyclic quadrilateral if ∠B - ∠D = 600, then the smaller of the angles B and D is :
(A) 300 (B) 450 (C) 60 0 (D) 75 0
6. Three wires of length  1 ,  2 ,  3 from a triangle surmounted by another circular wire, If  3 is the diameter

and  3 = 2 1 , then the angle between  1 and  3 will be

(A) 300 (B) 600 (C) 45 0 (D) 90 0


7. In a circle with centre O, OD ⊥ chord AB. If BC is the diameter, then :
(A) AC = BC (B) OD = BC (C) AC = 2OD (D) None of these
8. In the diagram two equal circles of radius 4 cm intersect each other such that each passes through the centre
of the other. Find the length of the common chord.

(A) 2 3 cm

(B) 4 3 cm

(C) 4 2 cm
(D) 8 cm
9. The sides AB and DC of cyclic quadrilateral ABCD are produced to meet at P, the sides AD and BC are
produced to meet at Q. If ∠ADC = 850 and ∠BPC = 40 0, then ∠CQD equals.
(A) 300 (B) 450 (C) 60 0 (D) 75 0
10. In the given figure, if ∠ACB = 40 0, ∠DPB = 1200, then will be :
(A) 400
(B) 200
(C) 0 0
(D) 600

11. Any cyclic parallelogram is a.


(A) rectangle (B) rhombus (C) trapezium (D) square

12. The locus of the centre of all circles of given radius r, in the same planes, passing through a fixed point is :
(A) A point (B) A circle (C) A straight line (D) Two straight lines

13. In a cyclic quadrilateral if ∠A - ∠C = 700, then the greater of the angles A and C is equal to :
(A) 950 (B) 105 0 (C) 1250 (D) 1150

144
14. The length of a chord a circle is equal to the radius of the circle. The angle which this chord subtends on the
longer segment of the circle is equal to :
(A) 300
(B) 450
(C) 600
(D) 900
15. If a trapezium is cyclic then,
(A) Its parallel sides are equal.
(B) Its non-parallel sides are equal.
(C) Its diagonals are not equal.
(D) None of these above

SUBJECTIVE DPP - 15.2

1. In the given figure, BC is diameter bisecting ∠ACD, find the values of a, b (o is centre of circle).

2. In the given figure, find the value of a & b.

3. Find the value of a & b.

145
4. Find the value of a & b.

5. Prove that a + 2b = 900

6. ABCD is a cyclic quadrilateral in which ∠A = (x + y + 10)0, ∠B = (y + 20)0, ∠C = (x + y - 30)0 and ∠D = (x +


y)0. Find x and y.

7. Find the value of a and b, if b = 2a.

8. Find the value of a if BC ║ EA

146
9. In the adjoining fig., O is centre of the circle, chord AC and BD are perpendicular to each other, ∠OAB = a
and ∠DBC = b. Show that a = b.

10. In the fig. given below, AB is diameter of the circle whose centre is O. Given that : ∠ECD = ∠EDC = 320.
Show that ∠COF = ∠CEF.

11. In the given fig., AC is the diameter of circle centre O. Chord BD is perpendicular to AB. Write down the
angles p,q & r in terms of x.

12. Prove that the line segment joining the mid-point of hypotenuse of a right triangle to its opposite vertex is
half of the hypotenuse.

147
ANSWER KEY

(Objective DPP # 14.1)

Qus. 1 2 3 4 5

Ans. C B D B C

(Subjective DPP # 14.2)

1. 12 cm 2. 7 cm 3. 13 cm 4. 9.6 cm

6. 10 cm 7. 13.29 cm 8. 4 6 cm

(Objective DPP # 15.1)

Qus. 1 2 3 4 5 6 7 8 9 10

Ans. A A C C C B C B A B

A 11 12 13 14 15

Ans. A B C A B

(Subjective DPP # 15.2)

1. b = 900, a = 450 2. a = 50, b = 1700 3. a = 1400, b = 700

4. a = 400, b = 900 6. x = 40, y = 60

7. a = 400, b = 800 8. a = 1080

x x x
11. p = 90 0- , q = , and r = 90 -
2 2 2

148
CONSTRCUTIONS

ML - 16

TO CONSTRUCT THE BISECTOR OF A LINE SEGMENT


Ex.1 Draw a line segment of length 7.8 cm draw the perpendicular bisector of this line segment.

Sol. Given the given the segment be AB = 7.8 cm.


STEPS :
(i) Draw the line segment AB = 7.8 cm.
(ii) With point A as centre and a suitable radius, more than half the length of AB, draw arcs on both the
sides of AB.
(iii) With point B as centre and with the same radius draw arcs on both the sides of AB. Let these arc cut
at points P & Q as shown on in the figure.
(iv) Draw a line through the points P and Q. The line so obtained is the required perpendicular bisector
of given line segment AB.
Line PQ is perpendicular bisector of AB.
(A) PQ bisects AB i.e., OA = OB.
(B) PQ is perpendicular to AB i.e., ∠PAO = ∠POB = 90 0.
Proof : In ∆APQ and ∆BPQ :
AP = BP [By construction]
AQ = BQ [By construction]
PQ = PQ [Common]
⇒ ∆APQ = ∠BPQ [By SSS]
⇒ ∠APQ = ∠BPQ [By cpctc]
Now, in ∆ APO & ∆BPO
AP = BP [By construction]
OP = OP [Common side]
∠APO = ∠BPO [Proved above]
⇒ ∆APO ≅ ∆BPO [By SAS]
And, ∠POA = ∠POB
1800
= = 900 [ ∵ ∠POA + ∠POB = 1800]
2
⇒ PQ is perpendicular bisector of AB.

149
TO CONSTRCUT THE BISECTOR OF A GIVEN ANGLE
Let ABC be the given angle to be bisected.

STEPS :
(i) With B as centre and a suitable radius, draw an arc which cuts ray BA at point D and ray BC at point E.
(ii) Taking D and E as centres and with equal radii draw arcs which intersect each other at point F. In this
step, each equal radius must be more than half the length DE.
(iii) Join B and F and produce to get the ray BF.
Ray BF is the required bisector of the given angle ABC.
Proof : Join DF and EF.
In ∆ BDF and ∆ BEF :
BD = BE [Radii of the same arc]
DF = EF [Radii of the equal arcs]
BF = BF [Common]
⇒ ∆BDF ≅ ∆BEF [By SSS]
⇒ ∠DBF = ∠EBF [By cpctc]
i.e., ∠ABF = ∠CBF
⇒ BF bisects ∠ABC. Hence Proved.
TO CONSTRUCT THE REQUIRED ANGLE
(a) To Construct the Required Angle of 600 :
STEPS :
(i) Draw a line BC of any suitable length.
(ii) With B as centre and any suitable radius, draw an arc which cuts BC at point D.
(iii) With D as centre and radius same, as taken in step (ii), draw one more arc
which cuts previous arc at point E.
(iv) Join BE and produce upto any point A.
Then, ∠ABC = 60 0

(b) To Construct an Angle of 1200 :


STEPS
(i) Draw a line BC of any suitable length.
(ii) Taking B as centre and with any suitable radius, draw an arc which cuts BC at point D.
(iii) Taking D as centre, draw an arc of the same radius, as taken in step (ii), which cuts the first arc at point
E.
(iv) Taking E as centre and radius same, as taken in step (ii),
draw one more arc which cuts the first arc at point F.
(v) Join BF and produce upto any suitable point A.
Then, ∠ABC = 1200

150
(c) To Construct and Angle of 30 0 :
STEPS :
(i) Construct angle ABC = 600 by compass.
(ii) Draw BD, the bisector of angle ABC.
The, ∠DBC = 30 0
(d) To Construct an Angle of 90 0 :
STEPS
(i) Construct angle ABC = 120.0 by using compass.
(ii) Draw PB, the bisector of angle EBG.
Then, ∠PBC = 90 0
Alternative Method :
(i) Draw a line segment BC of any suitable length.
(ii) Produce CB upto a arbitrary point O.
(iii) Taking B as centre, draw as arc which cuts OC at points D and E.
(iv) Taking D and E as centres and with equal radii draw arcs
with cut each other at point P.
[The radii in this step must be of length more than half of DE.]
(v) Join BP and produce.
Then, ∠PBC = 90 0
(d) To Construct an Angle of 45 0
STEPS
(i) Draw ∠PBC = 900
(ii) Draw AB which bisects angle PBC,
Then, ∠ABC = 45 0
Alternative Method :
STEPS :
(i) Construct ∠ABC = 600
(ii) Draw BD, the bisector of angle ABC.
(iii) Draw BE, the bisector of angle ABD.
Then, ∠EBC = 45 0
(e) To Construct an Angle of 1050 :
STEPS :
(i) Construct ∠ABC = 120 0 and ∠PBC = 900
(ii) Draw BO, the bisector of ∠ABP.
Then, ∠OBC = 1050

151
(f) To Construct an Angle of 150 0.
STEPS :
(i) Draw line segment BC of any suitable length. Produce CB upto any point O.
(ii) With B as centre, draw an arc (with any suitable radius) which buts OC at points D and E.
(iii) With D as centre, draw an arc of the same radius, as taken in step 2, which cuts the first arc at point F.
(iv) With F as centre, draw one more arc of the same radius, staken in step 2, which cuts the first arc at
point G.
(v) Draw PB, the bisector of angle EBG.
Now ∠FBD = ∠GBF = ∠EBG = 600
Then, ∠PBC = 1500
(g) To Construct an Angle of 1350.
STEPS :
(i) Construct ∠PBC = 1500 and ∠GBC = 1200
(ii) Construct BQ, the bisector of angle PBG.
Then, ∠QBC = 1350

TO CONSTRUCT A TRIANGLE
Case (i) To construct an equilateral triangle when its one side is given.
Ex.2 Draw an equilateral triangle having each side of 2.5 cm.
Sol. Given one side of the equilateral triangle be 2.5 cm.
STEPS :
(i) Draw a line segment BC = 2.5 cm.
(ii) Through B, construct ray BP making angle 60 0 with BC.
i.e. ∠PBC= 600
(iii) Through C, construct CQ making angle 600 with BC
i.e., ∠QCB = 600
(iv) Let BP and CQ intersect each other at point A.
The n, ∆ABC is the require equilateral triangle.
Proof : Since, ∠ABC= ∠ACB = 600
∴ ∠BAC = 1800 - (600 + 600) = 600
⇒ All the angles of the ∆ABC drawn are equal.
⇒ All the sides of the ∆ABC drawn are equal.
⇒ ∆ABC is the required equilateral triangle. Hence Proved.
Alternate method :
If one side is 2.5 cm, then each side of the required equilateral triangle is 2.5 cm.
STEPS :
(i) Draw BC = 2.5 cm
(ii) With B as centre, draw an arc of radius 2.5 cm
(iii) With C as centre, draw an arc of radius 2.5 cm
(iv) Let the two arc intersect each other at point A. Join AB and AC.
Then, ABC is the required equilateral triangle.

152
Case (ii) When the base of the triangle, one base angle and the sum of other two sides are given.
Ex.3 Construct a triangle with 3 cm base and sum of other two sides is 8 cm and one base angle is 600.
Sol. Given the base BC of the triangle ABC be 3 cm, one base angle ∠B = 600 and the sum of the other two sides
be 8 cm i.e, AB + AC = 8 cm.
STEPS :
(i) Draw BC = 3 cm
(ii) At point B, draw PB so that ∠PBC = 600
(iii) From BP, cut BC = 8 cm.
(iv) Join D and C.
(v) Draw perpendicular bisector of CD, which meets BD at point A.
(vi) Join A and C.
Thus, ABC is the required triangle.
Proof : Since, OA is perpendicular bisector of CD
⇒ OC = OD
∠AOC = ∠AOD = 900
Also, OA = OA [Common]
∴ ∆AOC ≅ ∆AOD [By SAS]
⇒ AC = AD
∴ BD = BA + AD
= BA + AC
= Given sum of the other two sides
Thus, base BC and ∠B are draw as given and BD = AC. Hence Proved.
Ex.4 Construct a right triangle, when one side is 3.8 cm and the sum of the other side and hypotenuse is 6 cm.
Sol. Here, if we consider the required triangle to be ∆ABC, as shown alongside.
Clearly, AB = 3.8 cm, ∠B = 900 and BC + AC = 6 cm.
STEPS :
(i) Draw AB = 3.8 cm
(ii) Through B, draw line BP so that ∠ABP = 900
(iii) From BP, cut BD = 6 cm
(iv) Join A and D.
(v) Draw perpendicular bisector of AD, which meets BD at point C.
Thus, ABC is the required triangle.

153
Case (iii) When the base of the triangle, one base angle and the difference of the other two sides are
given.
Ex.5 Construct a triangle with base of 8 cm and difference between the length of other two sides is 3 cm and one
base angle is 60 0
Sol. Given the base BC of the required triangle ABC be 8 cm i.e., BC = 8 m, base angle B = 600 ant the difference
between the lengths of other two sides AB and AC be 3 cm.
i.e., AB - AC = 3 cm or AC - AB = 3 cm.
(a) When AB - AC = 3 cm i.e., AB > AC :
STEPS :
(i) Draw BC = 8 cm
(ii) Through point B, draw BP so that ∠PBC = 60 0
(iii) From BP cut BD = 3 cm.
(iv) Join D and C.
(v) Draw perpendicular bisector of DC ; which meets BP at point A.
(vi) Join A and C.
Thus, ∆BC is the required triangle.
Proof : Since OA is perpendicular bisector of CD
⇒ OD = OC
∠AOD = ∠AOC = 90 0
And, OA = OA [Common]
∴ ∆AOD ≅ ∆AOC [By SAS]
⇒ AD = AC [By cpctc]
Now, BD = BA - AD
= BA - AD
= BA - AC
= Given difference of other two sides.
Thus, the base BC and ∠B are drawn as given and BD = BA - AC. Hence Proved.
(b) When AC - AB = 3 cm i.e, AB < AC :
STEPS :
(i) Draw BC = 8 cm
(ii) Through B, draw line BP so that angle PBC = 600.
(iii) Produce BP backward upto a suitable point Q.
(iv) Fro BQ, cut BD = 3 cm.
(v) Join D and C.
(vi) Draw perpendicular bisector of DC, which meets BP at point A.
(vii) Join A and C.
Thus, ∆ABC is the required triangle.
Proof : Since, OA is perpendicular bisector of CD
⇒ OD = OC
∠AOD = ∠AOC = 90 0
And OA = OA [Common]
∴ ∆AOD ≅ ∆AOC [By SAS]
⇒ AD = AC [By cpctc]
Now, BD = AD - AB
= AC - AB [ ∵ AD = AC]
= Given difference of other two sides.
Thus, the base BC and ∠B are drawn as given and BD = AC - AB. Hence Proved.

154
Case (iv) When the perimeter of the triangle and both the base angles are given :
Ex.6 Contruct a triangle ABC with AB + BC + CA = 12 cm ∠B = 450 and ∠C = 600
Sol. Given the perimeter of the triangle ABC be 12 cm i.e., AB + BC + CA = 12 cm and both the base angles be
45 0 and 600 i.e., ∠B = 450 and ∠C = 60 0
STEPS :
(i) Draw a line segment PQ = 12 cm
(ii) At P, construct line PR so that ∠RPO = 45 0 and at Q, construct a line QS so that ∠SQP = 600
(iii) Draw bisector of angles RPQ and SQP which meet each other at point A.
(iv) Draw perpendicular bisector of AP, which meets PQ at point B.
(v) Draw perpendicular bisector of AQ, which meets PQ at point C.
(vi) Join AB and AC.
Thus, ABC is the required triangle.
Proof : Since, MB is perpendicular bisector of AP
⇒ ∆QNC ≅ ∆ANC [By SAS]
PB = AC
Similarly, NC is perpendicular bisector of AQ.
⇒ ∆QNC ≅ ∆ANC [By SAS]
⇒ CQ = AC [By cpctc]
Now, PQ = PB + BC + CQ
= AB + BC + AC
= Given perimeter of the ∆ABC drawn.
Also, ∠BPA = ∠BAP [As ∆ PMB ≅ ∆ A MB]
∴ ∠ABC = ∠BPA + ∠BAP [Ext. angle of a triangle = sum of two interior opposite angles]
∠ABC = ∠BPA + ∠BAP = 2 ∠BPA = ∠RPB = ∠ACB [Given]
∠ACB = ∠CQA + ∠CQA
= 2 ∠CQA [ ∵ ∆QNC ≅ ∆ANC ∴ ∠CQA = ∠CAQ]
= ∠SQC = Given base angle ACB.
Thus, given perimeter = perimeter of ∆ABC.
given one base angle = angle ABC
and, given other base angle = angle ACB.
Ex.7 Construct and equilateral triangle if its altitude is 3.2 cm.
Sol. Given In an equilateral ∆ABC an altitude AD = 3.2 cm
Required to Construct an equilateral triangle
ABC from the given data.
STEPS :
(i) Draw a line PQ and mark and point D on it.
(ii) Construct a ray DE perpendicular to PQ.
(iii) Cut off DA = 3.2 cm from DE.
1 
(iv) Construct ∠DAR = 300 =  × 60 0  .
2 
The ray AR intersects PQ at B.
(v) Cut off line segment DC = BD.
(vi) Join A and C. We get the required ∆ABC.

155
Ex.7 Construct a right angled triangle whose hypotenuse measures 8 cm and one side is 6 cm.
Sol. Given Hypotenuse AC of a ∆ABC = 8 cm and one side AB = 6 cm.
Required To construct a right angled ∆ABC from the given data.
STEP.
(i) Draw a line segment AC = 8 cm.
(ii) Mark the mid point O of AC.
(iii) With O as centre and radius OA, draw a semicircle on AC.
(iv) With A as centre and radius equal to 6 cm, draw an arc, cutting the semicircle a B.
(v) Jon A and B ; B and C. We get the required right angled triangle ABC.
Ex.8 Construct a ∆ABC in which BC = 6.4 cm, altitude from A is 3.2 cm and the median bisecting BC is 4 cm.
Sol. Given : one side BC = 6.4 cm, Altitude AD = 3.2 am and the median AL = 4 cm.
Required : To construct a ∆ABC form the given data.
STEP :
(i) Draw BC = 6.4 cm
(ii) Bisect BC at L.
(iii) Draw EF ║ BC at a distance 3.2 cm for BC.
(iv) With L as centre and radius equal to 4 cm, draw an arc, cutting EF at A.
(v) Join A and B ; A and C. We get the required triangle ABC.
Ex.9 Construct a ∆ABC in which ∠B = 30 0 and ∠C = 600 and the perpendicular from the vertex A to the base BC
is 4.8 cm.
Sol. Given : ∠B = 300 ∠C = 600, length of perpendicular from vertex A to be base BC = 4.8 cm.
Required : To construct a ∆ABC from the given data.
STEP :
(i) Draw any ray line PQ.
(ii) Take a point B on line PQ and construct ∠QBR = 30 0
(iii) Draw a line EF ║ PQ a distance of 4.8 cm from PQ,
cutting BR at A.
(iv) Construct an angle ∠FAC = 60 0, cutting PQ C.
(v) Join A and C. We get the required triangle ABC.
Ex.10 Construct a triangle ABC, the lengths of whose medians are 6 cm, 7cm and 6 cm.
Sol. Given : Median AD = 6 cm Median BE = 7 cm Median CF = 6 cm.
Required : To construct a ∆ABC from the given data.
STEP :
(i) Construct a ∆APQ with AP = 6 cm, PQ = 7 cm and AQ = 6 cm.
(ii) Draw the medians AE and PF of ∆APQ intersecting each other at G.
(iii) Produce AE to B such that GE = EB
(iv) Join B and Q and produce it to C, such that BQ = QC
(v) Join A and C. We get the required triangle ABC.

156
EXERCISE

SUBJECTIVE DPP # 16.1

For each angle, given below, make a separate construction. Draw a ray BC and an another ray BA so that
the ∠ABC is equal to :
10 10
1. 15 0 2. 22 3. 750 4. 52
2 2
10
5. 67 6. 1650 7. 135 0
2

8. Construct and equilateral triangle with side :


(i) 5 cm (ii) 5.4 cm (iii) 6.2 cm

9. Construct a triangle ABC, in which :


(i) base AB = 5.4 cm, ∠B = 450 and AC + BC = 9 cm.
(ii) base BC = 6 cm, ∠B = 600 and AB + AC = 9.6 cm.
(iii) base AC = 5 cm, ∠C = 900 and AB + BC = 10.6 cm.

10. Construct a right triangle, with base = 4 cm and the sum of the other side and hypotenuse = 9.4 cm.

11. Construct a triangle ABC, in which :


(i) BC = 4.8 cm, ∠B = 45 0 and AB - AC = 2.4 cm.
(ii) BC = 4.8 cm, ∠B = 450 and AC - AB = 2.4 cm.
(iii) AB = 5.3 cm, ∠A = 60 0 and AC - BC = 2 cm.
(iv) AB = 5.3 cm, ∠A = 600 and BC - AC = 2 cm.

12. Construct a triangle ABC, with :


(i) perimeter = 12 cm, ∠B = 450 and ∠C = 60 0.
(ii) perimeter = 11.6 cm, ∠B = 60 0 and ∠C = 900
(iii) perimeter = 11 cm, ∠A = 600 and ∠C = 450.
(iv) perimeter = 10 cm, ∠B = ∠C = 600

13. Construct as equilateral triangle with perimeter 15.6 cm.


14. Without finding the length of each side of the equilateral triangle construct it. If its perimeter is 16 cm.
15. Construct an equilateral triangle whose altitude is 4.8 cm.
16. Construct a ∆ PQR in which base QR = 4 cm, ∠R = 30 0 and PR - PQ = 1.1 cm.
17. Construct a ∆ XYZ with perimeter 9.6 cm and base angle 300 and 600
18. Construct a ∆ PQR in which PQ = 3.7 cm, QR = 3.6 cm and median PA = 3.1 cm.
19. Construct a ∆ DEF, the lengths of whose medians are 6 cm, 7 cm and 8 cm.
20. Construct on equilateral triangle, one of whose altitudes measures 6.4 cm.

157
HERON’S FORMULA

ML - 17

MENSURTION
A branch of mathematics which concerns itself measurement of lengths, areas and volumes of plane and
solid figure is called Monsuration.
(a) Perimeter :
The perimeter of a plane figure is the length of its boundary. In case of a triangle or a polygon, the
perimeter is the sum of the lengths of its sides.
(b) Units of Perimeter :
The unit of perimeter is the same as the unit of length i.e. centimetre (cm), metre(m), kilometere (k m) etc.
1 centimetre (cm) = 10 milimetre (mm)
1 decimetre (dm) = 10 centrimetre
1 metre (m) = 10 decimetre
= 100 centimetre
= 1000 milimetre
1 decamete (dam) = 10 metre
= 1000 centimetre
1 hectometre (hm) = 10 decametre
= 100 metre
1 kilometre (km) = 1000 metre
= 100 decametre
= 10 hectometre
AREA
The area of a plane figure is the measure of the surface enclosed by its boundary.
The area of a triangle or a polygon is the measure of the surface enclosed by its sides.
(a) Units of Area :
The various units of measuring area are, squire centimetre (cm2), square metre (m2), 1 hectare etc.
1 square centrimetre (cm2) =1 cm × 1 cm.
= 10 mm × 10 mm = 100 sq. mm.
2
1 square decimetre (cm ) = 1 dm × dm
= 10 cm × 10 cm = 100 sq. cm.
2
1 square metre (m ) =1m×1m
= 10 dm × 10 dm = 100 sq. dm.
2
1 square decametre (dam ) = 1 dam × 1 dam
= 10 m × 10 m = 100 sq. m.

158
1 square hectometre (hm2) = 10 dam × 10 dam = 100 sq. dam
(or 1 hectare) = 1000 sq. m.
1 square kilometre (km2) = 1 km × 1 km
= 10 hm × 10 hm = 100 sq. hm.
(b) Heron’s formula :
In ∆ABC if sides of triangle BC, CA, & AB are a, b, c respectively then
perimeter = 2s = a + b + c
Area = s( s − a)(s − b)(s − c)

(c) Perimeter and Area of a Triangle :


(i) Right - angled triangle :
For an right-angled triangle, let b be the base, h be the perpendicular and d be the hypotenuse. Then
(A) Perimeter = b + h + d
1 1
(B) Area = (Base × Height) = bh
2 2

(C) Hypotenuse, d = b 2 + h 2 [Pythagoras theorem]


(ii) Isosceles right-angled triangle
For an isosceles right-angled triangle, let a bet the equal sides, then

(A) Hypotenuse = a2 + a2 = 2 a

(B) Perimeter = 2a + 2a
1 1 1
(C) Area = (Base × Height) = (a × a) = a2.
2 2 2

(iii) Equilateral triangle


For an equilateral triangle, let each side be a, and the height of the triangle is h, then
(A) ∠A = ∠B = ∠C = 600
(B) ∠BAD = ∠CAD = 300
(C) AB = BC = AC = a(say)
a
(D) BD = DC =
2
2
a 3a 2
(E)   + h2 = a2 ⇒ h2 =
2 4

3
∴ Height (h) = a
2
1 1 3 3 2
(F) Area = (Base × Height) = ×a× a= a
2 2 2 4
(G) Perimeter = a + a + a = 3a.

159
Ex.1 The area of a triangle is 30 cm2. Find the base if the altitude exceeds the base by 7 cm.
Sol. Let base BC = x cm then altitude = (x + 7) cm
1
Area of ∆ABC = × base × height
2
1
⇒ 30 = (x)(x + 7)
2
⇒ 60 = x2 + 7x
⇒ x2 + 7x - 60 = 0
⇒ x2 + 12x - 5x - 60 = 0
⇒ x(x + 12) - 5(x + 12) = 0
⇒ (x - 5) (x + 12) = 0
⇒ x = 5 or x = -12
⇒ x=5 [ ∵ x ≠ -12
∴ Base (x) = 5 cm and Altitude = x + 7 = 5 + 7 = 12 cm. Ans.
Ex.2 The cost of turfing a triangle field at the rate of Rs. 45 per 100 m2 is Rs. 900. Find the height, if double the
base of the triangle is 5 times the height.
Sol. Let the height of triangular field be h metres.
It is given that 2 x (base) = 5 × (Height)
5
∴ Base = h
2
1
Area = × Base × Height
2
1 5 5
Area = × h × h = h2 ....(i)
2 2 4
∴ Cost of turfing the field is Rs. 45 per 100 m2
Total cos t
∴ Area =
Rate per sq.
900
=
45 / 100
9000
=
45
= 2000 m2 ....(ii)
From (i) and (ii)
5 2
h = 2000
4
⇒ 5h2 = 8000
⇒ h2 = 1600
⇒ h = 40 m
∴ Height of the triangular field is 40 cm. Ans.

160
Ex.3 From a point in the interior of an equilateral triangle, perpendicular drawn to the three sides are 8 cm, 10
cm and 11 cm respectively. Find the area of the triangle.
Sol. Let each side of the equilateral ∆ABC = x cm,
From an interior point O, OD, OE and OF be drawn perpendicular to BC, AC and AB respectively. It is
given that OD = 11 cm, OE = 8 cm and OF = 10 cm. Join OA, OB and OC.
Area of ∆ABC = Area of ∆OBC + Area of ∆OCA + Area of ∆OAB
1 1 1
= .x.11 + .x.8 + .x.10
2 2 2
29
= x cm2
2
But, area of an equilateral triangle, whose ease side is x
3 2
= x cm2
4
3 2 29
Therefore, x = x
4 2
4 × 29 58
∴ x= = cm
2× 3 3
29 58 841
∴ Area of ∆ABC = × cm 2 = cm 2
2 3 1.73
∴ Area of ∆ABC = 486.1 cm2 Ans.
Ex.4 The difference between the sides at right angles in a right-angled triangle is 14 cm. The area of the triangle
is 120 cm2. Calculate the perimeter of the triangle.
Sol. Let the sides containing the right angle be x cm and (x - 14) cm.
1 
The, its area =  .x.( x − 14 ) cm 2 .
2 
But, area = 120 cm2 [Given]
1
∴ x (x - 14) = 120
2
⇒ x2 - 14x - 240 = 0
⇒ x2 - 24x + 10x - 240 = 0
⇒ x(x - 24) + 10 (x - 24) = 0
⇒ (x - 24) (x + 10) = 0
⇒ x = 24 [Neglecting x = -10]
∴ one side = 24 cm, other side = (24 - 14) cm = 10 cm

Hypotenuse = ( 24)2 + (10 )2 cm

= 576 + 100 cm
= 576 cm
= 26 cm.
∴ Perimeter of the triangle = (24 + 10 + 26) c m = 60 cm. Ans.

161
Ex.5 Find the percentage increase in the area of a triangle if its each side is doubled.
Sol. Let a,b,c be the sides of the given triangle and s be its semi-perimeter
1
∴ s= (a + b + c) ....(i)
2
The sides of the new triangle are 2a, 2b and 2c.
Let s’ be its semi-perimeter.
1
∴ s’ = (2a + 2b + 2c) = a + b + c = 2s [Using (i)]
2
Let ∆ = Area of given triangle

∆ = s(s − a )(s − b)(s − c) .....(ii)

And, ∆’ = Area of new triangle

∆’ = s'( s'−2 a)(s'−2 b(s'−2 c

= 2s( 2s − 2 )( 2s − 2 b)( 2s − 2 c) [Using (i)]

= 16s(s − a)(s − b)(s − c

∆’ = 4 ∆
∴ Increase in the area of the triangle = ∆’ - ∆ = 4∆ - ∆ = 3∆

 3∆ 
∴ % increase in area =  × 100 % = 300% % Ans.
 ∆ 
Ex.6 An umbrella is made by stitching 10 triangular pieces of cloth of two different colours (see figure), each
piece measuring 20 cm, 50 cm and 50 cm. How much cloth of each colour is required for the umbrella ?
Sol. The sides of a triangular piece are
20 cm, 50 cm and 50 cm
a + b + c 20 + 50 + 50
s= = = 60 cm = 60 cm
2 2
Area of one triangular piece

= s( s − a)(s − b)(s − c)

= 60( 60 − 20 )(60 − 50 )(60 − 50 )

= 60 × 40 × 10 × 10 = 24000

= 200 6 cm2

Area of cloth of each colour for five triangular pieces = 5 × 200 6 = 1000 6 cm2 Ans.

162
EXERCISE

OBJECTIVE DPP 17.1

1. The area of the field ABGFEA is :

(A) 7225 m2 (B) 7230 m2 (C) 7235 m2 (D) 7240 m2

2. Area of shaded portion as shown in the figure :

(A) 12 m2 (B) 13 m2 (C) 14 m2 (D) 15 m2


3. The lengths of four sides and a diagonal of the given quadrilateral are indicated in the diagram. If a denotes
the area of quadrilateral, then A is

(A) 12 6 (B) 6 (C) 6 6 (D) 6’


4. In the sides of a triangle are doubled, then its area :
(A) Remains the same (B) Becomes doubled (C) Becomes three times (D) Becomes four times
5. Inside a triangular garden there is a flower bed in the form of a similar triangle. Around the flower bed
runs a uniform path of such a width that the side of the garden are double of the corresponding sides of the
flower bed. The areas of the path and the flower bed are in the ratio :
(A) 1 : 1 (B) 1 : 2 (C) 1 : 2 (D) 3 : 1

163
SUBJECTIVE DPP - 17.2

1. In the given figure, ∆ABC is a equilateral triangle the length of whose


side is equal to 10 cm and ∆DBC is right-angled at D and BD = 8 cm.

Find the area of the shaded region. Take 3 = 1.732.

2. Calculate the area of the triangle whose sides are 18 cm, 24 cm and 30 cm in length. Also, find the length of
the altitude corresponding to the smallest side of the triangle.
3. The sides of a triangle are 10 cm, 24 cm and 26 cm. Find its area and the longest altitude.
4. Two sides of a triangular field are 85 m and 154 m in length, and its perimeter is 324 cm. Find (i) the area of
the field, and (ii) the length of the perpendicular from the opposite vertex on the side measuring 154 cm.
5. The sides of a triangular field are 165 cm, 143 cm and 154 cm. Find the cost of ploughing it at 12 paise per
sq. m.
6. The base of an isosceles triangle measures 80 cm and its area is 360 cm2. Find the perimeter of the triangle.
1
7. The perimeter of an isosceles triangle is 42 cm and its base is 1 times each of the equal sides. Find (i) the
2
length of each side of the triangle, (ii) the area of the triangle, and (iii) the height of the triangle.
8. The perimeter of a right angle triangle is 40 cm. Its hypotenuse is 17 cm. Find the sides containing the right
angle. Also find the area of the triangle.
9. Find the area and perimeter of an isosceles right-angled triangle, each of whose equal sides measures 10 cm.

Take 2 = 1.414.
10. The area of a square field in 8 hectares. How long would a man take to cross its diagonal by walking at the
rate of 4 km per hour ?
11. A rhombus shaped field has green for 18 cows to graze. If each side of the rhombus is 30 m and its longer
diagonal is 48 m, how much area of grass field will each cow be getting ?

164
EXERCISE

(Objective DPP # 17.1)

Qus. 1 2 3 4 5

Ans. A A A D D

(Subjective DPP # 17.2)

1. 19.3 cm2 2. 216 cm, 24 cm 3. 120 cm2, 24 cm

4. () 2772 cm2 (ii) 36 cm2 5. Rs. 1219.68 6. 162 cm

7. (i) 12 cm, 12 cm, 18 cm (ii) 71.42 cm2 (iii) 7.94 cm 8. 8 cm, 15 cm & 60 cm2

9. 50 cm2, 34.14 cm 10. 6 minutes 11. 48 m2

165
SURFACE AREA
AND VOLUME
ML - 18

SOLID FIGURES
If any figure such as cuboids, which has three dimensions length, width and height are known as three
dimensional figures. Where are rectangle has only two dimensions i.e. length and width. Three dimensional
figures have volume in addition to areas of surface from which these solid figures are formed.
(a) Cuboids :
There are six faces (rectangular), eight vertices and twelve edges in a cuboids.
Total Surface Area (T.S.A.) : The area of surface from which cuboids is formed.
(i) Total Surface Area (T.S.A.) = 2 [× b + b × h + h × ]
(ii) Lateral Surface Area (L.S.A.) = 2[b × h + h ×  ]
(or Area of 4 walls) = 2h[  + b]
(iii) Volume of Cuboids = (Area of base) × height
= (  × b) × h

(iv) Length of diagonal =  2 + b2 + h 2


(b) Cube :
Cube ahs six faces. Each face is a square.
(i) T.S.A. = 2 [x . x + x . x + x . x]
= 2 [x2 + x2 + x2] = 2(332) = 6x2
(ii) L.S.A. = 2[x2 + x2] = 4x2
(iii) Volume = (Area of base) × Height
= (x2) × x = x3
(iv) Length of diagonal = x 3
(c) Cylinder :
Curved surface area of cylinder (C.S.A.) : It is the area of surface from which the cylinder is formed. When
we cut this cylinder, we will find a rectangle with length 2πr are height h units.
(i) C.S.A. of cylinder = ( 2πr ) × h = 2 πrh.
(ii) T.S.A = C.S.A. + circular top & bottom
= 2 πrh + ( πr 2 ) + ( πr 2 )
= 2 πrh + 2 πr 2
= 2 πr( h + r ) sq. units

166
(iii) Volume of cylinder = Area of base × height
= ( πr 2 ) × h
= πr 2 h cubic units
Hollow cylinder :
(i) C.S.A. of hollow cylinder = 2 π (R + r)h sq. units
(ii) T.S.A. of hollow cylinder = 2 π (R + r)h + π (R2 - r2)
= π (R + r) [2h + R - r] sq. units
(iii) Volume of hollow cylinder = π (R2 - r2)h cubic units
Where, r = inner radius of cylinder
R = outer radius of cylinder
h = height of the cylinder
(d) Cone :
(i) C.S.A. of cone = π
(ii) T.S.A . of cone = C.S.A. + Base area
= πr + πr 2
= πr( + r)
1 2
(iii) Volume of cone = πr h
3
Where, h = height
r = radius of base
 = slant height
(e) Sphere :
(i) T.S.A. of sphere = 4πr 2
4 3
(ii) Volume of sphere = πr
3
(f) Hemisphere :
(i) C.S.A. = 2 πr 2
(ii) T.S.A. = C.S.A. + other area
= 2 πr 2 + πr 2
= 3πr 2
2 3
(iii) Volume = πr
3
(g) Hollow Hemisphere :
(i) C.S.A. = 2 π (R2 + r2)
(ii) T.S.A = 2 π (R2 + r2 ) + π (R2 - r2)
2
(iii) Volume = π (R3 - r3)
3

167
ILLUSTRATIONS :
Ex.1 Three equal cubes are placed adjacently in a row. Find the ratio of the total surface area of the new cuboids
to that of the sum of the surface areas of three cubes.
Sol. Let the side of each of the three equal cubes be a cm.
Then surface area of one cube = 6a2 cm2
∴ Sum of the surface areas of three cubes = 3 × 6a2 = 18a2 cm2
For new cuboids
length () = 3a cm
breadth (b) = a cm
height (h) = a cm
∴ Total surface are of the new cuboids = 2(  × b + b × h + h =  )
= 2[3a × a + a × a + a × 3a]
= 2[3a 2 + a2 + 3a2] = 14a2 cm.2
Total surface area of the new cuboid
∴ Required ratio =
Sum of the surace areas of three cubes
14a 2 7
= = =7:9 Ans.
18a 2 9
Ex.2 A class room is 7 m long, 6.5 m wide and 4 m high. It has one door 3 m × 1.4 m and three windows each
measuring 2 m × 1 m The interior walls are to be colour-washed. The contractor charges Rs. 15 per sq. m.
Find the cost of colour washing.
Sol.  = 7m, n = 6.5 m and h = 4 m
∴ Area of the room = 2(  + b)h = 2(7 + 6.5) 4 = 108 m2
Area of door = 3 × 1.4 = 4.2 m2
Area of one window = 3 × 2 = 6 m2
∴ Area of 3 windows = 3 × 2 = 6 m2
∴ Area of the walls of the room to be colour washed = 108 - (4.2 + 6)
= 108 - 10.2 = 97.8 m2
∴ Cost of colour washing @ Rs. 15 per square metre = Rs. 97.8 × 15 = Rs. 1467. Ans.
1
Ex.3 A cylindrical vessel, without lid, has to be tin coated including both of its sides. If the radius of its base is
2
m and its height is 1.4 m, calculate the cost of tin-coating at the rate of Rs. 50 per 1000 cm2.
1
Sol. Radius of the base (r) = m
2
1
= × 100 cm = 50 cm
2
Height (h) = 1.4 m
= 1.4 × 100 cm
= 140 cm.
Surface area of to tin-coated = 2 ( 2 πr + πr 2 )
= 2[2 × 3.14 × 50 × 140 + 3.14 × (50)2]
= 2[43960 + 7850] = 2(51810) = 103620 cm2
∴ Cost of tin-coating at the rate or Rs. 50 per 1000 cm2
50
= × 103620 = Rs 5181. Ans.
1000

168
Ex.4 The diameter of a roller 120 cm long is 84 cm. If its takes 500 complete revolutions to level a playground
22
determine the cost of leveling at the rate of Rs. 25 per square metre. (Use π = )
7
Sol. 2r = 84 cm
84
∴ r= cm = 42 cm
2
h = 120 cm
Area of the playground leveled in one complete revolution = 2 πrh
22
=2× × 42 × 120 × 31680 cm2
7
∴ Area of the playground = 31680 × 500 cm2
31680 × 500 2
= m = 1584 m2
100 × 100
∴ Cost of leveling @ Rs 25 per square metre = Rs 1584 × 25 = 39600. Ans.
Ex.5 How many metres of cloth of 1.1 m width will be required to make a conical tent whose vertical height is 12
m and base radius is 16 m ? Find also the cost of the cloth used at the rate of Rs 14 per metre.
Sol. h = 12 m
r = 16 m

∴  = r2 + h 2

= (16)2 + (12 )2 = 256 + 144

= 400 = 20 m
22 7040 2
∴ Curved surface area = πr = × 16 × 20 = m
7 7
Width of cloth = 1.1 m
7040 /7 70400 6400
∴ Length of cloth = = = m
1.1 77 7
6400
∴ Cost of the cloth used @ Rs 14 per metre = Rs × 14 = Rs 12800 Ans.
7
Ex.6 The surface area of a sphere of radius 5 cm is five times the area of the curved surface of cone of radius 4
cm. Find the height of the cone.
Sol. Surface area of sphere of radius 4 cm = π (4)  cm2 when  cm is the slant height of the cone.
According to the question,
4π( 5)2 = 5[ π( 4) ]

⇒  = 5 cm ⇒ r2 + h2 = 5
⇒ r2 + h2 = 25 ⇒ (4)2 + h2 = 25
2
⇒ 16 + h = 25 ⇒ h2 = 9
⇒ h=3
Hence the height of the cone is 3 cm. Ans.

169
Ex.7 The dimensions of a cinema hall are 100 m, 50 m and 18m. How many persons can sit in the hall, if each
required 150 m3 of air ?
Sol.  = 100 m
b = 50 m
h = 18 m
∴ Volume of the cinema hall = bh
= 100 × 50 × 18 = 90000 m3
Volume occupied by 1 person = 150 m3
Volume of the ball
∴ Number of persons who can sit in the hall =
Volume occupied by 1 person
90000
= = 600
150
Hence 600 persons can sit in the hall. Ans.
Ex.8 The outer measurements of a closed wooden box are 42 cm, 30 m and 27 cm. If the box is made of 1 cm
thick wood, determine the capacity of the box.
Sol. Outer dimensions
 = 42 cm
b = 30 cm
h = 27 cm
Thickness of wood = 1 cm
Inner dimensions
 = 42 - (1 + 1) = 40 cm
b = 30 - 1(1 + 1) = 28 cm
h = 27 - (1 + 1) = 25 cm
∴ Capacity of the box  × b × h
= 40 × 28 × 25 = 28000 cm2. Ans.
Ex.9 If v is the volume of a cuboids of dimensions a,b, and c and s is its surface area, then prove that
1 2  1 1 1
=  + + 
v aa b c
1 1
Sol. L.H.S. = = ....(i)
v abc
2  1 1 1
R.H.S. =  + + 
sa b c
2  bc + ca + ab 
=  
2( ab + bc + ca)  abc 
1
= ....(ii)
abc
1 2 1 1 1
from (i) and (ii) =  + + . Hence Proved.
v sa b c

170
Ex.10 The ratio of the volumes of the two cones is 4 : 5 and the ratio of the radii of their bases is 2 : 3. Find the
ratio of their vertical heights.
Sol. Let the radii of bases, vertically heights and volumes of the two cones be r1, h1, v1 and r2, h2, v2 respectively.
According to the question,
v1 4 r1 2
= ...(i) = ....(i)
v2 5 r 3

1 2
πr1 h 1
2 4
From (i), we have =
1 2
πr2 h 2 5
3
r12 h 1 4
⇒ =
r22 h 2 5
2
 r1  h 1 4
⇒  
r  h = 5
 2 2
2
2 h 4
⇒   = 1 =
3 h2 5
2
h1 4  3 
⇒ =   [Using (ii)]
h2 5  2 
h1 9
⇒ =
h2 5
Hence the ratio of their vertical height is 9 : 5. Ans.
Ex.11 If h, c and v be the height, curved surface and volume of a cone, show that 3 πvh 3 - c2h2 + 9v2 = 0.
Sol. Let the radius of the base and slant height of the cone be r and  respectively. The n ;

c = curved surface = πr = πr r 2 + h 2 ....(i)


1
v = volume = πr 2 h ...(ii)
3
2
1 1 
∴ 3πvh 3 − c 2 h 2 + 9 v 2 = 3π( πr 2 h )h 2 − π 2 r 2 (r 2 + h 2 )h 2 + 9 πr 2 h  [Using (i) and (ii)]
3  3 
= 2 r 2 h 4 − π2 r 2 h 2 − π2 r 2 h 4 + π2 r 4 h 2 = 0. Hence Proved.
Ex.12 How many balls, each of radius 1 cm, can be made from a solid sphere of lead of radius 8 cm ?
4
Sol. Volume of the spherical ball of radius 8 cm = π × 83 c3
3
4
Also, volume of each smaller spherical ball of radius 1 cm = cm 3 .
3π × 13
Let n be the number of smaller balls that can be made. Then, the volume of the larger ball is equal to the
sum of all the volumes of n smaller balls.
4 4
Hence, π × n = π × 83
3 3
⇒ n = 83 = 512
Hence, the required number of balls = 512. Ans.

171
Ex.13 By melting a solid cylindrical metal, a few conical materials are to be made. If three times the radius of the
cone is equal to twice the radius of the cylinder an the ratio of the height of the cylinder and the height of
the cone is 4 : 3, find the number of cones which can be made.
Sol. Let R be the radius and H be the height of the cylinder and let r and h be the radius and height of the cone
respectively. Then,
3r = 2R
And H;h=4:3 ....(i)
H 4
⇒ =
h 3
⇒ 3H = 4h ....(ii)
Let n be the required number of cones which can be made from the materials of the cylinder. Then, the
volume of the cylinder will be equal to the sum of the volumes of n cones. Hence, we have
n 2
πR 2 H = πr h
3
⇒ 3R2H = nr2h
9r 2 4 h
2 3× ×
3R H 4 3 3r 4h
⇒ n= = [ ∵ From (i) and (ii), R= and H = ]
r2h 2
r h 2 3
3×9× 4
= =9
3× 4
Hence, the required number of cones is 9. Ans.
Ex.14 Water flows at the rate of 10 per minute through a cylindrical pipe having its diameter as 5 mm. How much
time will it take to fill a conical vessel whose diameter of the base is 40 cm and depth 24 cm ?
5 1
Sol. Diameter of the pipe = 5 mm = cm = cm.
10 2
1 1 1
∴ Radius of the pipe = × cm = cm.
2 2 4
In 1 minute, the length of the water column in the cylindrical pipe = 10 m = 1000 cm.
1 1
∴ Volume of water that flows out of the pipe in 1 minute = π × × × 1000 cm3.
4 4
1
Also, volume of the cone = × × π × 20 × 20 × 24 cm3.
3
1 1 1 
Hence, the time needed to fill up this conical vessel =  π × 20 × 20 × 24 ÷ π × × × 1000  minutes
3 4 4 
 20 × 20 × 24 4 × 4  4 × 24 × 16 256
= × = minutes = minutes = 51.2 minutes.
 3 100  30 5
Hence, the required time is 51.2 minutes. Ans.

172
EXERCISE

OBJECTIVE DPP # 18.1

1. The height of a conical tent at the centre is 5m. The distance of any point on its circular base from the top of
the tent is 13m. The area of the slant surface is :
(A) 144 π sq m (B) 130 π sq m (C) 156 π sq m (D) 169 π sq m
2. A rectangular sheet of paper 22 m long and 12 cm broad can be curved to form the lateral surface of a right
22
circular cylinder in two ways. Taking π = , the difference between the volumes of the two cylinders thus
7
formed is :
(A) 200 c.c. (B) 210 c.c. (C) 250 c.c. (D) 252 c.c.
3
3. The percentage increase in the surface area of a cube when each side is increased to times the original
2
length is
(A) 225 (B) 200 (C) 175 (D) 125
2
4. A cord in the form of a square enclose the area ‘S’ cm . if the same cord is bent into the form of a circle, then
the area of the circle is
πS 2 S 4S
(A) (B) 4πS 2 (C) (D)
4 4π π
5. If ‘I’, ‘b’ and ‘h’ if a cuboids are increased, decreased and increased by 1%, 3% and 2% respectively, then the
volume of the cuboids
(A) increase
(B) decrease
(C) increase or decreases depending on original dimensions
(D) can’t be calculated with given data
6. The radius and height of a cone are each increased by 20%, then the volume of the cone is increased by
(A) 20% (B) 40% (C) 60% (D) 72.8%
7. There is a cylinder circumscribing the hemisphere such that their bases are common. The ratio of their
volume is
(A) 1 : 3 (B) 1 : 2 (C) 2 : 3 (D) 3 : 4
8. Consider a hollow cylinder of inner radius r and thickness of wall t and length  . The volume of the above
cylinder is given by
 t 
(A) 2 π( r 2 −  2 ) (B) 2 πr t + 1  (C) 2 π(r 2 + t 2 ) (D) 2 πr( r + t )
 2 r 
9. A cone and a cylinder have the same base area. They also have the same curved surface area. If the height
of the cylinder is 3m, then the slant height of the cone (in m) is
(A) 3 (B) 4 (C) 6 (D) 7
10. A sphere of radius 3 cm is dropped into a cylindrical vessel of radius 4 cm. If the sphere is submerged
completely, then the height (in cm) to which the water rises, is
(A) 2.35 (B) 2.30 (C) 2.25 (D) 2.15

173
SUBJECTIVE DPP # 18.2

1. The whole surface of a rectangular lock is 846 cm2. Find the length, breadth and height, if these dimensions
are in the ratio 5 : 4 : 3.
2. An open box is made of wood 3 cm thick. its external length, breadth and height are 1.48 m, 1.16 m and 8.3
dm. Find the cost of painting the inner surface at Rs 5 per m2.
1 1
3. A room 8 m long 6 m board and 3 m high has two windows 1 m × 1 m and a door 2 m × 1 m. Find the
2 2
cost of papering the walls will paper 50 cm wide at Rs. 40 per metre.
1
4. 50 circular plates, each of radius 7 cm and thickness cm, are placed one above the other to form a solid
2
right circular cylinder. Find the total surface area.
5. A tent in the shape of a right circular cylinder surmounted by a right circular cone. The heights of the
cylindrical and the conical parts are 40 m and 21 m respectively. If the base diameter of the tent is 56 m, find
the area of the required canvas to make this tent if 20% of the area is consumed in folding and sewing.
6. A toy is in the form of a right circular cylinder closed at one end and with a hemisphere on the other end.
The height and the radius of the base are 15 cm and 6 cm respectively. The radius of the hemisphere are
cylinder are same. Calculate the total surface area and the volume of the toy. if the toy is painted at the rate
of Rs. 2.50 per 10 cm2, find the cost of painting the toy.
7. An iron pillar has some portion in the form of a right circular cylinder an remaining in the form of a right
circular cone. The radius of the base of each of the cone and the cylinder is 8 cm. The cylindrical portion is
240 cm high and the conical part is 36 cm high. Find the weight of the pillar, if one cubic cm of iron weights
7.8 g.
8. A solid metallic sphere of diameter 28 is melted and recasted into a number of smaller cones, each of
2
diameter 4 cm and height 3 cm. Find the number of cones so formed.
3
ANSWER KEY

(Objective DPP # 18.1)

Qus. 1 2 3 4 5 6 7 8 9 10

Ans. C B D D B D C B C C

(Subjective DPP # 18.2)

1. 15 cm, 12 cm, 9 cm 2. Rs. 27.97 3. Rs. 62.40


2
4. 1408 cm
5. Total surface area = 12144 cm2
6. Surface area ≈ 678.86 cm2, Volume ≈ 1470.86 cm3, Cost ≈ Rs. 170 7. 395.37 kg.
8. 672 cones.

174
STATISTICS

ML - 19

INTRODUCTION
The branch of science known as Statistics has been used in India from ancient times. Statistics deals with
collection of numerical facts i.e., data, their classification & tabulation and their interpretation. In statistics
we shall try to study, in detail about collection, classification and tabulation of such data.
(a) Importance of Data :
Expressing facts with the helps of data is of great importance in our day-today life. For example, instead of
saying that India has a large population it is more appropriate to say that the population of India, based on
the census of 2000 is more than one billion.
(b) Collection of Data :
On the basis of methods of collection, data can be divided into two categories :
(i) Primary data : Data which are collected for the first time by the statistical investigator or with help of his
workers is called primary data. As example if an investigator wants to study the condition of the workers
working in a factory then fro this he collects some data like their monthly income, expenditure, number or
brother, sisters, etc.
(ii) Secondary data : These are the data already collected by a person or a society and these may be in
published or unpublished form. These data should be carefully used. These are generally obtained from the
following two sources.
(A) Published sources
(B) unpublished sources
(c) Classification of Data :
When the data is complied in the same form and order in which it is collected, it is known as Raw Data, It is
also Crude Data. For example, the marks obtained by 20 students of class X in English out of 10 marks are
as follows :
7, 4, 9, 5, 8, 9, 6. 7, 9, 2,
0 3, 7, 6, 2, 1, 9, 8, 3, 8,
(i) Geographical basis : Here, the data is classified on the basis of place or region. For example the
production of food grains of different state is shown in the following table :

175
S.No. State Production (in Tons)
1 Andhdra Pradesh 9690
2 Bihar 8074

3 Haryana 10065

4 Pubjab 17065

5 Uttar Pradesh 28095


(ii) Chronological classification : If data’s classification is based on hour, day, week and month or year,
then it is called chronological classification, For example, the population of India in different year is shown
in following table :

S.No Year Production (in Crores)


1 1951 46.1

2 1961 53.9
3 1971 61.8

4 1981 68.5

5 1991 88.4

6 2001 100.01
(iii) Qualitative basis : When the data are classified into different groups on the basis of their descriptive
qualities and properties, such a classification is known as descriptive or qualitative classification. Since the
attributes can not be measured directly, they are counted on the basis of presence or absence of qualities.
For example intelligence, literacy, unemployment, honesty etc. The following table shows classification on
the basis of sex and employment.
Table Population (in lacs)
Gender → Male Female
Position of Employment ↓
Employed 16.2 13.7
Unemployed 26.4 24.8
Total 42.6 38.5
(iv) Quantitative basis : if facts are such that they can be measured physically e.g. marks obtained height,
weight, age, income, expenditure etc. Such facts are known as variable values. If such facts are kept into
classes then it is called classification according to quantitative or class intervals.

Marks obtained 10-20 20-30 30-40 40-50


No. of students 7 9 15 6

DEFINITIONS
(i) Variate : The numerical quantify whose value varies in objective is called a variate, generally a variate is
represented by x. There are two types of variate.
(A) Discrete variate : its magnitude is fixed. For example, the number of teacher in different branches of a
institute are 30, 35, 40 etc.
(B) Continuous variate : is magnitude is not fixed. It is expressed in groups like 10 - 20, 20 - 30, .... etc.
(ii) Rage : The difference of the maximum and the minimum values of the variable x is called range.
(iii) Class frequency : In each class the number of times a data is repeated in known as its class frequency.

176
Range
(iv) Class Interval =
Number of classes
It is generally denoted by h or i.
(v) Class limits : The lowest and the highest value of the class are known as lower and upper limited
restively of that class.
(vi) Class mark : The average of the lower and the upper limits of a class is called the mid value or the class
mark of that class. It is generally denoted by x.
 h 4
If x be the mid value and h be the class interval, then the class limits are  x − ,× +  .
 2 2
Ex.1 The mid values of a distribution are 54, 64, 74, 84 and 94. Find the class interval and class limits.
Sol. The class interval is the difference of two consecutive class marks, therefore class interval (h) = 64- 54 = 10.
Here the mid values are given and the class interval is 10.
So class limits are
10 10
For 1st class 54 − to 54 + or 49 to 59
2 2
10 10
For 2nd class 64 − to 64 + or 59 to 69
2 2
10 10
For 3rd class 74 − to 74 + or 69 to 79
2 2
10 10
For 4th class 84 − to 84 + or 79 to 89
2 2
10 10
For 5th class 94 − to 94 + or 89 to 99
2 2
Therefore class limits are 49 - 59, 59 - 69, 79 - 89, and 89 - 99.
FREQUENCY DISTRIBUTION
The marks scored by 30 students of IX class, of a school in the first test of Mathematics our of 50 marks are
as follows :

6 32 10 17 22 28 0 48 6 22
32 6 36 26 48 10 32 48 28 22
22 22 28 26 17 36 10 22 28 0

The number of times a mark is repeated is called its frequency. It is denoted by f.

Marks Taly mark Frequency Marks Tally mark Frequency


obtained obtained
0 II 2 26 II 2
6 III 3 28 IIII 4
10 III 3 32 III 3
17 II 2 36 II 2
22 IIII I 6 48 III 3

Above type of frequency distribution is called ungrouped frequency distribution. Although this
representation of data is shorter than representation of raw data, but from the angle of comparison and
analysis it is quite bit. So to reduce the frequency distribution, it can be classified into groups is following
ways and it is called grouped frequency distribution.

177
Class Frequency
0-10 8
11-20 2
21-30 12
31-40 5
41-50 3

(a) Kinds of Frequency Distribution :


Statistical methods like comparison, decision taken etc. depends of frequency distribution. Frequency
distribution are of three types.
(i) Individual frequency distribution : Here each item or original price of unit is written separately. In n
this category, frequency of each variable is one.
Ex.2 Total marks obtained by 10 students in a class.
S.No. 1 2 3 4 5 6 7 8 9 10
Marks 46 18 79 12 97 80 5 27 67 54
obtained
(ii) Discrete frequency distribution : When number of terms is large and variable are discrete, i.e., variate
can accept some particular values only under finite limits and is repeated then its called discrete frequency
distribution. For example the wages of employees and their numbers is shown in following table.
Monthly wages No. Of employees
4000 10
6000 8
8000 5
11000 7
20000 2
25000 1
The above table shows ungrouped frequency distribution the same facts can be written in grouped
frequency as follows :
Monthly wages No. of employees

0-10,000 23

11,000-20,000 9

21,000-30,000 1

NOTE :
If variable is repeated in individual distribution then it can be converted into discrete frequency
distribution.
(iii) Continuous frequency distribution : When number of terms is large and variate is continuous. i.e.,
variate can accept all values under finite limits and they are repeated then it is called continuous frequency
distribution. For example age of students in a school is shown in the following table :
Age (in year) Class No. of students
Less than 5 year 0-5 72
Between 5 and 10 y ear 5-10 103
Between 10 and 15 year 10-15 50
Between 15 and 20 year 15-20 25

178
NOTE :
Continuous frequency distribution is generally represented in form of grouped frequency distribution and
variate is continuous in i, so 0 - 5, 6 - 10, 11 - 15, 16 - 20 types of classes can not be made here. If such classes
are made in the table then students of age 5 to 6 year or 10 to 11 year or 15 to 16 years can not be classified.
if such type of classes are given then they should be made continuous by following methods. Half of the
difference between classes should be added to the upper limit of lower class and subtracted from lower
limit o upper class. Thus the classes 0 - 5.5, 5.5 - 10.5, 10.5 - 15.5, 15.5 - 19.5 are obtained which are
continuous.
Classes can be made mainly by two methods :
(i) Exclusive series : In this method upper limit of the previous class and lower limit of the next class is
same. In this method the term of upper limit in a class is not considered in the same class, it is considered in
the next class.
(ii) Inclusive series : In this method value of upper and lower limit are both contained in same class. In this
method the upper limit of class and lower limit of other class are not same. Some time the value is not a
whole number, it is a fraction or in decimals and lies in between the two intervals then in such situation the
class interval can be constructed as follows
A B
Class Frequency Or Class Frequency
0-9 4 0-9.99 4
10-19 7 10-19.99 7
20-29 6 20-29.99 6
30-39 3 30-39.99 3
40-49 3 40-49.99 3
CUMULIVE FREQUENCY
(i) Discrete frequency distribution : From the table of discrete frequency distribution, it can be identified
that number of employees whose monthly income is 4000 or how many employees of monthly income 1100
are there. But if we want to know how many employees whose monthly income is upto 11000, then we
should add 10 + 8 + 5 7 i.e., number of employees whose monthly income is upto 11000 is 30. Here we add
all previous frequency and get cumulative frequency. If will be more clear from the following table
Class Frequency (f) Cumulative frequency (cf) Explanation
4000 10 10 10 = 1 0
6000 8 18 10 + 8
8000 5 23 18 + 5
11000 7 30 23 + 7
20000 2 32 30 + 2
25000 1 33 32 + 1
(ii) Continuous frequency distribution : In the previous page we obtained cumulative frequency for
discrete series. Similarly cumulative frequency table can be made from continuous frequency distribution
also. For example, for table :
Monthly income No. of employee Cumulative Explanation
Variate (x) Frequency (f) Frequency (cf)
0–5 72 72 72 = 72
5 – 10 103 175 72 + 103 = 175
10 – 15 50 225 175 + 50 = 225
15 - 20 25 250 225 + 25 = 250

179
Above table can also be written as follows :
Clas Cumulative Frequency
Less than 5 72
Less than 10 175

Less than 15 225

Less than 20 250

From this table the number of students of age less than the upper limit of a class, i.e., number of student
whose age is less than 5, 10, 15, 20 year can determined by merely seeing the table but if we need the
number students whose age is more than zero, more than 5, more than 10 or more than 15, then table
should be constructed as follows :
Class Frequency Age Cumulative frequency Explanation
0–5 72 0 and more 50 250 = 250
5 – 10 103 5 and more 78 250 – 72 = 178
10 - 15 50 10 and more 75 178 – 103 = 75
15 - 20 25 15 and more 25 75 – 50 = 25

GRAPHICAL REPRESANTATION OF DATA


(i) Bar graphs
(ii) Histograms
(iii) Frequency polygons
(iv) Frequency curves
(v) Cumulative frequency curves or Ogives.
(vi) Pie Diagrams
(a) Bar Graphs :

Ex.3 A family with monthly income of Rs. 20,000 had planned the following expenditure per month under
various heads: Draw bar graph for the data given below :
Heads Expenditure (in Rs. 1000)
Grocery 4
Rent 5
Education of children 5
Medicine 2
Fuel 2
Entertainment 1
Miscellaneous 1
Y
Sol.

180
Histogram : Histogram is rectangular representation of grouped and continuous frequency distribution in
which class intervals are taken as base and height of rectangles are proportional to corresponding
frequencies. To raw the histogram class intervals are marked along x-axis on a suitable scale. Frequencies
are marked along y-axis on a suitable scale, such that the areas of drawn rectangles are proportional to
corresponding frequencies.
Now we shall study construction of histograms related with four different kinds of frequency distributions.
(i) When frequency distribution is grouped and continuous and class intervals are also equal.
(ii) When frequency distribution is grouped and continuous but class interval are not equal.
(iii) When frequency distribution is grouped but not continuous.
(iv) When frequency distribution is ungrouped and middle points of the distribution are given.
Now we try to make the above facts clear with some examples.
Ex.4 Draw a histogram of the following frequency distribution.

Clas (Age in year) 0–5 5 – 10 10 – 15 15 – 20


No. of students 72 103 50 25

Sol. Here frequency distribution is grouped and continuous and class intervals are also equal. So mark the class
intervals on the x-axis i.e., age in year (scale 1 cm = 5 year). Mark frequency i.e., number of students (scale 1
cm = 25 students) on they y-axis.
Now, since the number of students in class interval 0 - 5 is 72, so draw a parallel line to x-axis in front of
frequency to construct a rectangle on class interval 0 - 5. Repeating this procedure construct rectangle A, B,
C and D.

Ex.5 The weekly wages of workers of a factory are given in the following table. Draw histogram for it.

Weekly wages 1000 - 2000 2000 – 2500 2500 – 3000 3000 – 5000 5000 – 5500

No. of workers 26 30 20 16 1

Sol. Here frequency distribution is grouped and continuous but class intervals are not same. Under such
circumstances the following method is used to find heights of rectangle so that heights are proportional to
frequencies.
(i) Write interval (h) of the least interval, here h = 500.
(ii) Redefine the frequencies of classes by the using the following formula.
h
Redefined frequency of class = × frequency of class interval.
clss int erval
So here the redefined frequency table is obtained as follows :

181
Weekly wages (in Rs.) No. of workers Redefined of workers
1000 – 2000 26 500
× 26 = 13
1000
2000 – 2500 30 500
× 30 = 30
500
2500 – 3000 20 500
× 20 = 20
500
3000 – 5000 16 500
× 16 = 4
2000
5000 – 5500 1 500
×1 = 1
500

Now mark class interval on x-axis (scale 1 cm = 500) and no. of workers on y-axis (scale 1 cm = 5). On the
basis of redefined frequency distribution construct rectangle A, B, C D and E.

This is the required histogram of the given frequency distribution


(a) Difference Between Bar Graph and Histogram
(i) In histogram there is no gap in between consecutive rectangle as in bar graph.
(ii) The width of the bar is significant in histogram. In bar graph, width is not important at all.
(iii) In histogram the areas of rectangles are proportional to the frequency, however if the class size of the
frequencies are equal then height of the rectangle are proportional to the frequencies.
Frequency polygon : A frequency polygon is also a form a graphical representation of frequency
distribution. Frequency polygon can be constructed in two ways :
(i) With the help of histogram
(ii) Without the help of histogram
(A) Following procedure is useful to draw a frequency polygon with the help of histogram.
(a) Construct the histogram for the given frequency distribution.
(b) Find the middle point of each upper horizontal line of the rectangle.
(c) Join these middle points of the successive rectangle by straight lines.
(d) Join the middle point of the initial rectangle with the middle point of the previous expected class
interval on the x-axis.

182
Ex.6 For the following frequency distribution, draw a histogram and construct a frequency polygon with it.

Class 20 – 30 30 – 40 40 – 50 50 – 60 60 - 70
Frequency 8 12 17 9 4

Sol. The given frequency distribution is grouped and continuous, so we construct a histogram by the method
given earlier. Join the middle points P,Q,R,S,T of upper horizontal line of each rectangles A,B,C,D,E by
straight lines.

Ex.7 Draw a frequency polygon of the following frequency distribution table.

Class 0 – 10 10 – 20 20 – 30 30 – 40 40 – 50 50 – 60 60 – 70 70 – 80 80 – 90 90 – 100
Frequency 8 10 6 7 9 8 8 6 3 4

Sol. Given frequency distribution is grouped and continuous. So we construct a histogram by using earlier
method. Join the middle points of P,Q,R,S,T,U,V, W,X, Y of upper horizontal lines of each rectangle
A,B,C,D,E,F,G,H,I,J by straight line in successions.

Ex.8 Draw a frequency polygon of the following frequency distribution.

Age (in years) 0 – 10 10 – 20 20 – 30 30 – 40 40 – 50 50 – 60


Frequency 15 12 10 4 11 14

183
Sol. Here frequency distribution is grouped and continuous so here we obtain following table on the basis of
class.

Age (in years) 0 – 10 10 – 20 20 – 20 30 – 40 40 – 50 50 – 60


Class mark 5 15 25 35 45 55
Frequency 15 12 10 4 11 14

Now taking suitable scale on graph mark the points (5, 15), (15,12), (25, 10) (35, 4), (45, 11), (55, 14). Since
age can not be negative so instead of joining corner (5,15) with middle point of zero frequency of earlier
assumed class, we draw vertical line from the lower limit of this class i.e., 0 and point of half frequency of
this lie i.e., (0, 7.5) is joined by the end point. Joint the last point (55, 14) with the points of zero frequency of
the next assumed class i.e, with (65, 0).

MEASURES OF CENTRAL TENDENCY

The commonly used measure of central tendency are -


(i) Mean
(ii) Median
(iii) Mode
(a) Mean :
The mean of a number of observation is the sum of the values of all the observations divided by the total
number of observations. It is denoted by the symbol x , read as x bar.
(i) properties of mean :
(a) If a constant real number ‘a’ is added to each of the observation than new mean will be x + a .
(b) If a constant real number ‘a’ is subtracted from each of the observation then new mean will be x − a
(c) If a constant real number ‘a’ is multiplied with each of the observation then new mean will be x
x
(d) If each of the observation is dived by a constant no ‘a’ then new mean will be .
a
(ii) Mean of ungrouped data : If x1, x2, x3,..... , xn are then n values (or observations) then A.M. (Arithmetic
mean) is

184
n

x 1 + x 1 + ..... + x n
∑x
i −1
i
x= =
n n
n
n x = Sum of observation = = ∑x
i −1
i

i.e. product of means & no. of items given sum of observation.

Ex.9 Find the mean of the factors of 10

Sol. factors of 10 are 1,2,5 & 10.

1 + 2 + 5 + 10 18
x= = = 4 .5
4 4

Ex.10 If the mean of 6,4,7 P and 10 is 8 find P.

6 + 4 + 7 + P + 10
Sol. 8= ⇒ P = 13 ⇒ P = 13
5

(iii) Method for Mean of ungrouped frequency distribution.

xi fi fix i
x1 f1 f1x1
x2 f2 f2x2
x3 f3 f3f3
. . . Then mean x =
∑f x
i i

. . . ∑f i
. . .
xn fn fnxn

∑fi = ∑f xi i =

(iv) Method for Mean of grouped frequency distribution.

Ex.11 (1) Direct Method : for finding mean

Marks No. of students fi mid values xi fixi


10 – 20 6 15 90
20 – 30 8 25 200
30 – 40 13 35 455 x=
∑f x
i i 1430
= 35.75
40 – 50 7 45 315 ∑f i 40
50 – 60 3 55 165
60 – 70 2 65 130
70 - 80 1 75 75

∑f 1 = 40 ∑f x
i i = 40

(v) Combined Mean :


n 1 x 1 + n 2 + x 2 + ...
x=
n 1 + n 2 + ....

185
(vi) Uses of Arithmetic Mean
(A) It is used for calculating average marks obtained by a student.
(B) It is extensively used in practical statistics.
(C) It is used to obtain estimates.
(D) It is used by businessman to find out profit per unit article, output per machine, average monthly
income and expenditure etc.
(b) Median :
Median of a distribution is the value of the variable which divides the distribution into two equal parts.
(i) Median or ungrouped data
(A) Arrange the data in ascending order.
(B) Count the no. of observations (Let there be ‘n’ observations)
th
 n +1
(C) If n is odd then median = value of   observation.
 2 
th th
n n 
(D) If n is even the median = value of mean of   observation and  + 1  observation.
2 2 
Ex.12 Find the median of the following values :
37, 31, 42, 43, 46, 25, 39, 45, 32
Sol. Arranging the data in ascending order, we have
25, 31, 32, 37, 39, 42, 43, 45, 46
Here the number of observations n = 9 (odd)
th
 9 + 1
∴ Median = Value of   observation
 2 
= Value of 5th observation
= 39.
Ex.13 The median of the observation 11, 12, 14, 18, x + 2, x + 4, 30, 32, 35, 41 arranged in ascending order is 24.
Find the value of x.
Sol. Here, the number of observations n = 10. Since n is even, therefore
th th
n n 
  conservation +  + 1  observation
 2  2 
Median =
2
5th observation + 6 th observation
⇒ 24 =
2
( x + 2 ) + ( x + 4)
⇒ 24 =
2
2x + 6
⇒ 24 = ⇒ 24 = x + 3 ⇒ x = 21.
2
Hence, x = 21

186
(ii) Uses of Median :
(A) Median is the only average to be used while dealing with qualitative data which cannot be measured
quantitatively but can be arranged in ascending or descending order or magnitude.
(B) It is used for determining the typical value in problems concerning wages, distribution of wealth etc.
(c) Mode :
(i) Mode or ungrouped data (By inspection only) : Arrange the data in an array and then count the
frequencies of each variate. The variate having maximum frequency is the mode.

Ex.13 Find the mode of the following array of an individual series of scores 7, , 10, 12, 12, 12, 11, 13, 13, 17.

Number 7 10 11 12 13 17

Frequency 2 1 1 3 2 1

∴ Mode is 12
(ii) Uses of Mode : Mode is the average to be used to find the ideal size, e.g., in business forecasting, in
manufacture of ready-made garments, shoes etc.
(c) Empirical Relation between Mode, Median & Mean :
Mode = 3 Median - 2 Mean

RANGE
The range is the difference between the highest and lowest scores of a distribution. It is the simplest
measure of dispersion. It gives a rough idea of dispersion. This measure is useful for ungrouped data.
(a) Coefficient of the Range :
h−
If  and h are the lowest and highest scores in a distribution then the coefficient of the Range =
h+

Ex.14 Find the range of the following distribution : 1, 3, 4, 7, 9, 10, 12, 13, 14, 16 and 19.
Sol.  = 1, h = 19
∴ Range = h -  = 19 - 1 = 18 Ans.

Ex.15 Find the range of the following frequency distribution :

Class – Interval Frequency


0–5 6
5 – 10 8
10 – 15 12
15 – 20 5
20 - 25 4

Sol. The range is the difference between the mid value of the least class-interval and the greatest class interval.
0+5
Mid value of least class interval = = 2 .5
2
20 + 25
Mid value of greatest class interval = = 22.5
2
∴ Range = 22.5 - 2.5 = 20 Ans.

187
EXERCISE

OBJECTIVE DPP # 19.1

1. The median of following series is 520, 20, 340, 190, 35, 800, 1210, 50, 80
(A) 1210 (B) 520 (C) 190 (D) 35
2. If the arithmetic mean of 5, 7, 9, x is 9 then the value of x is
(A) 11 (B) 15 (C) 18 (D) 16
3. The mode of the distribution 3,5,7,4,2,1,4,3,4 is
(A) 7 (B) 4 (C) 3 (D) 1
4. If the mean and median of a set of numbers are 8.9 and 9 respectively, then the mode will be
(A) 7.2 (B) 8.2 (C) 9.2 (D) 10.2
5. A student got marks in 5 subjects in a monthly test is given below :
(A) 2,3,4,5,6, in these obtained marks, 4 is the
(A) Mean and median (B) Median but no mean (C) Mean but no median (D) Mode
6. What is the mode from the following table :

Marks obtained 3 1 23 33 43

Frequency (f) 7 11 15 8 3

(A) 13 (B) 43 (C) 33 (D) 23


7. If the class intervals in a frequency distribution are (72 - 73.9), (74 - 75.9), (76 - 77.9), (78 - 79.9) etc., the mid-
point of the class (74 - 75.9) is
(A) 74.50 (B) 74.90 (C) 74.95 (D) 75.00
8. Which one of the following is not correct -
(A) Statistics is liable to be misused
(B) The data collected by the investigator to be used by himself are called primary data
(C) Statistical laws are exact
(D) Statistics do not take into account of individual cases
9. If the first five elements of a se replaced by (x1 + 5), where i = 1,2,3,...5 and the next five elements are
replaced by (xi - 5), where = 6 ..... 10 then the mean will change by
(A) 25 (B) 10 (C) 5 (D) 0
10. The following numbers are given 61, 62, 63, 61, 63, 64, 64, 60, 65, 63, 64, 65, 66, 64. The difference between
their mean and median is
(A) 0.4 (B) 0.3 (C) 0.2 (D) 0.1
n
11. The value of ∑(x
i=1
i − x) where x is the arithmetic mean of x1 is

(A) 1 (B) n x (C) 0 (D) None of these

188
12. The average of 15 numbers is 18. The average of first 8 is 19 and that last 8 is 17, then the 8th number is
(A) 15 (B) 16 (C) 18 (D) 20
13. In an examination, 10 students scores the following marks in Mathematics 35, 19, 28, 32, 63, 02, 47, 31, 13,
98. It rage is
(A) 96 (B) 02 (C) 98 (D) 50
Direction : question 15 is based on the histogram given in the adjacent figure.

14. The percentage of students in science faculty in 1990-91 is :


(A) 26.9% (B) 27.8% (C) 29.6% (D) 30.2%
15. For the scores 8,6,10,12,1,5,6 and 6 the Arithmetic mean is
(A) 6.85 (B) 6.75 (C) 6.95 (D) 7
Direction : Each question from 16 to 18 is based on the histogram given in the adjacent figure.

16. What is the number of worker earning Rs. 300 to 350 ?


(A) 50 (B) 40 (C) 45 (D) 130
17. In which class interval of wages there is the least number of workers ?
(A) 400 - 450 (B) 350 - 400 (C) 250 - 300 (D) 200 - 250
18. What is the upper limit of the class-interval 200-250
(A) 200 (B) 250 (C) 225 (D) None of these

189
SUBJECTIVE DPP # 19.2

1. Find the mean of following data 13,17,16,14,11,13,10,16,11,18,.12,17.


2. Find the median of following data 38,70,48,34,42,55,63,46,54,44.
3. Find the mode of following data 2,2,6,5,4,3,4,5,7,9,4,5,3,1,10,4.
4. Find the median of :
(i) 5,30,15,6,18,22,26,32,6,9,18
(ii) 92,88,62,53,55,59,60,61,85,89
(iii) 66,69,108,72,78,82,98,99,102,101
5. Find the value of pm if the median of following observations is 48.
14, 17, 33, 35, p-5, p + 7, 57, 63, 69, 80. The above observation are in ascending order.]
6. Find the missing frequencies of the following distribution if it is known that mean of the distribution is 50.
x: 10 30 50 70 90 Total
f : 17 f1 32 f2 19 120
7. Find the mean for following data.
Age (Years) 25 – 30 30 – 35 35 – 40 40 – 45 45 – 50 50 – 55
No. of teachers 30 23 20 14 10 3

8. Calculate the mean of the following frequency distribution :


Marks 20-30 30-40 40-50 50-60 60-70 70-80 80-90
No. of Students 3 6 13 15 14 5 4

9. The mean of a certain group of observations is 78. Find the resulting mean, if the value of each observation
is :
(i) increased by 2 (ii) decreased by 3 (iii) multiplied by 1.5
(iv) divided by 2 (v) increased by 30% (iv) diminished by 25%
10. Draw a histogram to represent the following data :

Class- 40-60 60-80 80-100 100-120 120-140 140-160 160-180 180-200


Interval
Frequency 20 40 30 50 30 20 10 40

11. Draw a bar-graph to represent the following

A B C D E F

60 70 55 40 90 50

ANSWER KEY

(Objective DPP 19.1)


Qus. 1 2 3 4 5 6 7 8 9 10
Ans. C B B C A D C C D B
Qus. 11 12 13 14 15 16 17 18
Ans. C C A C B A D B

1. 14 2. 47 3. 4 4. (i) 18 (ii) 61.5 (iii) 90


5. P = 47 6. 28.24 7. 35.5 8. 55.33
9. (i) 80 (ii) 75 (iii) 117 (iv) 39 (v) 101.4 (iv) 58.5

190
PROBABILITY

ML - 20

PROBABILITY

Theory of probability deals with measurement of uncertainty of the occurrence of same event or incident in terms of
percentage or ratio.
(i) Sample Space : Set of possible out comes.
(ii) Trial : Trial is an action which results in one of several outcomes.
(iii) An experiment : An experiment is any kind of activity such as throwing a die, tossing a coin, drawing a
card. outcome of an experiment. The different possibilities which can occur during an experiment.
e.g. on throwing a dice, 1 dot, 2 dots, 3 dots, 4 dots, 5 dots, 6 dots can occur.
(iv) An event : getting a ‘six’, in a throw of dice, getting a head, in a toss of a coin.
(v) A random experiment : Whenever we do some experiment at once.
(vi) Equally likely outcomes : there are equal uncertainty in getting 1 dot, 2 dots, 3 dots, 4 dots, 5 dots, 6
dots when we throw a single dice.
(vii) Probability of an event A: Written as P(A) in a random experiment and is defined as -
Number of outcomes in favour of A
P( A ) =
Total number of possible outcomes
(a) Important Properties :
(i) 0 ≤ P(A ) ≤ 1
(ii) P (not happening of (A) + P(happening of A) = 1
or P (A ) = P(A) = 1

∴ P (A ) = 1-P(A)
Number of favourable outcomes
Probability of the happening of A =
Total number possible outcomes
m
m+n
n
Probability of not happening of A (falling of A) =
m+n
where is for an event A can happen in m ways and fail in n ways all these ways being equally likely to
occur.

191
(b) Problems of Die :
(i) A die is thrown once. What is the probability of -
(A) Getting an even number in the throwing of a die, the total number of outcomes is 6.
Let be the event of getting an even number then there are three even numbers 2, 4, 6.
∴ number of favourable outcomes = 3.
no. of faourable outcomes 3 1
∴ P( A ) = = = .
total no. of outcomes 6 2
(B) Getting an odd number (A) total outcomes = 6, favourable outcomes = 3(1, 3, 5)
3 1
∴ P( A ) = =
6 2
6
(C) Getting a natural number P(A) = =1
6
 Fabourable cases 
(D) Getting a number which is multiple of 2 and 3 =  
 6 

(E) Getting a number ≥ 3(3,4,5,6)
4 2
P(A) = =
6 3
2 1
(F) Getting a number 5 or 6 (5 or 6) P(A) = =
6 3
5
(G) Getting a number ≤ 5 P(A) = (1,2,3,4,5)
6
(c) Problems Concerning Drawing a Card :
(i) A pack of 52 cards
(ii) Face cards (King, Queen, Jack)
Ex.1 A card is drawn from a well shuffled deck of 52 cards. Find the probability of
(i) A king.
(ii) A heart.
(iii) A seven of heart.
(iv) A jack, queen or a king.
(v) A two of heart or a two of diamond.
(vi) A face card.
(vii) A black card.
(viii) Neither a heart nor a king.
(ix) Neither an ace nor a king.

192
Sol. Total no. of outcomes = 52
(i) A king.
4 1
No. of kings = 4 (favorable cases) P (A ) = = .
42 13
13 1
(ii) A heart P (A ) = =
52 4
1
(iii) A seven of heart P (A ) =
52
12 3
(iv) A jack, queen or a king P (A ) = =
52 13
2 1
(v) A two of heat or a two of diamond. P (A ) = =
52 26
12 3
(vi) A face card P (A ) = =
52 13
26 1
(vii) A black card P (A ) = =
52 2
(viii) Neither a heart nor a king (13 heart + 4 king, but 1 common)
16 52 − 16 36 9
P (A ) = 1 − = = =
52 52 52 13
44 11
(ix) Neither an ace nor a king. P (A ) = =
52 13

Ex.2 Two coins are tossed simultaneously. Find the probability of getting
(i) two heads (ii) at least one head (iii) no head
∴ On tossing two coins simultaneously, all the possible outcomes are
HH, HT, TH, TT.
(i) The probability of getting two heads = P (HH)
Even of occurence of two heads 1
= =
Total number of possible outcomes 4

(ii) The probability of getting at least on head


Favourable outcomes 3
= =
Total no. of outcomes 4

1
(iii) The probability of getting no head P(TT) =
4

193
Ex.3 A bag contains 5 red balls, 8 white balls, 4 green balls and 7 black balls. If one ball is drawn at random, find
the probability that it is
(i) Black
(ii) Not red
(iii) Green
Sol. Number of red balls in the bag = 5
Number of white balls in the bag = 8
Number of green balls in the bag = 4
Number of black balls in the bag = 7
∴ Total number of balls in the bag = 5 + 8 + 4 + 7 = 24.
Drawing balls randomly are equally likely outcomes.
∴ Total number of possible outcomes = 24
Now,
(i) There are 7 black balls, hence the number of such favourable outcomes = 7
Number of favourable outcomes 7
∴ Probability of drawing a black ball = = Ans.
Total number of possible outcomes 24

(ii) There are 5 red balls, hence the number of such favourable outcomes = 5.
Numbe of favourable outcomes 5
∴ Probability of drawing a red ball = = Ans.
Total number of possible outcoes 24

5 19
∴ Probability of drawing not a red ball = P (Not Red ball) = 1 − = Ans.
24 24
(iii) There are 4 green balls.
∴ Number of such favourable outcomes = 4
Number of favourable outcomes 4 1
Probability of drawing a green ball = = = Ans.
Total number of possibl outcomes 24 6

Ex.4 A card is drawn from a well - shuffled deck of playing cards. Find the probability of drawing
(i) a face card (ii) a red face card
Sol. Random drawing of cards ensures equally likely outcomes
(i) Number of face cards (King, Queen and jack of each suits) = 4 × 3 = 12
Total number of cards in deck = 52
∴ Total number of possible outcomes = 52
12 3
P (drawing a face card) = =
52 13
(ii) Number of red face cards = 2 × 3 = 6
Number of favourable outcomes of drawing red face card = 6
6 3
P (drawing of red face red) = = Ans.
52 26

194
EXERCISE

OBJECTIVE DPP - 20.1

1. 3 Coins are tossed simultaneously. The probability of getting at least 2 heads is


3 3 3 1
(A) (B) (C) (D)
10 4 8 2
2. Two cards are drawn successively with replacement from a pack of 52 cards. The probability of drowsing
two aces is
1 1 1 4
(A) (B) (C) (D)
169 221 265 663
3. In a single throw of two dice, the probability of getting more than 7 is
7 7 5 5
(A) (B) (C) (D)
36 12 12 36

4. Two cards are drawn at random from a pack of 52 cards. The probability that both are the cards of space is
1 1 1
(A) (B) (C) (D) Nome of these
26 4 17

5. Two dice are thrown together. The probability that sum of the two numbers will be a multiple of 4 is
1 1 1 5
(A) (B) (C) (D)
9 3 4 9
6. If the odds in favour of an event be 3 : 5 then the probability of non-happening of the event is
3 5 3 5
(A) (B) (C) (D)
5 3 8 8

7. In a cricket match, a batswoman hits a boundary 6 times out of 30 balls she plays. Find the probability that
she did not hit a boundary.
(A) 0.8 (B) 0.6 (C) 0.5 (D) 0.2
8. If the three coins are simultaneously tossed again compute the probability of 2 heads coming up.
3 1 5 3
(A) (B) (C) (D)
8 4 8 4
9. A coin is tossed successively three times. The probability of getting one head or two heads is :
(A) 2/3 (B) 3/4 (C) 4/9 (D) 1/9
10. One card is drawn from a pack of 52 cards. What is the probability that the drawn card is either red or king:
(A) 15/26 (B) 1/2 (C) 7/13 (D) 17/32

SUBJECTIVE DPP - 20.2

1. Two dice are thrown together. Find the probability of getting a total of 9.
2. A coin and a dice are tossed simultaneously find the sample space.
3. A dice is thrown repeatedly until a six comes up. What is the sample space for this experiment.
4. On a simultaneous toss of three coins, find the probability of getting
(i) at least 2 heads
(ii) at most 2 heads
(iii) exactly 2 heads

195
5. Two dice are thrown simultaneously. Find the probability of getting
(i) an even number s the sum
(ii) the sum as a prime number
(iii) a doubled of even number
6. Three dice are thrown together. Find the probability of getting a total of a least 6.
7. Find the probability that a leap year selected at random will contain 53 Tuesday.
8. A coin is tossed 80 times with the following outcomes :
(i) head : 35
(ii) tail : 45
Find the probability of each event.
9. Two coins are tossed simultaneously 150 times and we get the following outcomes.
(a) No tail = 45
(b) One tail = 55
(c) Two tails = 50
Find the probability of each event.
10. In a cricket match a batsman hits a boundary 10 times out of 36 balls be play. Find the probability that he
did not hit the boundary.
11. In a cricket match a batsman hits a boundary 3 times in 3 over he play. Find the probability that the did not
hit the boundary.
12. A bag which contains 7 blue marbles, 4 black marbles and 9 white marbles. A marbles drawn at random
from the bag then what is the probability that the drawn marble is
(i) blue
(ii) white or black
13. The odds in favour of an event are 3 : 5 find the probability of occurrence of this event.
14. Cards marked with the numbers 2 to 101 are placed in a box and mixed thoroughly. One card is drawn
from this box, find the probability that the number on card is
(i) An even number
(ii) A number less than 14
(iii) A number which is a prefect square.
(iv) A prime number less than 20
15. An urn contains 6 oranges, 7 apples & 11 mango. A fruit is drawn at random, what is the probability of
drawing.
(i) An orange
(ii) Not apple
(iii) An apple or a mango
16. A card is drawn at random from a well shuffled desk of playing cards. Find the probability that the card
drawn is
(i) A card of spade or an ace
(ii) A red king
(iii) Neither a king nor a queen
(iv) Either a king or a queen

17. A box contains 19 balls bearing numbers 1,2,3..... 19. A ball is drawn at random from the box. Find the
probability that the number on the balls is
(i) A prime number
(ii) Divisible by 3 or 5
(iii) Neither divisible by 5 nor by 10
(iv) An even number

18. There are 30 cards of same size in a bag containing numbers 1 to 30. One card is taken out from the bag at
random. Find the probability that the number on the selected card is not divisible by 3.

196
EXERCISE
(Objective DPP # 20.1)

Qus. 1 2 3 4 5 6 7 8 9 10

Ans. D A C C C D A A B C

(Subjective DPP # 20.2)


1
1.
9

2. (H, 1) (H, 2) (H, 3) (H, 4) (H, 5) (T, 1) (T, 2) (T, 3) (T, 4) (T, 5) (T, 6)

3. {6, (1, 6) (2, 6) (3, 6) (4, 6) (5, 6) (1, 1, 6) (1, 2, 6).... }

1 7 3
4.  , , 
2 8 8

1 5 1   103  2
5.  , ,  6.   7.  
 2 12 12   108  7

7 9 3 11 1 13
8. (i ) (ii) 9. (a) (b) (c) 10.
16 16 10 30 3 18

3 7 13
11. 12. (i) (ii)
8 20 20

3 1 3 2 9 2
13. 14. (i) (ii) (iii) (iv)
8 2 25 25 100 25

1 17 3 4 1 11 2
15. (i) (ii) (iii) 16. (i) (ii) (iii) (iv)
4 24 4 13 26 13 13

8 8 16 9 2
17. , , , 18.
19 19 19 19 3

197
PROOF IN
MATHEMATICS

ML - 21
STATEMENT

is a sentence which is neither an order nor a question nor an exclamatory sentence.


A sentence or statement can be
(a) a true statement (b) a false statement (c) an ambiguous statement
Examples for true statement
(i) 1 + 3 = 4, 7 + 3 = 10. (ii) The number of days in a week is seven.
(iii) When 2x = 10, then x = 5. (iv) There are three sides in a triangle.
(v) New Delhi the capital of India.
Examples for false statement
(i) 2 + 7 = 5 is a false statement (ii) 9 × 2 = 15 is a false statement
(iii) 1 m = 1000 cm is a false statement
(iv) Patna is the capital of West Bengal is a false statement
(v) Sunday comes after Monday is a false statement
Examples for ambiguous statement
(i) The 7th of Mach falls on Monday. (ii) The sum of any two angles of a triangle is 1100.
(iii) Today is Friday.
Mathematically valid statement
Mathematically, a statement is valid or acceptable only if it is either always true or always false.
Deduction :
Deductive reasoning : To find the truth value of an unambiguous statement we use the deductive
reasoning. This is the main logical tool.

AXIOM CONJECTURE AND THEOREM


(a) Axiom : Axiom is a statement which is accepted as a true statement. An axiom does not require a proof.
Example :
(i) a = b, b = c ⇒ a =c
(ii) a > b, b > c ⇒ a>c
1 1
(iii) a=b ⇒ a= b
2 2

198
(b) Conjecture : It is a statement whose truth ness or falseness has not been established mathematically.
(c) Theorem : A theorem is a mathematical statement whose truth has been established logically.
Proof of a theorem
The main parts of a proof are as under.
(i) The Hypothesis (i.e. what is given)
(ii) The conclusion (i.e. what is to be proved)
(iii) Consists of successive mathematical statements derived logically from the previous statement or axiom
or hypothesis.
Ex.1 State whether the following statements are always true, always false or ambiguous, Justify your answer.
(i) There are 13 months in a year.
(ii) Diwali falls on Friday.
(iii) The temperature in Magadi is 260 C.
(iv) Dogs can fly.
(v) February has only 28 days.
Sol. (i) This statement is false because there are 12 months in a year.
(ii) This statement is always ambiguous because Diwali can fall on any day.
(iii) This statement is always ambiguous because it is not fixed.
(iv) This statement is always false.
(v) This is a false statement because February has 29 days in a leap year.
Ex.2 State whether the following statements are true or false. Give reasons for your answers.
(i) The sum of the interior angles of a quadrilateral is 3500.
(ii) For any real number x, x2 ≥ 0.
(iii) A rhombus is a parallelogram.
(iv) The sum of two even numbers is even.
(v) The sum of two odd numbers is odd.
Sol. (i) This statement is false because the sum of the interior angles of a quadrilateral is 3600.
(ii) This statement in always true. For example (-2)2 = 4, then we can say x2 ≥ 0 for any real number x.
(iii) This statement is always true.
(iv) This statement is always true. For example, 2 + 2 = 4 and 6 + 4 = 10.
(v) This statement is always false. For example, 3 + 5 = 7 and 3 + 9 = 10.
Ex.3 State whether the following statements are true or false :
(i) Opposite angles of a cyclic quadrilateral are supplementary.
(ii) Every odd number greater than 1 is prime.
(iii) Exterior angle of a cyclic quadrilateral is equal to the opposite angle.
(iv) For any real number x, 5x + x = 6x.
(v) For every real number x, x3 ≥ x.
(vi) An exterior angle is greater than each interior opposite angle.
Sol. (i) This statement is true.
(ii) This statement is false ; for example, 9 is not a prime number
(iii) This statement is true.
(iv) This statement is true.

199
3
1 1 1 1
(v) This statement is false, for example   = and is not greater than .
2 8 8 2
(vi) This statement is true.
Ex.4 Restate the following statements with appropriate condition so that they become true statements.
(i) Square of a real number is always greater than the number.
(ii) In a parallelogram the diagonals are equal.
(iii) There are four angles is a triangle.
Sol. (i) Square of a real number is always greater than the number when the magnitude of the number is
greater than one.
(ii) In a rectangle, the diagonals are equal.
(iii) There are three and only three angles in a triangle.
Ex.5 Restate the following statements with appropriate conditions, so that they become true statements.
(i) All prime numbers are odd. (ii) Two times a real numbers is always even.
(iii) For any x, 3x + 1 > 4. (iv) For any x, x3 ≥ 0.
(v) In an equilateral triangle the medians are also an angle bisector.
Ex.6 The sum of the angles of a triangle is 1800
Sol. Statement : The sum of the angles of a triangles is 1800
Given : A ∆ ABC
To Prove : ∠1 + ∠2 + ∠3 = 1800
Construction : Through A, draw a line DE parallel to BC.
Proof :

S.No. Statement Reason


1. DE ║ BC and AC is the transversal Alternative interior ∠s
∴ ∠1 = ∠4
2. Again DE ║ BC and AC is the Alternate interior ∠s
transversal ∴ ∠2 = ∠2 Adding. (1)
and (2)
3. ∠1 + ∠2 = ∠4 + ∠5 Adding the corresponding side of (1) & (2)
4. ∠1 + ∠2 + ∠3 = ∠4 + ∠5 + ∠3 Adding ∠3 on the sides.
5. But ∠4 + ∠5 + ∠3 = ∠DAE = 1800 ∠DAE is a straight line angle.
6. ∠1 + ∠2 + ∠3 = 1800 The sum of the angles of a triangle is 1800.

Ex.7 For each natural number, n( n + 1) is multiple of 2.


Sol. We have to prove that the product (n + 1) is divisible by 2.
Now we have two cases. Either is even or odd. Let us examine each case. Suppose n is even. Then we can
write n = 2 m, for same natural number m. And, then
n(n + 1) = 2m (2m + 1) which is clearly divisible by 2.
Next, suppose n is odd. Then n + 1 is even and we can write n + 1 = 2r, for some natural number 2.
We have n(n + 1) = (2r - 1 2r = 2r (2r - 1) which is clearly divisible by 2.
So, we can say that the natural number n(n + 1) is divisible by 2.

200
EXERCISE

SUBJECTIVE DPP # 22

1. Write down the truth value of each of the following statements.


(i) India is a democratic country.
(ii) Each prime number has exactly two factor.
(iii) 2 is an irrational number.
(iv) Jaipur is in U.P.
2. Write down the negation of the following
(i) Hindi is the mother tongue of India. (ii) India is progressing rapidly.
(iii) (a + b)3 = a 3 + 3a 2b + 3 ab2 + b3 (iv) 2 is the real part of 2 + 4i.
(v) 4 is multiple of 20. (vi) 2nd October is the birthday of Mahatma Gandhi.
(vii) Republic day of India held o 26th January.
(viii) The roots of the equation x4 + 4x3 + 6x2 + 4x + 1= 0 are equal.
(ix) New-York is in England.
3. State whether the following statements are true or false. Give reason for your answers
(i) 1 m = 100 cm (ii) The isosceles triangles have to sides equal.
(iii) The sum of two odd number is even. (iv) Three and three makes six.
(v) February has 30 days.
Prove that all n ∈ N (Q. No. 4 to 6 )
n( n + 1( 2 n + 1)
4. 12 + 2 2 + 32 + 4 2 + ..... + n2 =
6
5. 22n - 1 is divisible by 8.
6. 1 + 3 + 5 + ..... + (2n - 1) = n2.
7. Whish of the following sentences are statements :
(i) It is hot day. (ii) Qutubminar is in Lucknow.
(iii) Don’t talk, please (iv) Hurrah ! India has won the match.
(v) Rasika is a sincere girl. (vi) Will it rain today ?
(vii) 2 + 3 = 5. (viii) 5 + 7 = 10.
(ix) x + 2 = 11. (x) Every prime number has only one factor.
8. A number can be divided into three equal parts if the sum of its digits is divisible by 3. Based on the above
statement can 9875340 be divided into three equal groups.
9. Look at the following pattern :
11 1 = 11 =
11 2 = 121 =
11 3 = 1331 =
11 4 = 14641 =
11 5 = 161051 =
11 6 = 1771561 =
Is 19487171 power of 11. [Hint : Sum of digits at odd places - Sum of digits at even places = 0]
10. a2 + b2 is a prime for all whole numbers a, b.

201
ANSWER KEY

(Subjective DPP # 22)

1. (i) T (ii) T (iii) T (iv) F


2. (i) Hindi is not the mother tongue of India
(ii) India is not progressing rapidly.
(iii) (a + b)3 ≠ a3 + 3a2b + 3 ab2 + b3.
(iv) 2 is not the real part of 2 + 4i.
(v) 4 is not multiple of 20.
(vi) 2nd October is not the birthday of Mahatma Gandhi.
(vii) Republic day of India held not on 26th January.
(viii) The roots of the equation x4 + 4x3 + 6x2 + 4x + 1 = 0 are not equal.
(ix) New-York is not in England.
3. (i) T
(ii) T
(iii) T
(iv) F
(v) F
7. (i), (ii). (v), (vii), (ix)
8. Number 9 8 7 5 3 4 0 can be divided into three equal groups.
9. Yes 117 = 19487171 = [1 + 4 + 7 + 7] - [9 + 8 + 1 + 1] = 0
10. For a = 3, b = 4, a2 + b2, is not a prime.

202
MATHMETICAL MODELING

ML - 22

Definition : mathematical model is a mathematical relation that describes some real life situation. e.g. To
find the area of an equilateral triangle we can use
3
Area = (side)2
4
This formula is an example of mathematical model.

tS eps of Mathematical Modeling

Formulatin tS ta ement of the problem

Indentification of reevant na d irrelevant factors

Mathematical formulation

oS lution

Interpretation

Validation of os lution

Ex.1 A car travelled 416 kilometres on 52 litres of petrol. I have to go by same car to a place which is 96 km
away. How much petrol do I need?
Following steps include to solve the problem.
Sol. Formulation : Farther we travel, the more petrol we require, that is, the amount of petrol we need varies
directly with the distance we travel.
Petrol needed for travelling 416 km = 52 litres
Petrol needed for travelling 96 km = ?
Mathematical Description :
Let x = distance traveled
y = need of Petrol and y varies directly with x
So, y = Kx, where K is a constant.

203
I can travel 416 kilometres with 52 litres of petrol.
So, y = 52, x = 416
y 52 1
∴ K= = =
x 416 8
1
∴ y= x ....(1)
8
Equation (1) describes the relationship between the petrol needed and distance travelled.
Step 2 : We want to find the petrol we need to travel 96 kilometres. So we have to find the value of y when
96
x = 96. Putting x = 96 in (1), we have = = 12
8
Step 3 . Interpretation:
Since y = 12, we need 12 litres of petrol to travel 96 kilometres.
Step 4. Validation of the result :
This result is valid only if all the conditions remain same i.e. mileage of car track on which car is running,
gradient of the track (road), etc.
Ex.2 Suppose Rakesh has invested Rs. 20,000 at 12% simple interest per year. With the return from the
investment, he wants to buy a colour. T.V. that cost Rs. 25,000. For what period should he invest Rs. 20,000
so that he has enough money to buy a colour T.V. ?
Sol. Step 1. Formulation of the problem :
Here, we know the principal and the rte of interest in the amount Rakesh needs in addition to Rs. 20,000 to
buy the colour T.V. We have to find the number of years.
Mathematical Description :
The formula for simple interest is
Pnr.
S.I =
100
Where P = Principal
n = Number of years
r% = Rate of interest
S.I. = Interest earned
Here, the principal = Rs. 20,000
The money required by Rakesh for buying a colour T.V. = Rs. 25000
So, the interest to be earned = Rs. (25,000 - 20,000) = Rs. 5,000
The number of years for which Rs. 20,000 is deposited = n
The Interest of Rs. 20,000 for n years at the rate of 12% = S.I.
20,000 × n × 12
Then, S.I. =
100
So, S.I. = 2400 n ....(2)
Give the relationship between the number of years and interest, if Rs. 20,000 is invested an annual interest
rate of 12%. We have to find the period in which the interest earned is Rs. 5,000. Putting S.I. = 5,000 in (1),
we have
5,000 = 2400 n

204
Step 2. Solution of the problem :
Solving equations (2), we get
5000 50 1
n= = =2
2400 24 12
1
Step 3 : Interpretation : Since n = 2 and one twelfth of a year is one month Rakesh can buy a colour T.V.
12
after 2 years and one month.
Step 4. : Validation of result
We have to assume that the interest rate remains the same for the period for which we calculate the interest.
pnr
Otherwise, the formula S.I. = will into be valid. We have also assumed that the price of the colour T.V.
100
1
machine does not increase by the time 2 year.
12
Ex.3 A motor boat goes upstream on a river and covers the distance between two town on the river bank in &
hours. It covers this distance downstream in five hours. If the speed of the stream is 4 km/h, find the speed
of the boat in still water.
Sol. Step 1 : Formulation : We know the speed of the river and the time taken to cover the distance between two
places. We have to find the speed of the boat in still water.
Mathematical Description : Let us write x for the speed of the boat, t for the time taken and y for the
distance travelled. Then y = tx ....(1)
Let d be the distance between the two places. While going upstream.
The actual speed of boat = speed of the boat - speed of the river
∴ The boat is travelling against the flow of the river.
So, the speed of the boat in upstream = (x - 4) km/h.
It takes 8 hours to cover the distance between the towns upstream. So from (1), we have
d = 8 (x + 4) ....(2)
When going downstream,
The speed of the boat in downstream = (x + 4) km/h
The boat takes five hours to cover the same distance downstream, so
d = 5 (x + 4) .....(3)
From (2) and (3), we have
5(x + 4) = 8(x - 4)
Step 2. Finding the solution.
52
Solving for a in equation (4), we get x =
3
Step 3. Interpretation.
52 52
Since x = , therefore the speed of the motorboat in still water is km/h.
3 3
We have assumed that
1. The speed of the river and the boat remains constant all the time.
2. The effect of the friction between the boat and water and the friction due to air is negligible.

205
Step 4. Validation of result :
52
The speed of the motor boat is km/h and the distance between two towns,
3
y = 8(x - 4)
 52 
⇒ y = 8 − 4
 3 
 52 − 12 
⇒ y = 8 
 3 
8 × 40 320
⇒ y= =
3 3
⇒ y = 106.66 km.
Hence the distance between two towns = 106.66 km.
Ex.4 Four hundred entrance tickets were sold for a school fair. The cost of the ticket for adults was Rs. 20 and
that for students was Rs. 10. The total collection from of the sale of entrance tickets was Rs. 6000. How
many adults visited the fair ?
Sol. Step 1. Formulation of the problem
We know that
The total number of tickets sold = 400
The cost of a ticket for adults = Rs. 20
The cost of the ticket for student = Rs. 10
and the total proceedings were = Rs. 6000
Mathematical formulation :
Let the number of adults who visited the fair be x
∴ number of students visited = (400 - x)
Total amount received from adults = Rs. 20 x
Total amount received from students = Rs. 10(400 - x)
Total amount collected = Rs. 6000
∴ the model (relation) is
20x + 10(400 - x) = 6000
Step 2. Finding the solutions
20x + 4000 - 10x = 6000
⇒ 10x + 4000 = 6000
⇒ 10x = 2000
⇒ x = 200
Step 3. Interpretation of the solution
We assumed that the number of adults who visited the fair was x ∴ 200 adults visited the fair.
Step 4. Validation of the result
No. of adults who visited the fair = 200
No. of students who visited the fair = (400 - 200) = 200
∴ Total receipts = Rs (20 × 200) + Rs. (10 × 200)
= Rs. 4000 + Rs. 2000
= Rs. 6000
Thus, the total collection from 200 adults and 200 students is Rs. 6000.

206
Ex.5 The price of sugar has gone up by 40%. By what percent should a family reduce the consumption of sugar
so that the expenditure on sugar may remain the same ?
Sol. Let us consider various steps of mathematical modeling and solve this problem.
Step 1. Formulation of the problem : Price of sugar goes up by 40% i.e. if the family spends Rs. 100 on
sugar, then with the increase in price, the family will have to spend Rs. 140. But the family decides not to
increase the expenditure, instead it prefers to reduce the consumption of sugar.
Mathematical formulation :
Suppose the family consumed × kg of sugar for Rs. 100 before the price hike.
∴ the increased prince of x k of sugar is Rs. 140.
x 10x
Quantity of sugar that can be bought for Rs. 100 at the increased price = × 100 = kg
140 14
∴ reduction in quantity of sugar
10x 4x
= x− = kg
14 14
4x
400 200
Step 2. Finding the solution : Percent reduction in consumption of sugar = 14 × 100 = =
x 14 7
Step 3. Interpretation of the solution :
4
The family should reduce the consumption of sugar by 28 %
7
Step 4. Validation of result :
After increase in price of sugar, the amount of sugar bought by the family.
 4  200 
=  100 − 28  kg =  100 −  kg
 7  7 
700 − 200 500 × 2 1000
= kg = = kg
7 7×2 14
1000
The cost of kg sugar at Rs. 140 for 100 kg
14
1000 140
= Rs. × Rs. 100
14 100
Hence the result
Ex.6 Suppose company need a computer for some period of time. The company can either hire a computer for
Rs. 2,000 per month or buy one for Rs. 25,000. If the company has to use the computer for a long period, the
company will pay such a high rent, that buying a computer will be cheaper. On the other hand, if the
company has to use the computer for say, just one month, then hiring a computer will be cheaper. For the
number of months beyond which it will be cheaper to buy a computer.
Sol. Step 1. Formulation :
We know that the company can hire a computer for Rs. 2,000 per month or the company can buy the
compute for Rs. 25,000. The company has to use the computer for 0 just one month then the hiring the
computer will be cheaper. Here we have find out number of months beyond which it will be cheaper to buy
a computer.
Mathematical formulation :
Let the number of months beyond which it will be cheaper to buy a compute = x months.
Rate of hiring computer = Rs. 2,000 per month
The amount of hiring a computer for x months = Rs. 2,000 x
The cost of the compute = Rs. 25,000
The company will not have to pay more if cost of computer is less than the hiring charges for computer.

207
⇒ 25,000 < 2,000 x
Step 2. Solution :
25 ,000
⇒ <x
2 ,000
25
⇒ <x
2
Step 3. Interpretation :
25 1
If < x. The least value of x is 13 month (more than 12 ) It will be cheaper for the company to buy a
2 2
computer if it has to hire a computer 13 months or more than 13 months.
we have assumed that :
(i) The rate of hiring a computer remains same throughout the period.
(ii) After 13 months the cost of computer may not increase
Ex.7 We have given the timings of the gold medalists in the 400-metre race from the time the event was
included in the Olympics, in the table below. Construct a mathematical model relating the years and
timings. Use it to estimate the timing in the next Olympics.

Year Timing (in


seconds)
1964 52.01
1968 52.03
1972 51.08
1976 49.28
1978 48.88
1984 48.83
1988 48.65
1992 48.83
1996 48.25
2000 49.11
2004 49.41

Sol. Formulation : In the figure, the times of the gold medalist of 400 metres race are given of the Olympics
(1964 - 2004). We take 1964 as zeroth years and write 1 for 1968, 2 for 1972 and 3 for 1976. We prepare a new
table.

Year Timing (in


seconds)
0 52.01
1 52.03
2 51.08
3 49.28
4 48.88
5 48.83
6 48.65
7 48.83
8 48.25
9 49.11
10 49.41

208
The reduction in timings of gold medalist in 400 metres rave in Olympics given in the following table.

Year Timings Change in timings


0 52.01 0
1 52.03 +0.02
2 51.08 -0.95
3 49.28 +1.80
4 48.88 -0.40
5 48.3 -0.05
6 48.65 -0.18
7 48.83 +0.18
8 48.25 -0.58
9 49.11 +.086
10 49.41 +0.30

At the end of 4 years period from 1964 - 1968 the timing has increased by 0.02 second from 52.01 to 52.03
second.
At the end of second Olympic the reduction in timing is 0.95 second from 52.03 to 51.08. From the table
above we cannot find a definite relationship between the number of years and change in timing. But the
reduction is fairly steady except in the first 7th year, 9 th year and 10 th year.
The mean of the value is
0.02 − 0.95 − 1.80 − 0.40 − 0.05 − 0.18 + 0.18 − 0.58 + 0.86 + 0.30
=
10
−2.6
= = −0.26
10
Le us assume that the timings in 400 m race of Olympic reduced at the rate of 0.26 per Olympic.
Mathematical description :
We have assumed that the timings reduces at the rate of 0.26 second per Olympic.
So, the reduction in timings in the first Olympic = 52.01 - 0.26
Reduction in the second Olympic = 52.01 - 0.26 - 0.26 = 52.01 - 2 × 0.26
Reduction in the third Olympic = 52.01 - 0.26 - 0.26 - 0.26 = 52.01 - 3 × 0.26
So, the reduction in the 11th Olympic = 52.01 - 11 × 0.26
Now, we have to estimate the timings in the next (11th) Olympic i.e., 2008
But the timings in the nth Olympic will be = 52.01 - 0.26 n ....(1)
Step 2. Solution : Substituting n = 11, in (1), we get
52.01 - 0.26 × 11 = 52.01 - 2.86 = 49.15
Step 3. Interpretation : The timings for 400 m race in the next Olympic i.e., (2008) is estimated as 49.15 sec.
Step 4. Validation : Let us check if formula (1) is in agreement with the reality. Let us find the values for the
years we already know using formula (1) and compare it with known values by finding the difference.

209
EXERCISE

SUBJECTIVE DPP # 23

1. A sailor goes 8 kg downstream in 40 minutes and returns in 1 hour. Determine the speed of the sailor in still
water and the speed of the current.
2. While covering a distance of 30 km, Ajeet takes 2 hours more than Amit. If Ajeet doubles his speed,. he
would take 1 hour less than Amit. Find their speed of walking.
3. Places A and B are 80 k m apart from each other on a highway. A car starts from A and other from B at the
same speed. If they move in the same direction, they meet in 8 hours and if they more in opposite
directions, they meet in 1 hour and 20 minutes. Find the speed of the cars.
4. A bag contains one rupee, 50 paise and 25 paise coin in the ratio 5 : 6 : 7. if total amount is Rs. 390, find the
number of coins of each kind.
5. The ages of two person are in the ratio of 5 : 7. Sixteen years ago, the ratio was 3 : 5. Find their present ages.
6. Suppose Sudhir has invested Rs. 15,000 at 8% simple interest per year with the return from the investment.
He wants to buy a washing machine that costs Rs. 19,000. For what period should he invest Rs. 15,000 so
that he has enough money to buy a washing machine ?
7. A motorboat goes upstream on a river and covers the distance between two points on the riverbank in six
hours. It covers this distance downstream in five hours. If the speed of the steam is 2km/hr. find the speed
of the boat in still water.
8. Suppose you have a room of length 6m and breadth 5m. You want to cover the floor of the room with

square mosaic tiles of side 30 cm. How many tiles will you need ? Solve this by constructing a mathematical
model.
9. A travelled 432 kilometers on 48 litres of petrol in my car. I have to go by my cat to a place which is 180 km
away. how much petrol do I need ?
10. Suppose a car starts from Delhi at a speed of 70 km/h towards Chandigarh. At that instance, a motorcycle
starts from Chandigarh towards Delhi at a speed of 55 km/h. If the distance between Delhi and
Chandigarh is 250 km, after how much time will the car and motorcycle meet ?

210
ANSWER KEY

(Subjective DPP # 23)

1. Speed of sailor = 10 km/h., Speed of current = 2km/h.


2. Ajeet’s speed = 5km/h., Amit’s speed = 7.5km/h.
3. 35km/h., 25km/h.
4. No. of one rupee coins = 200 ; No. of 50 paise coin = 240 ; No. of 25 paise coin = 280
5. 40 and 56 years.
6. 3 years, 4 months
7. 22km/hr.
8. 340 tiles
9. 20 litres
10. 2 hours

211

S-ar putea să vă placă și